The full dataset viewer is not available (click to read why). Only showing a preview of the rows.
Error code: DatasetGenerationError Exception: TypeError Message: Couldn't cast array of type string to null Traceback: Traceback (most recent call last): File "/src/services/worker/.venv/lib/python3.9/site-packages/datasets/builder.py", line 1831, in _prepare_split_single writer.write_table(table) File "/src/services/worker/.venv/lib/python3.9/site-packages/datasets/arrow_writer.py", line 644, in write_table pa_table = table_cast(pa_table, self._schema) File "/src/services/worker/.venv/lib/python3.9/site-packages/datasets/table.py", line 2272, in table_cast return cast_table_to_schema(table, schema) File "/src/services/worker/.venv/lib/python3.9/site-packages/datasets/table.py", line 2223, in cast_table_to_schema arrays = [ File "/src/services/worker/.venv/lib/python3.9/site-packages/datasets/table.py", line 2224, in <listcomp> cast_array_to_feature( File "/src/services/worker/.venv/lib/python3.9/site-packages/datasets/table.py", line 1795, in wrapper return pa.chunked_array([func(chunk, *args, **kwargs) for chunk in array.chunks]) File "/src/services/worker/.venv/lib/python3.9/site-packages/datasets/table.py", line 1795, in <listcomp> return pa.chunked_array([func(chunk, *args, **kwargs) for chunk in array.chunks]) File "/src/services/worker/.venv/lib/python3.9/site-packages/datasets/table.py", line 2052, in cast_array_to_feature casted_array_values = _c(array.values, feature.feature) File "/src/services/worker/.venv/lib/python3.9/site-packages/datasets/table.py", line 1797, in wrapper return func(array, *args, **kwargs) File "/src/services/worker/.venv/lib/python3.9/site-packages/datasets/table.py", line 2086, in cast_array_to_feature return array_cast( File "/src/services/worker/.venv/lib/python3.9/site-packages/datasets/table.py", line 1797, in wrapper return func(array, *args, **kwargs) File "/src/services/worker/.venv/lib/python3.9/site-packages/datasets/table.py", line 1948, in array_cast raise TypeError(f"Couldn't cast array of type {_short_str(array.type)} to {_short_str(pa_type)}") TypeError: Couldn't cast array of type string to null The above exception was the direct cause of the following exception: Traceback (most recent call last): File "/src/services/worker/src/worker/job_runners/config/parquet_and_info.py", line 1456, in compute_config_parquet_and_info_response parquet_operations = convert_to_parquet(builder) File "/src/services/worker/src/worker/job_runners/config/parquet_and_info.py", line 1055, in convert_to_parquet builder.download_and_prepare( File "/src/services/worker/.venv/lib/python3.9/site-packages/datasets/builder.py", line 894, in download_and_prepare self._download_and_prepare( File "/src/services/worker/.venv/lib/python3.9/site-packages/datasets/builder.py", line 970, in _download_and_prepare self._prepare_split(split_generator, **prepare_split_kwargs) File "/src/services/worker/.venv/lib/python3.9/site-packages/datasets/builder.py", line 1702, in _prepare_split for job_id, done, content in self._prepare_split_single( File "/src/services/worker/.venv/lib/python3.9/site-packages/datasets/builder.py", line 1858, in _prepare_split_single raise DatasetGenerationError("An error occurred while generating the dataset") from e datasets.exceptions.DatasetGenerationError: An error occurred while generating the dataset
Need help to make the dataset viewer work? Make sure to review how to configure the dataset viewer, and open a discussion for direct support.
source_file
string | problem_type
string | problem
string | solution
string | remark
string | figures
list |
---|---|---|---|---|---|
./raw_volume-zh/volume1/chapter1.tex
|
proof
|
例2. 设 $A$ 是两个整数平方差的集合,即 $A=\left\{x\mid x=m^{2}-n^{2},\,m,n\in\mathbf{Z}\right\}$.证明:
(1) 若 $s,\ t\in A$ ,则 $s t\in A.$
(2) 若 $s,\ t\in A,\ t\neq0$,则$\frac{s}{t}=p^{2}-q^{2}.$ ,其中 $p,q$ 是有理数.
|
分析: 想办法将 $st$ 表示为两个整数的平方差.
证明: (1)由 $s,t\in A$ ,可设
$$
s=m_{1}^{2}-n_{1}^{2}\,,\;t=m_{2}^{2}-n_{2}^{2}\,,
$$
其中 $m_{1}, n_{1}, m_{2}, n_{2}$ 均为整数.
是
$$
\begin{array}{l}{{s t=(m_{1}^{2}-n_{1}^{2})\,(m_{2}^{2}-n_{2}^{2})}}\\ {{=m_{1}^{2}m_{2}^{2}+2m_{1}m_{2}n_{1}n_{2}+n_{1}^{2}n_{2}^{2}-m_{1}^{2}n_{2}^{2}-2m_{1}m_{2}n_{1}n_{2}-m_{2}^{2}n_{1}^{2}}}\\ {{=(m_{1}m_{2}+n_{1}n_{2})^{2}-(m_{1}n_{2}+m_{2}n_{1})^{2}\,,}}\end{array}
$$
即 $st$ 是两个整数的平方差,故 $s t\in A.$
(2) 由于 $s, t \in A$ ,由(1)知 $s t\in A.$ 令 $s t=m^{2}-n^{2},m,n$ 是整数.
$t\neq 0$,因此
$$
{\frac{s}{t}}\,={\frac{s t}{t^{2}}}=\left({\frac{m}{t}}\right)^{2}-\left({\frac{n}{t}}\right)^{2}.
$$
而 ${\frac{m}{t}},{\frac{n}{t}}$ 均为有理数,故命题得证.
|
[] |
|
./raw_volume-zh/volume1/chapter1.tex
|
proof
|
例3. 设函数 $f(x)=x^{2}+a x+b\ (a,\,b\in\mathbb{R})$ ,集合 $A=\{x\mid x=f(x)$ $x\in\mathbb{R})\,,\,B=\{x\mid x=f(f(x))\,,\,x\in\mathbb{R}\}.$
(1)证明: $A\subset B$ ;
(2)当 $A=\{-1,\,3\}$ 时,求集合 $B$ .
|
分析: 欲证 $A\subseteq B$, 只需证明方程 $x=f(x)$ 的根必是方程 $x=f(f(x))$ 的根.
解: (1)对任意的 $x_{0}\in A$ ,有 $x_{0}=f(x_{0}), \, x_0 \in \mathbb{R}.$
于是
$$
f(f(x_{0}))=f(x_{0})=x_{0}.
$$
故 $x_{0}\in B$ ,所以 $A\subseteq B$.
(2)因 $A=\{-1,\,3\}$ ,所以
$$
\begin{align*}
\left\{
\begin{aligned}
(-\,1)^{2}+a*(-\,1)+b=-\,1, \\
3^{2}+a*3+b=3,
\end{aligned}
\right.
\end{align*}
$$
解之得 $a=-1$, $b=-3$,故 $f(x)=x^{2}-x-3.$ 由 $x=f(f(x))$ 得
$(x^2-x-3)^2-(x^2-x-3)-x-3= 0.$
即
$$
(x^{2}-2x-3)\,(x^{2}-3)\,=\,0\,.
$$
解得 $x=-1,\,3,\,\pm{\sqrt{3}}.$
所以 $,B=\{-1,\ 3,-{\sqrt{3}}\,,{\sqrt{3}}\}$ .
|
[] |
|
./raw_volume-zh/volume1/chapter1.tex
|
proof
|
例6. 求所有的角 $\alpha$, 使得集合
$$
\{\sin \alpha, \sin 2 \alpha, \sin 3 \alpha\}=\{\cos \alpha, \cos 2 \alpha, \cos 3 \alpha\} .
$$
|
解: 设 $\alpha \in[0,2 \pi)$. 由已知得
$$
\sin \alpha+\sin 2 \alpha+\sin 3 \alpha=\cos \alpha+\cos 2 \alpha+\cos 3 \alpha,
$$
即
$$
\begin{aligned}
2 \sin 2 \alpha \cos \alpha+\sin 2 \alpha & =2 \cos 2 \alpha \cos \alpha+\cos 2 \alpha, \\
\sin 2 \alpha(2 \cos \alpha+1) & =\cos 2 \alpha(2 \cos \alpha+1) .
\end{aligned}
$$
所以 $\sin 2 \alpha=\cos 2 \alpha$ 或 $\cos \alpha=-\frac{1}{2}$ (舍去).
从而
$$
\begin{aligned}
0 & =\sin 2 \alpha-\cos 2 \alpha \\
& =\sin 2 \alpha-\sin \left(\frac{\pi}{2}-2 \alpha\right) \\
& =2 \cos \frac{\pi}{4} \sin \left(2 \alpha-\frac{\pi}{4}\right) .
\end{aligned}
$$
于是 $\alpha=\frac{\pi}{8}, \frac{5 \pi}{8}, \frac{9 \pi}{8}, \frac{13 \pi}{8}$.
又 $\sin \alpha \sin 2 \alpha \sin 3 \alpha=\cos \alpha \cos 2 \alpha \cos 3 \alpha$, 且 $\sin 2 \alpha=\cos 2 \alpha$, 因此
$$
\begin{aligned}
\cos 4 \alpha & =0, \\
\alpha=\frac{(2 k-1) \pi}{8}, k & =1,2, \cdots, 8 .
\end{aligned}
$$
经验证, $\alpha=\frac{k \pi}{4}+\frac{\pi}{8}(k \in \mathbf{Z})$ 满足题意.
说明: 元素之和(积)相等只是两个集合相等的必要条件, 因此这里还必须检查集合的元素是否互异.
|
[] |
|
./raw_volume-zh/volume1/chapter1.tex
|
proof
|
例7. 设 $S$ 为非空数集, 且满足: (i) $2 \notin S$; (ii) 若 $a \in S$, 则 $\frac{1}{2-a} \in S$. 证明:
(1) 对一切 $n \in \mathbf{N}^*, n \geqslant 3$, 有 $\frac{n}{n-1} \notin S$;
(2) $S$ 或者是单元素集,或者是无限集.
|
分析: 对于 (1), 因为 $n \in \mathbf{N}^*$, 可以考虑采用数学归纳法.
证明: (1) 因为 $S$ 非空, 所以存在 $a \in S$, 且 $a \neq 2$.
我们用数学归纳法证明下面的命题:
若 $a \in S$, 则对 $k \in \mathbf{N}^*, \frac{(k-1)-(k-2) a}{k-(k-1) a} \in S$, 且 $a \neq \frac{k+1}{k}$.
当 $k=1$ 时, 显然 $a \in S$, 且 $a \neq 2$ 成立.
设 $k \in \mathbf{N}^*, \frac{(k-1)-(k-2) a}{k-(k-1) a} \in S$ 且 $a \neq \frac{k+1}{k}$ 成立.
由 (ii) 得
$$
\begin{gathered}
\frac{1}{2-\frac{(k-1)-(k-2) a}{k-(k-1) a}} \in S, \\
\frac{k-(k-1) a}{(k+1)-k a} \in S .
\end{gathered}
$$
化简得
$$
\frac{k-(k-1) a}{(k+1)-k a} \in S \text {. }
$$
又 $\frac{k-(k-1) a}{(k+1)-k a} \neq 2$, 所以 $a \neq \frac{k+2}{k+1}$.
综上, 由归纳原理知, 对 $k \in \mathbf{N}^*$ 命题成立.
从而, 对一切 $n \in \mathbf{N}^*, n \geqslant 3$, $\frac{n}{n-1} \notin S$ 成立.
(2) 由 (1) 知, 若 $a \in S, a \neq \frac{m}{m-1}\left(m \in \mathbf{N}^*, m \geqslant 3\right)$, 则 $\frac{(m-1)-(m-2) a}{m-(m-1) a} \in S$.
所以, 当 $n \geqslant 2, m \geqslant 2, m \neq n$ 时,
$$
\begin{aligned}
& \frac{(n-1)-(n-2) a}{n-(n-1) a}=\frac{(m-1)-(m-2) a}{m-(m-1) a} \\
& \Leftrightarrow m(n-1)-(n-1)(m-1) a-m(n-2) a+(m-1)(n-2) a^2 \\
& =n(m-1)-(n-1)(m-1) a-n(m-2) a+(n-1)(m-2) a^2 \\
& \Leftrightarrow n-m+2(m-n) a+(n-m) a^2=0 \\
& \Leftrightarrow(n-m)\left(1-2 a+a^2\right)=0 \\
& \Leftrightarrow a=1 (\text { 因为 } n \neq m) .
\end{aligned}
$$
因为 $\mathbf{N}^*$ 是无限集, 所以 $S$ 或者为单元素集 $\{1\}$ (当且仅当 $a=1$ ), 或者为无限集.
|
[] |
|
./raw_volume-zh/volume1/chapter1.tex
|
proof
|
例8. 用 $\sigma(S)$ 表示非空的整数集合 $S$ 的所有元素的和.
设 $A=\left\{a_1\right.$, $\left.a_2, \cdots, a_{11}\right\}$ 是正整数的集合, 且 $a_1<a_2<\cdots<a_{11}$; 又设对每个正整数 $n \leqslant$ 1500 , 都存在 $A$ 的子集 $S$, 使得 $\sigma(S)=n$. 求 $a_{10}$ 的最小可能值.
|
分析: 要求 $a_{10}$ 的最小值, 显然应使 $\sigma(A)=1500$. 又由题设, 应使 $a_{11}$ 尽可能大, 且前 10 个数之和不小于 750 , 故取 $a_{11}=750$. 考虑整数的二进制表示, 由 $1+2+\cdots+2^7=255$ 知, 前 8 个数应依次为 $1,2,4,8,16,32,64,128$. 这时 $a_9+a_{10}=495$, 从而有 $a_{10}=248$.
解: 取 $A_0=\{1,2,4,8,16,32,64,128,247,248,750\}$, 易知 $A_0$ 满足题目要求,且 $a_{10}=248$. 故 $a_{10}$ 的最小可能值不超过 248 .
另一方面, $a_{10}$ 不可能比 248 更小.
这是因为前 10 个数之和不能小于 750 , 否则设 $\sum_{i=1}^{10} a_i=m, m<750$, 则 $a_{11}=1500-m$, 对 $n \in(m, 1500-m)$, 显然不存在 $A$ 的子集 $S$, 使 $\sigma(S)=n$. 因 $1+2+\cdots+2^?=255$, 由整数的二进制表示知, 其前 8 个数之和最大为 255 . 故 $a_9+a_{10}$ 的最小可能值为 495 , 从而 $a_{10}$ 至少为 248 .
综上知, $a_{10}$ 的最小可能值为 248 .
说明: 本例采用了构造法.
直接构造一个符合题设的 $A_0$, 然后证明 $A_0$ 具有所要求的性质.
这种方法在解有关集合和组合的问题中经常用到.
|
[] |
|
./raw_volume-zh/volume1/chapter1.tex
|
proof
|
例9. 设 $S$ 是由 $2 n$ 个人组成的集合.
求证: 其中必定有两个人,他们的公共朋友的个数为偶数.
|
证明: 用反证法: 设 $S$ 为一个由 $2 n$ 个人组成的集合, $S$ 中每两个人的公共朋友数为奇数.
对 $S$ 中的任意一个人 $A$, 记 $M=\left\{F_1, \cdots, F_k\right\}$ 为 $A$ 的朋友集, 可以证明: 对每个 $A, k$ 都为偶数.
事实上, 对每个 $F_i \in M$, 考虑他在 $M$ 中的朋友数, 所有这 $k$ 个 $F_i$ 的这些朋友数之和为偶数 (因为朋友是相互的), 而对 $A 、 F_i$ 而言, 其公共朋友数为奇数, 故每个 $F_i$ 的这样的朋友数为奇数, 故 $k$ 为偶数.
设 $k=2 m$, 现在考虑每个 $F_i \in M$, 他的所有朋友集不包括 $A$, 但不局限于 $M$ 中, 他的这样的朋友数为奇数 (因为 $F_i$ 的朋友数为偶数, 而 $A$ 不算在内). 因此,所有 $2 m$ 个这样的朋友集的元素个数之和为偶数.
从而在 $2 n-1$ 个人 ( $A$ 除外) 中, 必有一个人在偶数个这样的朋友集中出现, 他与 $A$ 的公共朋友数为偶数.
这个矛盾表明: 有两个 $S$ 中的人,他们的公共朋友数为偶数.
说明: 上述解法采用了奇偶性分析来“制造”矛盾.
|
[] |
|
./raw_volume-zh/volume1/chapter1.tex
|
proof
|
例10. 设 $n$ 是大于 1 的正整数,证明存在一个集合 $A \varsubsetneqq\{1,2, \cdots, n\}$, 使得
(1) $|A| \leqslant 2[\sqrt{n}]+1$;
(2) $\{|x-y| \mid x, y \in A, x \neq y\}=\{1,2, \cdots, n-1\}$.
|
分析: 由 $|A| \leqslant 2[\sqrt{n}]+1$ 想到, 设 $n=k^2+b, 0 \leqslant b \leqslant 2 k$.
证明: 设 $n=k^2+b, 0 \leqslant b \leqslant 2 k$.
(1) 当 $b \leqslant k$ 时,考虑集合
$$
\begin{gathered}
A=\left\{1,2, \cdots, k, 2 k, 3 k, \cdots, k^2, k^2+b\right\} \varsubsetneqq\{1,2, \cdots, n\}, \\
|A|=2 k \leqslant 2[\sqrt{n}]+1=2 k+1,
\end{gathered}
$$
而易知 $\{|x-y| \mid x, y \in A, x \neq y\}=\left\{1,2, \cdots, k^2+b-1\right\}$.
(2) 当 $b>k$ 时,考虑集合
$$
A=\left\{1,2, \cdots, k, 2 k, 3 k, \cdots, k^2, k^2+k, k^2+b\right\} \varsubsetneqq\{1,2, \cdots, n\},
$$
同样有
$$
|A|=2 k+1 \leqslant 2[\sqrt{n}]+1 \text {, }
$$
且 $\{|x-y| \mid x, y \in A, x \neq y\}=\left\{1,2, \cdots, k^2+b-1\right\}$.
综上知, 原命题成立.
|
[] |
|
./raw_volume-zh/volume1/chapter2.tex
|
proof
|
例6. 设 $n \in \mathbf{N}$, 且 $n \geqslant 15, A 、 B$ 都是 $\{1,2, \cdots, n\}$ 的真子集, $A \cap B=\varnothing$, 且 $\{1,2, \cdots, n\}=A \cup B$. 证明: $A$ 或者 $B$ 中必有两个不同数的和为完全平方数.
|
证明:由题设, $\{1,2, \cdots, n\}$ 的任何元素必属于且只属于它的真子集 $A$ 、$B$ 之一.
假设结论不真, 则存在如题设的 $\{1,2, \cdots, n\}$ 的真子集 $A 、B$, 使得无论是 $A$ 还是 $B$ 中的任何两个不同的数的和都不是完全平方数.
不妨设 $1 \in A$, 则 $3 \notin A$. 否则 $1+3=2^2$, 与假设矛盾, 所以 $3 \in B$. 同样, $6 \notin B$, 所以 $6 \in A$. 这时 $10 \notin A$, 即 $10 \in B$. 因 $n \geqslant 15$, 而 15 或者在 $A$ 中, 或者在 $B$ 中,但当 $15 \in A$ 时, 因 $1 \in A, 1+15=4^2$, 矛盾; 当 $15 \in B$ 时, 因 $10 \in B, 10+15=5^2$, 仍然矛盾.
因此假设不真, 即 $A$ 或者 $B$ 中必有两个不同数的和为完全平方数.
说明由 $A 、 B$ 地位对称, 在上面的解法中我们采用了“不妨设 $1 \in A$ ”这种技巧,有效简化了解题过程.
例6 实际上给出了一个关于集合的方程组:
$$
\left\{\begin{array}{l}
A \cup B=\{1,2, \cdots, n\}, \\
A \cap B=\varnothing .
\end{array}\right.
$$
如果交换 $A 、 B$ 算两组解 (有序解), 那么这个方程组有多少组有序解呢?
设 $U=\{1,2, \cdots, n\}$, 由 $A \cup B=U, A \cap B=\varnothing$,知 $A$ 与 $B$ 互补, 对于 $A \subseteq U$, 可取 $B=\complement_U A$. 故上述集合方程的有序解的个数为 $2^n$.
|
[] |
|
./raw_volume-zh/volume1/chapter2.tex
|
proof
|
例8. 设集合 $S$ 含有 $n$ 个元素, $A_1, A_2, \cdots, A_k$ 是 $S$ 的不同子集, 它们两两的交集非空,而 $S$ 的其他子集不能与 $A_1, A_2, \cdots, A_k$ 都相交.
求证: $k=2^{n-1}$.
|
分析: $S$ 有 $2^n$ 个子集, 将两个互为补集的子集作为一组, 则可将 $2^n$ 个子集分成 $2^{n-1}$ 个组, 记为 $\left\{A_i^{\prime}, B_i^{\prime}\right\}, i=1,2, \cdots, 2^{n-1}$, 显然 $A_i$ 只能选取每组中的一个子集.
证明: 设 $a \in S$. 因为 $|S|=n$, 故 $S$ 的子集中含 $a$ 的子集有 $2^{n-1}$ 个.
显然它们两两的交非空.
所以, $k \geqslant 2^{n-1}$.
又可将 $S$ 的 $2^n$ 个子集分成 $2^{n-1}$ 组, 每组有两个集合, 它们互为补集.
若 $k>2^{n-1}$, 则必有两个集合 $A_i 、 A_j(i \neq j)$ 来自上述同一组, 但 $A_i \cap A_j=\varnothing$,与题意不符.
所以, $k=2^{n-1}$.
|
[] |
|
./raw_volume-zh/volume1/chapter2.tex
|
proof
|
例9. 有 1987 个集合,每个集合有 45 个元素,任意两个集合的并集有 89 个元素, 问此 1987 个集合的并集有多少个元素?
|
分析: 由每个集合有 45 个元素, 且任意两个集合的并集有 89 个元素知, 任意两个集合有且只有一个公共元素.
解显然可以由题设找到这样的 1987 个集合, 它们都含有一个公共元素 $a$,而且每两个集合不含 $a$ 以外的公共元素.
下面,我们来排除其他可能性.
由任意两个集合的并集有 89 个元素可知, 1987 个集合中的任意两个集合有且只有一个公共元素, 则容易证明这 1987 个集合中必有一个集合 $A$ 中的元素 $a$ 出现在 $A$ 以外的 45 个集合中, 设为 $A_1, A_2, \cdots, A_{45}$, 其余的设为 $A_{46}, A_{47}, \cdots, A_{1986}$.
设 $B$ 为 $A_{46}, \cdots, A_{1986}$ 中的任一个集合, 且 $a \notin B$, 由题设 $B$ 和 $A, A_1$, $A_2, \cdots, A_{45}$ 都有一个公共元素, 且此 46 个元素各不相同, 故 $B$ 中有 46 个元素,与题设矛盾.
所以这 1987 个集合中均含有 $a$.
故所求结果为 $1987 \times 44+1=87429$, 即这 1987 个集合的并集有 87429 个元素.
说明: 在这里我们先设计一种符合题设的特殊情形, 然后再排除其他可能的情形, 从而达到解题目的.
这是一种“先猜后证”的解题策略.
|
[] |
|
./raw_volume-zh/volume1/chapter2.tex
|
proof
|
例10. 设 $A$ 是集合 $S=\{1,2, \cdots, 1000000\}$ 的一个恰有 101 个元素的子集.
证明: 在 $S$ 中存在数 $t_1, t_2, \cdots, t_{100}$, 使得集合
$$
A_j=\left\{x+t_j \mid x \in A\right\}, j=1,2, \cdots, 100
$$
中, 每两个的交集为空集.
|
分析: 先弄清楚在什么情况下 $A_i \cap A_j \neq \varnothing$. 设 $a \in A_i \cap A_j$, 则 $a=x+ t_i==y+t_j, x, y \in A$. 于是 $t_i-t_j=y-x$. 这说明选取 $t_1, t_2, \cdots, t_{100}$ 时, 只要保证其中任意两个之差不等于 $A$ 中任二元素之差即可.
证明: 考虑集合 $D=\{x-y \mid x, y \in A\}$, 则
$$
|D| \leqslant 101 \times 100+1=10101 \text {. }
$$
若 $A_i \cap A_j \neq \varnothing$, 设 $a \in A_i \cap A_j$, 则 $a=x+t_i, a=y+t_j$, 其中 $x, y \in A$, 则 $t_i-t_j=y-x \in D$.
若 $t_i-t_j \in D$, 即存在 $x, y \in A$, 使得 $t_i-t_j=y-x$, 从而 $x+t_i=y+t_j$, 即 $A_i \cap A_j \neq \varnothing$.
所以, $A_i \cap A_j \neq \varnothing$ 的充要条件是 $t_i-t_j \in D$. 于是, 我们只需在集 $S$ 中取出 100 个元素, 使得其中任意两个的差都不属于 $D$.
下面用递推方法来取出这 100 个元素.
先在 $S$ 中任取一个元素 $t_1$, 再从 $S$ 中取一个 $t_2$, 使得 $t_1 \notin t_2+D=\{t_2+ x \mid x \in D\}$, 这是因为取定 $t_1$ 后, 至多有 10101 个 $S$ 中的元素不能作为 $t_2$, 从而在 $S$ 中存在这样的 $t_2$, 若已有 $k(\leqslant 99)$ 个 $S$ 中的元素 $t_1, t_2, \cdots, t_k$ 满足要求, 再取 $t_{k+1}$, 使得 $t_1, \cdots, t_k$ 都不属于 $t_{k+1}+D=\left\{t_{k+1}+x \mid x \in D\right\}$. 这是因为 $t_1, t_2, \cdots, t_k$ 取定后, 至多有 $10101 k \leqslant 999999$ 个 $S$ 中的数不能作为 $t_{k+1}$, 故在 $S$ 中存在满足条件的 $t_{k+1}$. 所以, 在 $S$ 中存在 $t_1, t_2, \cdots, t_{100}$, 其中任意两个的差都不属于 $D$.
综上所述,命题得证.
说明条件 $|S|=10^{\varepsilon}$ 可以改小一些.
一般地, 我们有如下更强的结论: 若 $A$ 是 $S=\{1,2, \cdots, n\}$ 的一个 $k$ 元子集, $m$ 为正整数, 且 $m$ 满足条件 $n>(m-1) \cdot\left(C_k^2+1\right)$, 则存在 $S$ 中的元素 $t_1, \cdots, t_m$, 使得 $A_j=\left\{x+t_j \mid x \in A \right\}, j=1, \cdots, m$ 中任意两个的交集为空集.
|
[] |
|
./raw_volume-zh/volume1/chapter3.tex
|
proof
|
例4. 设 $a_1, a_2, \cdots, a_n$ 为 $1,2, \cdots, n$ 的一个排列, $f_k=\mid\{a_i \mid a_i<a_k, i>k\}\left|, g_k=\right|\left\{a_i \mid a_i>a_k, i<k\right\} \mid$, 其中 $k=1,2, \cdots, n$. 证明:
$$
\sum_{k=1}^n g_k=\sum_{k=1}^n f_k
$$
|
分析:一般来说 $f_k \neq g_k$, 且分别计算 $f_k 、 g_k$ 是困难的.
令 $A_k=\left\{a_i\mid a_i<a_k, i>k\right\}$, 对 $A_k$ 换一种写法: $A_k=\left\{\left(a_i, a_k\right) \mid a_i<a_k, i>k\right\}$, 显然是合理的.
易知 $k \neq k^{\prime}$ 时, $A_k \cap A_k{ }^{\prime}=\varnothing$. 所以, $\sum_{k=1}^n f_k=\left|A_1\right|+\left|A_2\right|+\cdots+\left|A_n\right|=\left|A_1 \cup A_2 \cup \cdots \cup A_n\right|=\left|\left\{\left(a_i, a_j\right) \mid a_i<a_j, i>j\right\}\right|$.
证明考虑集合 $A=\left\{\left(a_i, a_j\right) \mid a_i<a_j, i>j\right\}$ 的元素的数目 $|A|$. 一方面, 固定 $a_j$ 时, $a_i$ 的个数为 $f_j$. 所以
$$
|A|=\sum_{j=1}^n f_j .
$$
另一方面, 固定 $a_i$ 时, $a_j$ 的个数为 $g_i$, 所以
$$
|A|=\sum_{i=1}^n g_i
$$
所以, $\sum_{k=1}^n g_k=\sum_{k=1}^n f_k$.
说明在这里, 我们没有直接证明 $\sum_{k=1}^n g_k=\sum_{k=1}^n f_k$, 而是引人一个中间量 $|A|=\left|\left\{\left(a_i, a_j\right) \mid a_i<a_j, i>j\right\}\right|$ 来过渡.
|
[] |
|
./raw_volume-zh/volume1/chapter3.tex
|
proof
|
例5. 设 $p \geqslant 5$ 是一个素数, $S=\{1,2, \cdots, p-1\}, A=\{a \mid a \in S, a^{p-1} \not \equiv 1\left(\bmod p^2\right) \}$. 证明 : $|A| \geqslant \frac{p-1}{2}$.
|
分析:如果 $1 \leqslant a \leqslant p-1$, 显然 $1 \leqslant p-a \leqslant p-1$. 将 $a$ 与 $p-a$ 配对, 如果 $a^{p-1}$ 与 $(p-a)^{p-1}$ 模 $p^2$ 不同余, 则结论成立.
证明设 $a \in S$, 则 $p-a \in S$. 由二项式定理,有
$$
(p-a)^{p-1}-a^{p-1} \equiv-(p-1) a^{p-2} \cdot p \not \equiv 0\left(\bmod p^2\right) .
$$
于是, $a$ 和 $p-a$ 中至少有一个在 $A$ 中, 从而有
$$
|A| \geqslant \frac{p-1}{2} \text {. }
$$
|
[] |
|
./raw_volume-zh/volume1/chapter3.tex
|
proof
|
例6. $A_1, A_2, \cdots, A_{30}$ 是集合 $\{1,2, \cdots, 2003\}$ 的子集,且 $\left|A_i\right| \geqslant 660$, $i=1,2, \cdots, 30$. 证明: 存在 $i \neq j, i, j \in\{1,2, \cdots, 30\}$, 使得 $\left|A_i \cap A_j\right| \geqslant$ 203.
|
证明:不妨设每一个 $A_i$ 的元素都为 660 个(否则去掉一些元素). 作一个集合、元素的关系表: 表中每一行(除最上面的一行外)分别表示 30 个集合 $A_1, A_2, \cdots, A_{30}$, 表的 $n$ 列 (最左面一列除外) 分别表示 2003 个元素 1 , $2, \cdots, 2003$. 若 $i \in A_j(i=1,2, \cdots, 2003,1 \leqslant j \leqslant 30)$, 则在 $i$ 所在的列与 $A_j$ 所在行的交叉处写上 1 , 若 $i \notin A_j$, 则写上 0 .
\begin{tabular}{c|ccccc}
& 1 & 2 & 3 & $\cdots$ & 2003 \\
\hline$A_1$ & $\times$ & $\times$ & $\times$ & $\cdots$ & $\times$ \\
$A_2$ & $\times$ & $\times$ & $\times$ & $\cdots$ & $\times$ \\
$\cdots$ & $\times$ & $\times$ & $\times$ & $\cdots$ & $\times$ \\
$A_{30}$ & $\times$ & $\times$ & $\times$ & $\cdots$ & $\times$
\end{tabular}
表中每一行有 660 个 1 , 因此共有 $30 \times 660$ 个 1 . 设第 $j$ 列有 $m_j$ 个 1 $(j=1,2, \cdots, 2003)$, 则
$$
\sum_{j=1}^{2003} m_j=30 \times 660 .
$$
由于每个元素 $j$ 属于 $\mathrm{C}_{m_j}^2$ 个交集 $A_s \cap A_t$, 因此
$$
\sum_{j=1}^{2003} \mathrm{C}_{m_j}^2=\sum_{1 \leqslant s<t \leqslant 30}\left|A_s \cap A_t\right| .
$$
由柯西不等式, 得
$$
\begin{aligned}
\sum_{j=1}^{2003} \mathrm{C}_{m_j}^2 & =\frac{1}{2}\left(\sum_{j=1}^{2003} m_j^2-\sum_{j=1}^{2003} m_j\right) \\
& \geqslant \frac{1}{2}\left(\frac{1}{2003}\left(\sum_{j=1}^{2003} m_j\right)^2-\sum_{j=1}^{2003} m_j\right) .
\end{aligned}
$$
所以, 必有 $i \neq j$, 满足
$$
\begin{aligned}
\left|A_i \cap A_j\right| \geqslant & \frac{1}{\mathrm{C}_{30}^2} \times \frac{1}{2}\left(\frac{1}{2003}\left(\sum_{j=1}^{2003} m_j\right)^2-\sum_{j=1}^{2003} m_j\right) \\
= & \frac{660(30 \times 660-2003)}{29 \times 2003}>202, \\
& \left|A_i \cap A_j\right| \geqslant 203 .
\end{aligned}
$$
从而说明本题中所作的表,称为元素、集合从属关系表.
它在讨论涉及多个集合的问题时非常有用.
|
[] |
|
./raw_volume-zh/volume1/chapter3.tex
|
proof
|
例7. 设 $n, k \in \mathbf{N}^*$, 且 $k \leqslant n$. 并设 $S$ 是含有 $n$ 个互异实数的集合, $T=\left\{a \mid a=x_1+x_2+\cdots+x_k, x_i \in S, x_i \neq x_j(i \neq j), 1 \leqslant i, j \leqslant k\right\}$. 求证: $|T| \geqslant k(n-k)+1$.
|
分析:设 $S_n=\left\{s_1, s_2, \cdots, s_{n-1}, s_n\right\}$, 且 $s_1<s_2<\cdots<s_{n-1}<s_n$. 作 $S_n$ 的子集 $S_{n-1}=\left\{s_1, s_2, \cdots, s_{n-1}\right\}$, 设 $S_{n-1} 、 S_n$ 分别对应 $T_{n-1} 、 T_n$. 对固定的 $k (k \leqslant n-1)$, 由 $s_{n-1}+s_{n-2}+\cdots+s_{n-k}<s_n+s_{n-2}+\cdots+s_{n-k}<s_n+s_{n-1}+s_{n-3}+\cdots+s_{n-k}<\cdots<s_n+s_{n-1}+\cdots+s_{n-k+1}$, 知 $\left|T_n\right| \geqslant\left|T_{n-1}\right|+k$. 而 $k(n-k)+ 1=k(n-1-k)+k+1$, 这提示我们对 $n$ 进行归纳证明.
证明设 $s_1<s_2<\cdots<s_n$ 是 $S$ 的 $n$ 个元素.
对元素数目 $n$ 使用数学归纳法.
首先, 当 $k=1$ 和 $k=n$ 时, 结论显然成立.
设 $k \leqslant n-1$, 且结论对 $S_0=\left\{s_1, s_2, \cdots, s_{n-1}\right\}$ 成立, 并设 $T_0$ 是当把 $S$ 换成 $S_0$ 时与 $T$ 相应的集合.
于是有
$$
\left|T_0\right| \geqslant k(n-k-1)+1 .
$$
令 $x=s_n+s_{n-1}+\cdots+s_{n-k}$, 并令
$$
y_i=x-s_{n-i}, i=0,1, \cdots, k .
$$
显然 $y_i \in T$, 且有 $y_0<y_1<y_2<\cdots<y_k$. 因为 $y_0$ 是 $T_0$ 中的最大元素, 所以
$$
y_i \in T, y_i \notin T_0, i=1,2, \cdots, k .
$$
故有
$$
|T| \geqslant\left|T_0\right|+k \geqslant k(n-k-1)+1+k=k(n-k)+1 .
$$
这就完成了归纳证明.
|
[] |
|
./raw_volume-zh/volume1/chapter3.tex
|
proof
|
例10. 设 $a_1, a_2, \cdots, a_{20}$ 是 20 个两两不同的整数, 且集合 $ \{a_i+a_j \mid 1 \leqslant i \leqslant j \leqslant 20\}$ 中有 201 个不同的元素.
求集合 $\left\{\left|a_i-a_j\right| \mid 1 \leqslant i<j \leqslant 20\right\}$ 中不同元数个数的最小可能值.
|
分析:从 $a_1, a_2, \cdots, a_{20}$ 中任取两个(可以相等) 相加, 至多有 $\mathrm{C}_{20}^2+ 20=210$ 个不同的和, 由题设知, 所有 $a_i+a_j$ 中有些和数相等.
另一方面, 应使所有的 $\left|a_i-a_j\right|$ 中出现尽可能多的相等的情况.
由此, 可构造一组特殊的数: $a_1^{\prime}, a_2^{\prime}, \cdots, a_{20}^{\prime}$.
解所给集合的元素个数的最小值为 100 .
首先, 令 $a_i=10^{11}+10^i, a_{10+i}=10^{11}-10^i, i=1,2, \cdots, 10$. 则 $\{a_i+a_j \mid 1 \leqslant i \leqslant j \leqslant 20\}$ 中共有 $(20+19+\cdots+1)-10+1=201$ 个不同的元素, 而 $\left\{\left|a_i-a_j\right| \mid 1 \leqslant i<j \leqslant 20\right\}=\left\{2 \times 10^i \mid i=1,2, \cdots, 10\right\} \bigcup\{\left|10^i \pm 10^j\right| \mid 1 \leqslant i<j \leqslant 10\}$ 共有 $10+2 \mathrm{C}_{10}^2=100$ 个不同的元素.
下面用反证法证明: 所给集合的不同元素的个数不小于 100 .
若存在一个使所给集合的元素个数小于 100 的集合 $S=\left\{a_1, a_2, \cdots, a_{20}\right\}$. 我们计算 $S$ 的 “好子集” $\{x, y, z, w\}$ 的个数, 这里 $x<y \leqslant z<w$, 且 $x+w=y+z$.
对 $S$ 中满足 $b>c$ 的数对 (b,c) (共 190对), 考虑它们的差 $b-c$, 由于至多有 99 个不同的差 (这里用到反证法假设), 故必有至少 91 个数对 $(b, c)$, 使得存在 $b^{\prime}, c^{\prime} \in S$, 满足 $b^{\prime}<b, c^{\prime}<c$, 且 $b-c=b^{\prime}-c^{\prime}$. 对这样的 91 个数对 $(b$, $c)$, 它与其相应的 $b^{\prime}, c^{\prime}$ 形成 $S$ 的一个 4 元集 $\left\{b, c, b^{\prime}, c^{\prime}\right\}$, 可得到 $S$ 的一个 “好子集” $\{x, y, z, w\}$, 且至多两个数对 $(b, c)$ 形成相同的子集 $\{x, y, z, w\}$ (只能是 $(b, c)=(w, z)$ 和 $(w, y)$ ). 故 $S$ 的“好子集”至少有 46 个.
另一方面, $S$ 的 “好子集” $\{x, y, z, w\}$ 的个数等于 $\sum \frac{1}{2} s_i\left(s_i-1\right)$, 这里 $s_i$ 为 $S$ 中满足 $b+c=i, b \leqslant c$ 的数对 $(b, c)$ 的个数, 其中 $i$ 为正整数.
注意到, 对每个 $i, S$ 中的每个元素 $s$ 至多出现在上面的一个数对 $(b, c)$ 中 (事实上, 当 $s \leqslant i-s$ 时, $s$ 出现在数对 $(s, i-s)$ 中, 其余情况出现在 $(i-s, s)$ 中), 于是 $s_i \leqslant 10$. 从而在 $s_i \neq 0$ 时, $1 \leqslant s_i \leqslant 10$, 故 $\frac{1}{2} s_i\left(s_i-1\right) \leqslant 5 s_i-5$. 由于集合 $\left\{a_i+a_j \mid 1 \leqslant i \leqslant j \leqslant 20\right\}$ 中有 201 个不同的元素,故使得 $s_i \geqslant 1$ 的正整数 $i$ 有 201 个, 设 $T$ 为这样的 $i$ 组成的集合.
易知 $S$ 中有 $\mathrm{C}_{20}^2$ 对 $(b, c)$ 满足 $b<c$, 有 20 对 $(b, c)$ 满足 $b=c$, 所以 $\sum_{i \in T} s_i=\mathrm{C}_{20}^2+20=210$. 于是,
$$
\sum_{i \in T} \frac{1}{2} s_i\left(s_i-1\right) \leqslant \sum_{i \in T}\left(5 s_i-5\right)=5 \times(210-201),
$$
这与 $S$ 的“好子集”至少有 46 个矛盾.
所以,所给集合中,至少有 100 个不同的元素.
|
[] |
|
./raw_volume-zh/volume1/chapter4.tex
|
proof
|
例2. 集合 $H=\{1,2, \cdots, 9\}$ 的分拆 $p$ 是将 $H$ 表示为两两不相交的子集的并.
对于 $n \in H$ 和分拆 $p$, 将包含 $n$ 的子集中元素的数目记为 $p(n)$. 例如, 若 $p:\{1,4,5\} \cup\{2\} \cup\{3,6,7,8,9\}$, 则 $p(6)=5$. 证明: 对 $H$ 的任意两个分拆 $p_1 、 p_2$, 存在 $H$ 的两个不同的元素 $m 、 n$, 使得
$$
p_1(m)=p_1(n), p_2(m)=p_2(n) .
$$
|
分析:因为 $H$ 只有 9 个元素, 对于一个确定的分拆 $p, p(i)(i=1$, $2, \cdots, 9)$ 只有三种不同的取值, 这是因为若有四种不同的取值, 则至少需要 $1+2+3+4=10$ 个元素.
这就给我们打开了一条通过对 $p(i)$ 可能的取值个数的研究解决问题的思路.
解用反证法.
假设可以找到 $H$ 的两个分拆 $p_1 、 p_2$, 使不存在 $H$ 的两个不同的元素 $m 、 n$, 满足
$$
p_1(m)=p_1(n), p_2(m)=p_2(n) .
$$
对于确定的 $p_1$, 若 $p_1(i)(i=1,2, \cdots, 9)$ 有四种不同的取值, 则至少需要 $1+2+3+4=10$ 个元素, 而 $|H|=9$, 矛盾.
所以, $p_1(i)$ 至多有三种不同的取值.
若同时有四个元素的 $p_1(i)$ 取值相等, 由于 $p_i(i)$ 至多有三个不同取值, 所以,必有四个中的两个元素 $m 、 n$,使得 $p_2(m)=p_2(n)$ ,与假设矛盾.
若 $p_1(i)$ 至多有两种不同的取值, 由抽屉原理知, 至少有 $H$ 的四个不同元素的 $p_1(i)$ 值相同.
这说明对于任意确定的 $p_1, p_1(i)$ 恰有三种不同取值, 且每种取值有三个元素取到.
也就是说对于分拆 $p_1, H$ 的每一个子集的元素个数不超过 3 . 不妨设
$$
\begin{aligned}
& p_1(1)=p_1(2)=p_1(3)=1, \\
& p_1(4)=p_1(5)=p_1(6)=2,
\end{aligned}
$$
$$
p_1(7)=p_1(8)=p_1(9)=3 .
$$
但 $p_1(4)=p_1(5)=p_1(6)=2$ 是不可能的.
这就否定了假设.
|
[] |
|
./raw_volume-zh/volume1/chapter4.tex
|
proof
|
例5. 证明: 可以把自然数集分划为 100 个非空子集,使得对任何 3 个满足关系式 $a+99 b=c$ 的自然数 $a 、 b 、 c$, 都可以从中找出两个数属于同一子集.
|
分析:当然, 只要能具体地构造一个满足条件的 100 -分划即可.
在构造之前, 有必要对关系式 $a+99 b=c$ 进行讨论.
有两点是显然的: $a(\bmod 99) \equiv c(\bmod 99) ; a 、 b 、 c$ 中偶数的个数为奇数.
我们的证明由此人手.
证明按如下法则构造自然数集的子集: 在第 $i$ 个子集 $(1 \leqslant i \leqslant 99)$ 中放人所有被 99 除余 $i-1$ 的偶数,而在第 100 个子集中放人所有的奇数.
显然, 这是一个自然数集的 $100-$ 分划.
在任何满足方程 $a+99 b=c$ 的自然数 $a 、 b 、 c$ 中, 偶数的个数为奇数, 且
$a(\bmod 99) \equiv c(\bmod 99)$.
如果 $a 、 b 、 c$ 中有两个为奇数,则此二奇数同属第 100 个子集; 否则, 它们全为偶数,且 $a$ 和 $c$ 被 99 除同余,故 $a$ 与 $c$ 仍属于同一子集.
|
[] |
|
./raw_volume-zh/volume1/chapter4.tex
|
proof
|
例6. 设集合 $A_1, A_2, \cdots, A_n$ 和 $B_1, B_2, \cdots, B_n$ 是集合 $M$ 的两个 $n$ 一分划, 已知对任意两个交集为空集的集合 $A_i, B_j(1 \leqslant i, j \leqslant n)$, 均有 $\left|A_i \cup B_j\right| \geqslant n$. 求证: $|M| \geqslant \frac{n^2}{2}$.
|
分析:由 $A_i 、 B_j$ 的交集为空集,有 $\left|A_i \cup B_j\right|=\left|A_i\right|+\left|B_j\right| \geqslant n$. 当每一个 $\left|A_i\right| \geqslant \frac{n}{2}$ 时, 结论显然成立.
当某个 $\left|A_i\right|$, 不妨设为 $\left|A_1\right|$ 小于 $\frac{n}{2}$ 时, 设 $\left|A_1\right|=k$, 这时与 $A_1$ 相交的 $B_j$ 至多有 $k$ 个; 而至少有 $n-k$ 个集合与 $A_1$ 不相交, 它们每一个的元素个数不小于 $n-k$. 假如 $k$ 是所有 $\left|A_i\right| 、\left|B_j\right|$ 中最小的, 则有 $|M| \geqslant k \cdot k+(n-k)(n-k) \geqslant \frac{n^2}{2}$.
证明设 $k=\min \left\{\left|A_i\right|,\left|B_j\right|, 1 \leqslant i, j \leqslant n\right\}$, 不妨设 $\left|A_1\right|=k$.
若 $k \geqslant \frac{n}{2}$, 则
$$
|M|=\sum_{i=1}^n\left|A_i\right| \geqslant n k \geqslant \frac{n^2}{2}
$$
若 $k<\frac{n}{2}$, 由于 $B_1, B_2, \cdots, B_n$ 两两不相交, 故 $B_1, B_2, \cdots, B_n$ 中至多有 $k$ 个集合与 $A_1$ 的交集不空, 从而另外的 $n-k$ 个集合均与 $A_1$ 的交集为空集, 且这些集合的元素个数不小于 $n-k$. 由 $n>2 k$, 得 $n-k>k$. 于是我们有
$$
\begin{aligned}
|M| & =-\sum_{i=1}^n\left|B_i\right| \geqslant k \cdot k+(n-k) \cdot(n-k) \\
& =k^2+(n-k)^2 \\
& \geqslant \frac{1}{2}(k+(n-k))^2=\frac{n^2}{2} .
\end{aligned}
$$
综上所述,命题成立.
说明本例的解答应用了最小数原理.
关于最小数原理的应用, 我们将在后面作专门的介绍.
|
[] |
|
./raw_volume-zh/volume1/chapter4.tex
|
proof
|
例7. 设自然数集分划成 $r$ 个互不相交的子集: $\mathbf{N}=A_1 \cup A_2 \cup \cdots \cup A_r$. 求证其中必有某个子集 $A$, 它具有如下性质 $P$ : 存在 $m \in \mathbf{N}$, 使对任何正整数 $k$, 都能找到 $a_1, a_2, \cdots, a_k \in A$, 满足
$$
1 \leqslant a_{j+1}-a_j \leqslant m, j=1,2, \cdots, k-1 .
$$
|
分析:显然具有性质 $P$ 的子集 $A$, 不可能是 $\mathbf{N}$ 的 $r$-分划中的有限集.
不妨设 $\mathbf{N}$ 的 $r$-分划中的无限集为 $A_1, A_2, \cdots, A_{r^{\prime}}$, 令 $B=A_2 \cup A_3 \cup \cdots \cup A_{r^{\prime}}$. 设 $b$ 是集合 $A_{r^{\prime}+1} \cup \cdots \cup A_{r-1} \cup A_r$ 中的最大自然数, 则 $b$ 以后的自然数都在 $N^{\prime}= A_1 \cup B$ 中, 即 $N^{\prime}$ 中存在任意有限长度的相继自然数段.
只需证明: 若 $A_1$ 不具有性质 $P$, 则 $B$ 必具有性质 $P$.
证明先证下面的引理:
引理设 $\mathbf{N}=A_1 \cup A_2 \cup \cdots \cup A_r$, 且 $A_1, A_2, \cdots, A_r$ 两两不相交.
若 $A_i \cup A_{i+1} \cup \cdots \cup A_r$ 包含任意有限长度的相继自然数段.
而 $A_i$ 不具有性质 $P$, 则 $A_{i+1} \cup \cdots \cup A_r$ 中必定含有任意有限长度的相继自然数段.
引理的证明若 $A_i$ 不具有性质 $P$, 则对于任给的 $m \in \mathbf{N}$, 存在 $k(m) \in \mathbf{N}$, 使得对于 $A_i$ 的任何 $k(m)$ 个数 $a_1<a_2<\cdots<a_{k(m)}$, 都可找到下标 $j \in\{1,2, \cdots, k(m)-1\}$, 数 $a_j$ 与 $a_{j+1}$ 之间至少有 $m$ 个相继自然数都不属于 $A_i$.
在 $A_i \cup A_{i+1} \cup \cdots \cup A_r$ 中选取一个长度为 $L=k(m) m$ 的相继自然数段.
若该段数中有 $k(m)$ 个数属于 $A_i$, 则因 $A_i$ 不具有性质 $P$, 故在这 $k(m)$ 个数中, 存在两个数 $a_j$ 与 $a_{j+1}$, 它们之间有 $m$ 个相继自然数都不属于 $A_i$, 当然就都属于 $A_{i+1} \cup \cdots \cup A_r$. 若选出的长度为 $L$ 的相继自然数段中属于 $A_i$ 的数少于 $k(m)$ 个, 则当把这 $L$ 个相继自然数依次分成 $k(m)$ 段, 每段恰有 $m$ 个数时, 由抽庶原理知其中必有一段 $m$ 个数中不含 $A_i$ 中的数, 当然都属于 $A_{i+1} \cup \cdots \cup A_r$. 由 $m \in \mathbf{N}$ 的任意性知引理成立.
回到原题的证明.
若 $A_1$ 具有性质 $P$, 则结论成立; 若 $A_1$ 不具有性质 $P$, 则由引理知 $A_2 \cup A_3 \cup \cdots \cup A_r$ 满足引理的条件.
若 $A_2$ 具有性质 $P$, 则结论成立; 若 $A_2$ 不具有性质 $P$, 则 $A_3 \cup \cdots \cup A_r$ 又满足引理的条件.
这样继续下去, 或者在某一步得出 $A_{i_0}$ 具有性质 $P$, 或者进行到最后, 得到 $A_r$ 含有任意有限长度的自然数段, 当然具有性质 $P$.
说明由上面的证明可以看出, 本例可作如下的加强:设 $M \subset \mathbf{N}, M$ 中存在任意有限长度的相继自然数段, 作 $M$ 的 $r$-分划: $A_1, A_2, \cdots, A_r$, 则在这些子集中必存在某个子集 $A$ 具有性质 $P$. 可以对 $r$ 进行归纳证明.
|
[] |
|
./raw_volume-zh/volume1/chapter4.tex
|
proof
|
例8. 将正整数集拆分为两个不相交的子集 $A 、 B$, 满足条件:
(1) $1 \in A$;
(2) $A$ 中没有两个不同的元素, 使它们的和形如 $2^k+2(k=0,1,2, \cdots)$;
(3)B 中也没有两个不同的元素, 其和具有上述形式.
证明: 这种拆分可以以唯一的方式实现, 并确定 1987, 1988, 1989 所属的子集.
|
分析:对任意的自然数 $n$, 总存在非负整数 $h$, 使 $2^h \leqslant n<2^{h+1}$. 若 $m<n$, 则存在 $n+m=2^h+2$ 或 $n+m=2^{h+1}+2$ 两种可能, 只要将 $n$ 与 $m$ 置于不同的集合即可.
证明因为 $1+2=2^0+2$, 所以 $2 \in B$. 设对小于 $n$ 的数均有惟一的归属, 且满足条件 (1)、(2)、(3). 考虑 $n(n \geqslant 3)$, 总有自然数 $h$, 使
$$
2^h \leqslant n<2^{h+1} .
$$
若 $n=2^h, h>1$, 因 $2 \in B$, 故 $n \in A$. 这时, 对 $A$ 中任一元素 $m<n$,有
$$
2^h<n+m<2^{h+1} .
$$
而 $m \neq 2$, 所以 $n+m$ 不能写成 $2^h+2$ 的形式.
条件(1)、(2)、(3)成立.
若 $n=2^h+1$, 则 $1+n=2^h+2$, 故 $n \in B$. 这时, 对 $B$ 中任一元素 $m<n$,
$$
2^h+2<n+m<2^{h+1}+2 .
$$
条件(1)、(2)、(3)成立.
若 $n>2^h+1$, 则 $2^{h+1}+2-n<n$, 所以必须令 $n$ 与 $2^{n+1}+2-n$ 在不同集合中.
这时, 设 $m<n$, 且与 $n$ 在同一集合中, 则
$$
2^h+2<n+m<2^{h+2}+2,
$$
而 $n+m \neq 2^{h+1}+2$. 所以, 条件 (1)、(2)、(3)仍然成立.
这说明所说的拆分可以惟一地实现.
由于 $1987=2^{11}+2-63$, 而 $63=2^6+2-3,3=2^1+2-1 \in B$, 所以 $1987 \in B$.
同理可知 $1988 \in A, 1989 \in B$.
|
[] |
|
./raw_volume-zh/volume1/chapter4.tex
|
proof
|
例9. 平面上横纵坐标都为有理数的点称为有理点.
求证: 平面上的全体有理点可分成 3 个两两不相交的集合,满足条件:
(i)在以每个有理点为圆心的任一圆内一定包含 3 个点分属这 3 个集合;
(ii)在任何一条直线上都不可能有 3 个点分别属于这 3 个集合.
|
分析:由有理数的稠密性知, 以坐标平面上任何点 $D$ 为圆心, 任何正数 $r$ 为半径的圆内都有无数多个有理点.
关键是怎样使这些点分属三个不同的集合, 这似乎比较容易办到.
如果直线 $a x+b y+c=0$ 上有 1 个以上的有理点, 则直线方程化简后的系数必皆为有理数, 这时直线上有无数多个有理点, 如果 3 -分划能使同一直线上的有理点至多属于分划的两个子集, 则问题获解.
证明显然, 任一有理点均可惟一地写成 $\left(\frac{u}{w}, \frac{v}{w}\right)$ 的形式, 其中 $u 、 v 、 w$ 都是整数, $w>0$ 且 $(u, v, w)=1$.
令
$$
\begin{aligned}
& A=\left\{\left(\frac{u}{w}, \frac{v}{w}\right) \mid 2 \nmid u\right\}, \\
& B=\left\{\left(\frac{u}{w}, \frac{v}{w}\right)|2| u, 2 \nmid v\right\}, \\
& C=\left\{\left(\frac{u}{w}, \frac{v}{w}\right)|2| u, 2 \mid v\right\} .
\end{aligned}
$$
让我们来验证这 3 个集合满足条件(i)和(ii).
设平面上的直线方程为
$$
a x+b y+c=0 .
$$
如果其上有两个不同的有理点 $\left(x_1, y_1\right)$ 和 $\left(x_2, y_2\right)$, 则有
$$
\left\{\begin{array}{l}
a x_1+b y_1+c=0, \\
a x_2+b y_2+c=0 .
\end{array}\right.
$$
如果 $c=0$, 则可取 $a 、 b$ 为有理数.
如果 $c \neq 0$, 不妨设 $c=1$, 于是, 从上面的联立方程中可解得 $a$ 和 $b$ 的值, 当然都是有理数.
再通分即知, 可以使 $a 、 b 、 c$ 都是整数且满足 $(a, b, c)=1$.
设有理点 $\left(\frac{u}{w}, \frac{v}{w}\right)$ 在直线 $a x+b y+c=0$ 上, 于是, 有
$$
L: a u+b v+c w=0 .
$$
(1) 先证集合 $A 、 B 、 C$ 满足条件(ii). 分三种情形.
(a) $2 \nmid c$. 若 $2|u, 2| v$, 则由 (1) 知 $2 \mid c w$, 从而 $2 \mid w$, 此与 $(u, v, w)=$ 1 矛盾.
所以,集合 $C$ 中的点都不能在直线 $L$ 上.
(b) $2 \mid c, 2 \nmid b$. 若 $2 \nmid v$, 则 $2 \nmid a u$, 从而 $2 \nmid u$. 因此, 集合 $B$ 中的点都不能在直线 $L$ 上.
(c) $2|c, 2| b$. 由(1)得 $2 \mid a u$. 又因 $(a, b, c)=1$, 故 $2 \nmid a$. 所以 $2 \mid u$. 这表明集合 $A$ 中的点都不在直线 $L$ 上.
综上可知, $A 、 B 、 C$ 这 3 个集合满足条件(ii).
(2) 再证满足条件 (i).
设 $D$ 是以有理点 $\left(\frac{u_0}{w_0}, \frac{w_0}{w_0}\right)$ 为圆心, 以 $r$ 为半径的圆.
取正整数 $k$, 使得
$$
2^k>\max \left\{w_0, \frac{1}{r}\left(\left|u_0\right|+\left|v_0\right|+1\right)\right\} .
$$
于是易验证, 下列 3 个有理点
$$
\begin{gathered}
\left(\frac{u_0 2^k+1}{w_0 2^k}, \frac{v_0 2^k}{w_0 2^k}\right) \in A,\left(\frac{u_0 2^k}{w_0 2^k}, \frac{v_0 2^k+1}{w_0 2^k}\right) \in B, \\
\left(\frac{u_0 2^k}{w_0\left(2^k+1\right)}, \frac{v_0 2^k}{w_0\left(2^k+1\right)}\right) \in C
\end{gathered}
$$
都在 $\odot D$ 的内部.
注意, 在上述 3 点中, $u_0 、 v_0 、 w_0$ 不一定互质.
但由于 $2^k> w_0$, 故约分之后不改变分子的奇偶性.
这表明条件(i)成立.
最后,我们来看一个非常特殊的集合分划的例子.
|
[] |
|
./raw_volume-zh/volume1/chapter5.tex
|
proof
|
例1. 试证: 任一有限集的全部子集可以排定次序, 使得任何相邻的两个子集都相差一个元素.
|
分析:不妨设有限集 $A=\{1,2,3, \cdots, n\}$. 先来看一些简单情形:
当 $n=1$ 时,显然可以排成: $\varnothing,\{1\}$;
当 $n=2$ 时,共有 $2^2=4$ 个子集,可排成: $\varnothing,\{1\},\{1,2\},\{2\}$;
当 $n=3$ 时,共有 $2^3=8$ 个子集,可排成: $\varnothing,\{1\},\{1,2\},\{2\},\{2,3\},\{1,2,3\},\{1,3\},\{3\}$.
显然符合条件的排序方式不是惟一的.
请注意 $n=3$ 时的上述排法: 所有子集可分为两组, 前 4 个子集都不含元素 3 ; 后 4 个均含元素 3, 且去掉 3 后恰是前 4 个子集排列的逆序.
事实上, $n=2$ 时也如此.
这说明我们可以考虑用数学归纳法来证明.
证明设有限集为 $M_n=\left\{w_1, w_2, \cdots, w_n\right\}$, 我们对 $n$ 进行归纳.
当 $n=1$ 时, $M_1=\left\{w_1\right\}$,将它的两个子集排列成 $\varnothing,\left\{w_1\right\}$ 即可.
假设当 $n=k$ 时,命题成立.
当 $n=k+1$ 时,
$$
M_{k+1}=\left\{w_1, w_2, \cdots, w_k, w_{k+1}\right\},
$$
它是由集合 $M_k=\left\{w_1, w_2, \cdots, w_k\right\}$ 添加元素 $w_{k+1}$ 而形成的.
$M_k$ 的子集个数为 $2^k$. 由归纳假设知, 可将 $M_k$ 的全体子集排成满足题设要求的一列, 不妨设
$$
A_1, A_2, A_3, \cdots, A_{2^k}\left(A_i \subseteq M_k, i=1,2,3, \cdots, 2^k\right)
$$
就是这样的一个排列.
我们来看排列
$$
A_1, A_2, A_3, \cdots, A_{2^k}, A_{2^k} \bigcup\left\{w_{k+1}\right\}, A_{2^k-1} \bigcup\left\{w_{k+1}\right\}, \cdots, A_1 \bigcup\left\{w_{k+1}\right\},
$$
它恰好由 $M_{k+1}$ 的 $2^{k+1}$ 个不同子集排成, 且任意两个相邻集合的元素都仅相差 1 个.
可见当 $n=k+1$ 时, 命题也成立.
所以,对任意的 $n \in \mathbf{N}^*$, 所述命题成立.
说明一个复杂的问题,也许一时找不到解题的突破口, 这时可考虑“以退求进”的策略.
先解决一些简单的或特殊的情形, 从中发现规律和方法, 从而找到解决一般问题的办法.
这也就是从特殊到一般的思维方法.
|
[] |
|
./raw_volume-zh/volume1/chapter5.tex
|
proof
|
例2. 在某次竞选中各政党作出 $n$ 种不同的诺言 $(n>0)$, 有些政党可以作某些相同的诺言.
现知其中每两个政党都至少作了一个相同的诺言, 但没有两个政党的诺言完全相同.
求证: 政党个数 $\leqslant 2^{n-1}$.
|
证明:设有 $m$ 个政党.
以 $A$ 记所有诺言的集合, $A_i$ 记第 $i$ 个政党的诺言的集合 $(i=1,2, \cdots, m)$. 由题设知
$$
|A|=n, A_i \cap A_j \neq \varnothing, A_i \neq A_j, 1 \leqslant i<j \leqslant m .
$$
因 $\left(\complement_A A_i\right) \cap A_i=\varnothing$, 故 ${ }_A A_i \neq A_j(i, j=1,2, \cdots, m)$, 即 $\complement_A A_i$ 不同于任何一个政党的诺言的集合.
所以
$$
A_1, A_2, \cdots, A_m, \complement_A A_1, \complement_A A_2, \cdots, \complement_A A_m
$$
各不相同, 而它们的个数不超过集合 $A$ 的所有子集的数目 $2^n$, 即 $2 m \leqslant 2^n$, 所以
$$
m \leqslant 2^{n-1} .
$$
说明上述解法的特点是将一个趣味问题转化为集合问题, 然后借助集合的知识和方法来解决.
|
[] |
|
./raw_volume-zh/volume1/chapter5.tex
|
proof
|
例3. 设正整数 $n \geqslant 5, n$ 个不同的正整数 $a_1, a_2, \cdots, a_n$ 有下列性质: 对集合 $S=\left\{a_1, a_2, \cdots, a_n\right\}$ 的任何两个不同的非空子集 $A$ 和 $B, A$ 中所有数的和与 $B$ 中所有数的和都不会相等.
在上述条件下,求
$$
\frac{1}{a_1}+\frac{1}{a_2}+\cdots+\frac{1}{a_n}
$$
的最大值.
|
分析:因为 $S$ 的任何两个不同的非空子集的各自元素之和不相等, 由集合元素的互异性及正整数二进制表示的惟一性的启示, 似乎集合 $S$ 中的数应是形如 $2^r(r \in \mathbf{N})$ 的数.
下面的工作就是由此展开的.
解不妨设 $a_1<a_2<\cdots<a_n$.
先证明对任意自然数 $k \leqslant n$, 都有
$$
\sum_{i=1}^k a_k \geqslant 2^k-1
$$
用反证法.
若 $\sum_{i=1}^k a_k<2^k-1$, 则 $\left\{a_1, a_2, \cdots, a_k\right\}$ 的每个非空子集的元素和不超过 $2^k-2$. 但 $\left\{a_1, a_2, \cdots, a_k\right\}$ 有 $2^k-1$ 个非空子集, 根据抽屉原则, 必有两个非空子集的元素和相等, 这与题设矛盾.
故所证结论(1)成立.
接着证明:
$$
\frac{1}{a_1}+\frac{1}{a_2}+\cdots+\frac{1}{a_n} \leqslant 1+\frac{1}{2}+\cdots+\frac{1}{2^{n-1}}=2-\frac{1}{2^{n-1}} .
$$
事实上,
$$
\begin{gathered}
1+\frac{1}{2}+\cdots+\frac{1}{2^{n-1}}-\left(\frac{1}{a_1}+\frac{1}{a_2}+\cdots+\frac{1}{a_n}\right) \\
=\frac{a_1-1}{a_1}+\frac{a_2-2}{2 a_2}+\cdots+\frac{a_n-2^{n-1}}{2^{n-1} a_n} . \\
\text { 令 } C_i=\frac{1}{2^{i-1} a_i}, d_i=a_i-2^{i-1}, D_k=\sum_{i=1}^k d_i \text { 显然 } C_1>C_2>\cdots>C_n, \text { 且 } \\
D_k=\sum_{i=1}^k a_i-\left(1+2+\cdots+2^{k-1}\right)=\sum_{i=1}^k a_i-\left(2^k-1\right) \geqslant 0 .
\end{gathered}
$$
于是我们有
$$
\begin{aligned}
& 1+\frac{1}{2}+\cdots+\frac{1}{2^{n-1}}-\left(\frac{1}{a_1}+\frac{1}{a_2}+\cdots+\frac{1}{a_n}\right) \\
= & \sum_{i=1}^n C_i d_1 \\
= & C_1 D_1+C_2\left(D_2-D_1\right)+\cdots+C_n\left(D_n-D_{n-1}\right) \\
= & \left(C_1-C_2\right) D_1+\left(C_2-C_3\right) D_2+\cdots+\left(C_{n-1}-C_n\right) D_{n-1}+C_n D_n \\
\geqslant & 0,
\end{aligned}
$$
故(2)式得证.
注意到, 当 $S=\left\{1,2,2^2, \cdots, 2^{n-1}\right\}$ 时, 题设条件成立.
此时有
$$
\frac{1}{a_1}+\frac{1}{a_2}+\cdots+\frac{1}{a_n}=1+\frac{1}{2}+\cdots+\frac{1}{2^{n-1}}=2-\frac{1}{2^{n-1}} \text {. }
$$
因此, 所求的最大值是 $2-\frac{1}{2^{n-1}}$.
|
[] |
|
./raw_volume-zh/volume1/chapter5.tex
|
proof
|
例5. 对于整数 $n(n \geqslant 2)$, 如果存在集合 $\{1,2, \cdots, n\}$ 的子集族 $A_1$, $A_2, \cdots, A_n$ 满足;
(a) $i \notin A_i, i=1,2, \cdots, n$;
(b) 若 $i \neq j, i, j \in\{1,2, \cdots, n\}$, 则 $i \in A_j$, 当且仅当 $j \notin A_i$;
(c) 任意 $i, j \in\{1,2, \cdots, n\}, A_i \cap A_j \neq \varnothing$.
则称 $n$ 是 “好数”.
证明: (1) 7 是好数;
(2)当且仅当 $n \geqslant 7$ 时, $n$ 是好数.
|
分析:对于 $n=7$, 可以作出满足条件的子集族来验证; 当 $n \geqslant 7$ 时, 可考虑用数学归纳法证明.
证明 (1) 当 $n=7$ 时, 取
$$
\begin{aligned}
& A_1=\{2,3,4\}, A_2=\{3,5,6\}, A_3=\{4,5,7\}, \\
& A_4=\{2,6,7\}, A_5=\{1,4,6\}, A_6=\{1,3,7\}, \\
& A_7=\{1,2,5\}
\end{aligned}
$$
即可.
(2) 先证当 $n \geqslant 7$ 时, $n$ 是好数.
对 $n$ 进行归纳.
由 (1) 知, 当 $n=7$ 时, 结论成立.
假设 $n(n \geqslant 7)$ 是好数, 则存在子集族 $A_1, A_2, \cdots, A_n$ 满足条件.
对于 $n+$ 1 , 取子集族 $B_1=A_1, B_2=A_2, \cdots, B_n=A_n, B_{n+1}=\{1,2, \cdots, n\}$. 由归纳假设易知, 它们也是满足条件的.
下面证明每一个好数 $n$ 都至少为 7 .
如果 $A_1, A_2, \cdots, A_n$ 是一个 $n$ 为好数的集合的子集族,那么, 每一个 $A_i$ 至少有三个元素.
事实上,若 $A_i \subset\{j, k\}$, 则
$$
A_i \cap A_j=\{k\}, A_i \cap A_k=\{j\} .
$$
所以, $k \in A_j, j \in A_k$. 矛盾.
考虑一个由元素 $0 、 1$ 构成的 $n \times n$ 阶正方形表格,当且仅当 $j \in A_i$ 其第 $i$ 行第 $j$ 列的元素为 1 . 表中对角线上的元素为 0 , 对于余下的元素,因为 $i \neq j$, 当且仅当 $a_{j i}=1$ 时 $a_{i j}=0$, 所以 0 的个数等于 1 的个数.
因此, 表中元素的和为 $\frac{n^2-n}{2}$. 又每行元素的和大于等于 3 , 所以 $n^2-n \geqslant 6 n$, 故 $n \geqslant 7$.
|
[] |
|
./raw_volume-zh/volume1/chapter5.tex
|
proof
|
例6. 集合 $X=\{1,2, \cdots, 6 k\}, k \in \mathbf{N}^*$. 试作出 $X$ 的三元子集族 $\mathscr{A}$, 满足:
(1) $X$ 的任一二元子集至少被族 $\mathscr{A}$ 中的一个三元子集包含;
(2) $|\mathscr{A}|=6 k^2$.
|
解:先证明下面的引理:
引理对 $n \in \mathbf{N}^*$, 集合 $X_1=\{1,2, \cdots, 2 n\}$ 的全部二元子集可分成 $2 n-1$ 组, 且每组是 $X_1$ 的一个分划.
引理的证明: 如图(<FilePath:./images/volume1/figures/fig-c5e6.png>),将 $1,2, \cdots, 2 n-1$ 这 $2 n-1$ 个数按顺时针方向放到一个正 $2 n-1$ 边形的顶点上,数 $2 n$ 放在外接圆圆心.
连结 $2 n$ 与 1 , 作 $n-1$ 条以 $2 n-1$ 边形顶点为端点且垂直于 1 与 $2 n$ 连线的线段,便得到 $X_1$ 的 $n$ 个二元子集构成 $X_1$ 的一个分划.
将 $2 n$ 与 1 的连线依次顺时针旋转 $\frac{2 \pi}{2 n-1}, \frac{4 \pi}{2 n-1}, \cdots, \frac{(4 n-4) \pi}{2 n-1}$, 作出相应的图及
$X_1$ 的 $n$ 个二元子集.
这样, $X_1$ 的全部 $\mathrm{C}_{2 n}^2=n(2 n-1)$ 个二元子集被分成 $2 n-1$ 组, 且每组 $n$ 个集合构成 $X_1$ 的一个分划.
下面来作满足题设的子集族:
$$
\text { 令 } A=\{1,2, \cdots, 2 k\}, B=\{2 k+1,2 k+2, \cdots, 4 k\}, C=\{4 k+1 ,4 k+2, \cdots, 6 k\}
$$.
由引理, $A$ 的全部二元子集可分成 $2 k-1$ 组, 每组是 $A$ 的一个分划.
将其中一组重复一次, 得到 $A$ 的 $2 k$ 个分划, 让其中每个分划与 $B$ 的一个元素搭配作出 $k$ 个 $X$ 的三元子集.
类似地,作出 $B$ 的 $2 k$ 个二元子集构成的分划, 包含 $B$ 的全部二元子集, 让其中每个分划与 $C$ 的一个元素搭配作出 $k$ 个 $X$ 的三元子集; 作出 $C$ 的 $2 k$ 个二元子集构成的分划, 包含 $C$ 的全部二元子集, 让其中每个分划与 $A$ 的一个元素搭配作出 $k$ 个 $X$ 的三元子集.
上面得到的 $k \times 2 k \times 3=6 k^2$ 个 $X$ 的三元子集组成的族 $\mathscr{A}$ 满足题设要求.
说明 $X$ 的二元子集有 $\mathrm{C}_{6 k}^2=3 k(6 k-1)=18 k^2-3 k$ 个.
而所作的三元子集族中的每个集合 (子集族的元素) 都包含 3 个二元子集, 子集族共可生成二元子集 $3 \times 6 k^2=18 k^2$ 个.
这说明有 $3 k$ 个(次)二元子集在子集族中被重复生成.
那么, 满足条件 (1) 的 $|\mathcal{A}|$ 的最小值是 $6 k^2$ 吗?
三、有关子集族的最值问题有关集合子集族的最值主要有三类:(1)求子集族阶的最值; (2) 求子集族中的集合阶的最值; (3) 求符合特定条件的集合元素的最值.
|
[
"./images/volume1/figures/fig-c5e6.png"
] |
|
./raw_volume-zh/volume1/chapter5.tex
|
proof
|
例7. 集合 $A=\{0,1,2, \cdots, 9\},\left\{B_1, B_2, \cdots, B_k\right\}$ 是 $A$ 的一族非空子集, 当 $i \neq j$ 时, $B_i \cap B_j$ 至多有两个元素.
求 $k$ 的最大值.
|
分析:集合 $A$ 的一元、二元、三元子集显然符合要求.
而 $A$ 的任一多于三元的子集 $B^{\prime}$ 必包含了.
$A$ 的三元子集, 故 $B^{\prime}$ 与其包含的三元子集不能同在题中的子集族内.
解首先至多含 3 个元素的 $A$ 的非空子集有
$$
\mathrm{C}_{10}^1+\mathrm{C}_{10}^2+\mathrm{C}_{10}^3=10+\frac{10 \times 9}{2}+\frac{10 \times 9 \times 8}{6}=175 \text { (个). }
$$
这些集合的交集至多有两个元素, 否则两集合相等, 矛盾.
因此 $k_{\max } \geqslant 175$.
下面证明 $k_{\max } \leqslant 175$.
设 $\mathscr{b}$ 为满足题设的子集族.
若 $B \in \mathscr{C}$, 且 $|B| \geqslant 4$, 设 $b \in B$, 则 $B$ 与 $B- \{b\}$ 不能同时含于 $\mathscr{C}$, 以 $B-\{b\}$ 代 $B$, 则 $\mathscr{C}$ 中元素数目不变.
仿此对 $\mathscr{C}$ 中所有元素数目多于 4 的集合 $B$ 作相应替代, 替代后子集族 $\mathscr{C}$ 中的每个集合都是元素数目不多于 3 的非空集合.
故 $k_{\max } \leqslant 175$.
所以, $k$ 的最大值为 175 .
说明上述解答采用了“两边夹”的策略: 先得出 $k$ 的最大值不小于 175 , 然后指出 $k$ 不大于 175 , 从而得出 $k_{\max }=175$.
|
[] |
|
./raw_volume-zh/volume1/chapter5.tex
|
proof
|
例9. 设 $n$ 为正整数, 在数集
$$
\{-n,-n+1,-n+2, \cdots,-1,0,1, \cdots, n-1, n\}
$$
中最多选取多少个数, 可使任意三个数的和均不为 0 (三个数可以相同)?
|
分析:显然, 当选取的数的绝对值充分大时, 可使任意三个数的和均不为 0 .
解设从题中数集中最多选取 $k$ 个数, 可使任意三个数的和均不为 0 . 考察子集
$$
\left\{-n, \cdots,-\left[\frac{n}{2}\right]-1,\left[\frac{n}{2}\right]+1, \cdots, n\right\},
$$
其中 $[x]$ 表示不超 $x$ 的最大整数.
知当 $n$ 为偶数时, $k \geqslant n$; 当 $n$ 为奇数时, $k \geqslant n+1$.
设 $A=\left\{a_1, a_2, \cdots, a_m\right\}, B=\left\{b_1, b_2, \cdots, b_l\right\}$ 都是元素为整数的非空集合.
定义集合
$$
A+B=\{a+b \mid a \in A, b \in B\},
$$
可以证明 $A+B$ 至少有 $m+l-1$ 个元素.
事实上, 不妨设 $a_1<a_2<\cdots<a_n, b_1<b_2<\cdots<b_l$, 则
$$
a_1+b_1, a_1+b_2, \cdots, a_1+b_l, a_2+b_l, \cdots, a_m+b_l
$$
是一个有 $m+l-1$ 项的严格递增的数列, 其中每一个数都是集合 $A+B$ 的元素.
假设 $S$ 是一个满足题设的子集.
显然 $0 \notin S$. 取
$$
\begin{aligned}
& A=S \cap\{-n,-n+1, \cdots,-1\}, \\
& B=S \cap\{1,2, \cdots, n\} .
\end{aligned}
$$
于是, $A+B$ 和 $-S=\{-s \mid s \in S\}$ 是集合 $\{-n,-n+1, \cdots, n\}$ 的两个不相交的子集.
由前证知
$$
\begin{aligned}
2 n+1 & \geqslant|A+B|+|-S| \\
& \geqslant|A|+|B|-1+|S| \\
& =2|S|-1,
\end{aligned}
$$
即 $|S| \leqslant n+1$.
当 $n$ 为奇数时, 就证明了 $k=n+1$.
当 $n$ 为偶数时, 还需要证明 $|S|=n+1$ 是不可能的.
由于 $A+B \subseteq\{-n+1,-n+2, \cdots, n-1\}$, 若有
$$
|A+B|+|-S|=2 n+1 \text {, }
$$
则必有 $-n, n \in-S$, 即 $-n, n \in S$. 于是
$$
\{1, n-1\}, \cdots,\left\{\frac{n}{2}-1, \frac{n}{2}+1\right\},\left\{\frac{n}{2}\right\}
$$
每个集合中至多有一个元素在 $B$ 中.
因此,
$$
|B| \leqslant \frac{n}{2} .
$$
同理,
$$
|A| \leqslant \frac{n}{2} .
$$
由 $A 、 B$ 的定义, 知
$$
|S|=|A|+|B| \leqslant n .
$$
与 $|S|=n+1$ 矛盾.
因此,当 $n$ 为偶数时, $k=n$.
|
[] |
|
./raw_volume-zh/volume1/chapter5.tex
|
proof
|
例10. 集合 $A=\{1,2, \cdots, 1997\}$, 对 $A$ 的任意一个 999 元子集 $X$, 若存在 $x, y \in X$, 使得 $x<y$ 且 $x \mid y$, 则称 $X$ 集为好集.
求最大自然数 $a(a \in A)$, 使任一含有 $a$ 的 999 元子集都为好集.
|
分析:抓住 $A$ 的 999 元子集 $X_0=\{999,1000, \cdots, 1997\}$ 是关键.
因为 $999 \times 2=1998>1997$, 所以 $a<999$. 考虑集合 $A$ 的这样的元素 $b: 2 b \in X_0$, $3 b \notin X_0$. 易知 $b=666+i, i=0,1, \cdots, 332$. 由 $B_i=\{666+i\} \cup X_0 \backslash \{2(666+i)\}, i=0,1, \cdots, 332,\left|B_i\right|=999$, 知 $a \leqslant 665$.
解我们证明 $\max a=665$.
先证 $a \leqslant 665$. 显然 $A$ 的 999 元子集 $X_0=\{999,1000,1001, \cdots, 1997\}$ 中不存在 $x, y \in X_0$, 使得 $x<y$ 且 $x \mid y$. 事实上, $X_0$ 的最小元素为 999 , 它的最小倍数除本身外为 $2 \times 999=1998>1997$, 即比 $X_0$ 的最大元素还大.
这样, $a$ 就不能为 $999,1000,1001, \cdots, 1997$ 中的任一个数.
构造集合
$$
B_i=\{666+i\} \bigcup X_0 \backslash\{2(666+i)\}, i=0,1, \cdots, 332 .
$$
对 $B_i$ 来说, $(666+i) \times 3 \geqslant 1998$, 而 $(666+i) \times 2 \notin B_i$, 故 $666+i$ 除本身外其他倍数都不在 $B_i$ 中.
上面已证 $X_0$ 的任一非本身的倍数都不在 $X_0$ 中; 而 $666+i<999(i=0,1,2, \cdots, 332)$, 故 $X_0$ 中任 $\cdots$ 元素的倍数不可能为 $666+i(i=0,1, \cdots, 332)$. 这样 $B_i$ 中仍不存在两元素满足 $x<y$ 且 $x \mid y$. 而 $B_i$ 中 $(i=0,1, \cdots, 332)$ 包含了 $666,667, \cdots, 998$, 故 $a \neq 666,667, \cdots, 998$. 所以 $a \leqslant 665$.
下证 665 是可取的.
反设存在一个含 665 的 999 元子集 $X$,不存在这样的 $x, y \in X, x<y$ 使 $x \mid y$, 则 $665 \times 2 、 665 \times 3 \notin X$.
构造如下 997 个抽庶, 它包含了 $A$ 中除 $665 、 665 \times 2 、 665 \times 3$ 外的所有元素,且每个元素只出现一次
$$
\begin{aligned}
& \left\{1,1 \times 2,1 \times 2^2, \cdots, 1 \times 2^{10}\right\}, \\
& \left\{3,3 \times 2,3 \times 2^2, \cdots, 3 \times 2^9\right\}, \\
& \left\{5,5 \times 2,5 \times 2^2, \cdots, 5 \times 2^8\right\}, \\
& \cdots \ldots \\
& \{663,663 \times 2,663 \times 3\}, \\
& \{667,667 \times 2\}, \\
& \{669,669 \times 2\}, \\
& \cdots \ldots . \\
& \{1991\},\{1993\},\{1997\} .
\end{aligned}
$$
$X$ 中除 665 外的其他 998 个元素归人这 997 个抽屉里, 定有两个在同一抽屉, 而同一抽屉里的数互为倍数关系, 矛盾.
证毕.
|
[] |
|
./raw_volume-zh/volume1/chapter6.tex
|
proof
|
例1. 已知数集 $M$ 至少有 3 个元素, 且对 $M$ 中任何两个不同的元素 $a 、 b$, 数 $a^2+b \sqrt{2}$ 都是有理数,证明: 对于 $M$ 中任何数 $a$,数 $a \sqrt{2}$ 都是有理数.
|
分析:设 $a, b \in M$ 且 $a \neq b$, 则 $a^2+b \sqrt{2} \in \mathbf{Q}, b^2+a \sqrt{2} \in \mathbf{Q}$. 于是有 $a^2+ b \sqrt{2}-\left(b^2+a \sqrt{2}\right)=\frac{1}{2}(a \sqrt{2}-b \sqrt{2})(a \sqrt{2}+b \sqrt{2}-2) \in \mathbf{Q}$. 若能证明 $a \sqrt{2}- b \sqrt{2} \in \mathbf{Q}$ 或 $a \sqrt{2}+b \sqrt{2} \in \mathbf{Q}$, 则问题迎刃而解.
但已给条件似乎不够用! 不过另设 $c \in M, c \neq a, c \neq b$, 则 $c^2+a \sqrt{2} \in \mathbf{Q}, c^2+b \sqrt{2} \in \mathbf{Q}$, 便得到
$$
c^2+a \sqrt{2}-\left(c^2+b \sqrt{2}\right)=a \sqrt{2}-b \sqrt{2} \in \mathbf{Q} .
$$
证明任取 $a, b, c \in M$, 且 $a 、 b 、 c$ 互不相等, 则 $a^2+b \sqrt{2}, b^2+a \sqrt{2}, c^2+ a \sqrt{2}, c^2+b \sqrt{2} \in \mathbf{Q}$. 因此
$$
\begin{aligned}
& a^2+b \sqrt{2}-\left(b^2+a \sqrt{2}\right)=(a-b)(a+b-\sqrt{2}) \\
= & \frac{1}{2}(a \sqrt{2}-b \sqrt{2})(a \sqrt{2}+b \sqrt{2}-2) \in \mathbf{Q}, \\
& c^2+a \sqrt{2}-\left(c^2+b \sqrt{2}\right)=(a \sqrt{2}-b \sqrt{2}) \in \mathbf{Q} .
\end{aligned}
$$
从而
$$
a \sqrt{2}+b \sqrt{2}-2 \in \mathbf{Q}
$$
所以 $a \sqrt{2}+b \sqrt{2} \in \mathbf{Q}$.
所以
$$
a \sqrt{2}=\frac{1}{2}(a \sqrt{2}+b \sqrt{2}+a \sqrt{2}-b \sqrt{2}) \in \mathbf{Q} .
$$
|
[] |
|
./raw_volume-zh/volume1/chapter6.tex
|
proof
|
例2. 设 $\alpha=\frac{r}{s}$, 这里 $r 、 s$ 是正整数,且 $r>s,(r, s)=1$. 令集合
$$
N_\alpha=\{[n \alpha] \mid n=1,2, \cdots\} .
$$
求证:对任何 $m \in N_\alpha, r \nmid m+1$.
|
分析:$n \alpha=n \cdot \frac{r}{s}$. 当 $s=1$ 时, 结论显然成立.
当 $s>1$ 时, 若 $1 \leqslant n \leqslant s-1$, 由 $\frac{r}{s}>1$ 知, $1 \leqslant n \alpha \leqslant r-\frac{r}{s}<r-1$, 即 $1 \leqslant[n \alpha]<r-1$, 结论成立; 若 $n \geqslant s$, 令 $n=q s+k\left(0 \leqslant k \leqslant s-1, q \in \mathbf{N}^*\right)$, 则 $n \alpha=q r+k \alpha,[n \alpha]= q r+\left[k_\alpha\right]$, 又转化为前面情形的讨论.
证明分两种情形讨论.
(1) 若 $s=1$, 则 $N_\alpha=\{r n \mid n=1,2, \cdots\}$. 因 $r>1$, 结论显然成立.
(2) 若 $s>1$, 因 $\frac{r}{s}>1$, 故
$$
1 \leqslant\left[\frac{r}{s}\right]<\left[\frac{2 r}{s}\right]<\cdots<\left[\frac{(s-1) r}{s}\right]=r+\left[-\frac{r}{s}\right]<r-1 .
$$
任取 $m=\left[n_0 \alpha\right] \in N_\alpha$, 令 $n_0=q s+k(0 \leqslant k \leqslant s-1)$, 则
$$
\begin{aligned}
& {\left[n_0 \alpha\right]=[q r+k \alpha]=q r+[k \alpha],} \\
& m+1=\left[n_0 \alpha\right]+1=q r+[k \alpha]+1 .
\end{aligned}
$$
但由不等式<1>, 有 $0 \leqslant[k \alpha]<r-1$, 即
$$
1 \leqslant[k \alpha]+1<r \text {. }
$$
于是, 由(2)式可知 $r \nmid m+1$.
综上可知, 命题成立.
|
[] |
|
./raw_volume-zh/volume1/chapter6.tex
|
proof
|
例3. 在平面上给定无穷多个点, 已知它们之间的距离都是整数, 求证这些点都在一条直线上.
|
分析:“无穷” 和“整数” 是两个关键词, 去其一, 则结论不成立.
下面我们就是利用这两点“制造”矛盾来反证结论成立.
证明若不然, 则存在三点 $A 、 B 、 C$, 使三点不共线且 $A B=r$ 和 $A C=s$ 都是整数.
设点 $P$ 是任一给定点, 则由三角不等式有
$$
|P A-P B| \leqslant A B=r,
$$
即 $|P A-P B|$ 是整数 $0,1,2, \cdots, r$ 中之一.
因此, 点 $P$ 或位于直线
$H_0=$ 直线 $A B$ 的垂直平分线,
$H_r=$ 直线 $A B$
之一上,或落在双曲线
$$
H_i=\{X|| X A-X B \mid=i\}, i=1,2, \cdots, r-1
$$
之一上.
同理, 点 $P$ 又或者位于直线
$$
\begin{aligned}
& K_0=\text { 线段 } A C \text { 的垂直平分线, } \\
& K_s=\text { 直线 } A C
\end{aligned}
$$
之一上, 或者落在双曲线
$$
K_j=\{X|| X A-X C \mid=j\}, j==1,2, \cdots, s-1
$$
之一上.
由此可知,任一给定点必落在集合
$$
H_i \cap K_j, i=0,1, \cdots, r, j=0,1, \cdots, s
$$
之一上.
由于直线 $A B$ 与 $A C$ 不重合, 所以任一 $H_i$ 与任一 $K_j$ 都不相同.
从而知(1)中每个集合都不多于 4 点,故知集合
$$
M=\bigcup_{i, j}\left(H_i \cap K_j\right)
$$
的点数不多于 $4(r+1)(s+1)$, 此与给定点有无穷多个矛盾.
|
[] |
|
./raw_volume-zh/volume1/chapter6.tex
|
proof
|
例4. 设 $M$ 为一个无限的有理数集, 满足: $M$ 的任意一个 2009 元子集的元素之积为一个整数,且这个整数不能被任何质数的 2009 次幕整除.
证明: $M$ 的元素均为整数.
|
分析:这里的“2009”并不是一个关键的数字, 与上例一样, 我们还是得围绕“无限”做文章.
证明设 $a_1, a_2, \cdots, a_{2008} \in M$. 记
$$
A=a_1 a_2 \cdots a_{2008}=\frac{p}{q},(p, q)=1 .
$$
假设 $M$ 中包含了无数多个形如
$$
\alpha_i=\frac{p_i}{q_i},\left(p_i, q_i\right)=1, q_i>1
$$
的数, 且 $\alpha_i \neq a_1, a_2, \cdots, a_{2008}$. 由于
$$
\alpha_i \cdot A=\alpha_i a_1 a_2 \cdots a_{2008}=\frac{p_i}{q_i} \cdot \frac{p}{q}
$$
为整数, 所以
$$
q_i \mid p .
$$
由于 $p$ 只有有限个因子, 故有无数个分母为 $q_i^{\prime}$ 的既约分数属于 $M$. 这些分数中的任意 2009 个的乘积都不是整数.
这与题设矛盾.
这说明 $M$ 中包含了无限多个整数, 记这些整数的集合为 $M^{\prime}$.
假设有 $\frac{a}{b} \in M,(a, b)=1, b>1$.
设 $p$ 为 $b$ 的一个质因子.
由于 $\frac{a}{b}$ 与 $M^{\prime}$ 中任意 2008 个整数的乘积为整数, 故 $p$ 为 $M^{\prime}$ 中无数多个整数的质因子.
而 $M^{\prime}$ 中任意 2009 个含有因数 $p$ 的数的乘积可被 $p^{2009}$ 整除.
这又与题设矛盾.
这就证明了 $M$ 的元素均为整数.
而这样的整数集是存在的, 如全部质数的集合.
|
[] |
|
./raw_volume-zh/volume1/chapter6.tex
|
proof
|
例5. 三维空间中所有整点 (3 个坐标都为整数的点) 的集合记为 $T$. 两个整点 $(x, y, z)$ 和 $(u, v, w)$ 当且仅当 $|x-u|+|y-v|+|z-w|=1$ 时称为相邻.
求证: 存在 $T$ 的一个子集 $S$, 使对每个 $P \in T$, 点 $P$ 以及 $P$ 的所有邻点中恰有一点属于 $S$.
|
分析:设 $(u, v, w) \in T$, 它的 6 个邻点分别为 $(u \pm 1, v, w),(u, v \pm 1$, $w),(u, v, w \pm 1)$. 若函数 $f(x, y, z)$ 在以上 7 点的函数值为整数, 且除以 7 的余数都不相同,则原题获证.
事实上, 取 $f=x+2 y+3 z$ 即可.
证明显然, 两个整点相邻, 当且仅当两点的各 3 个坐标中的两对分别相等,而第 3 个坐标相差 1 .
令 $\quad S=\{(x, y, z)\mid \, 7 | x+2 y+3 z\}$,
则 $S$ 满足题中要求.
事实上, 对于任何 $(u, v, w) \in T$, 它有 6 个邻点 $(u \pm 1, v, w),(u, v \pm 1, w),(u, v, w \pm 1)$. 这 7 点所对应的 7 个整数
$$
u+2 v+3 w+j, j=-3,-2,-1,0,1,2,3
$$
中, 恰有一个是 7 的倍数, 从而相应的整点属于 $S$, 即 $S$ 满足题中要求.
|
[] |
|
./raw_volume-zh/volume1/chapter6.tex
|
proof
|
例6. 设 $A \subset \mathrm{N}^*$ 是无限集, $A$ 中每个数 $a$ 是至多 1990 个质数的乘积.
证明: 必有.
$A$ 的无限子集 $B$, 使得 $B$ 中任何两个不同数的最大公约数都相同.
|
分析:如果 $A$ 中含有无限多个两两互质的整数, 则结论显然成立.
否则, 存在质数 $p_1$ 为 $A$ 的无限多个数的因数, 故 $A_1=\left\{\frac{a}{p_1} \mid \frac{a}{p_1} \in \mathbf{Z}, a \in A\right\}$ 为无限集.
若 $A_1$ 中含有无限多个两两互质的整数, 则结论亦成立.
否则, 继续上面的步骤.
证明如果 $A$ 中含有无限多个两两互质的正整数, 将它们全部选出作成子集 $B$, 则结论成立.
若存在质数 $p_1$ 为 $A$ 中无限多个数的因数,则集合
$$
A_1=\left\{\frac{a}{p_1} \mid \frac{a}{p_1} \in \mathbf{Z}, a \in A\right\}
$$
为无限集.
依此类推 (用 $A_1$ 代替 $A$ ). 由于 $A$ 中每个数的质因数个数 $\leqslant 1990$, 所以必有无限集
$$
A_k=\left\{\frac{a}{p_1 p_2 \cdots p_k} \mid \frac{a}{p_1 p_2 \cdots p_k} \in \mathbf{Z}, \frac{a}{p_1 p_2 \cdots p_{k-1}} \in A_{k-1}\right\},
$$
每个质数 $p_i$ 都仅是 $A_k$ 中有限多个数的因数.
任取 $a_1 \in A_k$. 在取定 $a_1, a_2, \cdots, a_n$ 两两互质后, 由于每个质数都仅是 $A_k$ 中有限多个数的因数, 在 $A_k$ 中存在 $a_{n+1}$, 它与 $a_1, a_2, \cdots, a_n$ 均互质.
这样就得到 $A_k$ 的一个无穷子集 $B_k, B_k$ 中的元素两两互质.
将 $B_k$ 中每个元素乘以 $p_1 p_2 \cdots p_k$, 得到 $A$ 的无穷子集, 其中每两个数的最大公约数均为 $p_1 p_2 \cdots p_k$.
|
[] |
|
./raw_volume-zh/volume1/chapter6.tex
|
proof
|
例7. 记 $\mathbf{Q}$ 为有理数集合, $\mathbf{Q}$ 的非空子集 $S$ 具有以下性质:
(1) $0 \notin S$;
(2) 若 $s_1 \in S, s_2 \in S$, 则 $s_1 / s_2 \in S$;
(3) 存在一非零有理数 $q, q \notin S$, 且每一个不在 $S$ 中的非零有理数都可写成 $q s$ 的形式,其中 $s \in S$.
证明: 若 $x \in S$, 则存在 $y, z \in S$, 使 $x=y+z$.
|
分析:设 $\alpha, \beta \in \mathbf{Q}$, 且 $\alpha+\beta=1$, 则
$$
x=x(\alpha+\beta)=x \alpha+x \beta .
$$
我们希望出现: $x \alpha \in S$ 且 $x \beta \in S$. 由(3)似乎应该有 $\alpha, \beta \in S$. 于是我们要解决两个问题: (1) 怎样的 $\alpha$ 、必定属于 $S$; (2) 如 $x_1 \in S, x_2 \in S$, 则 $x_1 x_2 \in S$.
证明假设 $s \in S$. 令 $s_1=s_2 \in S$, 则 $s_1 / s_2=1 \in S$. 令 $s_1=1, s_2=s$, 则 $1 / s \in S$.
若 $t \in S$, 令 $s_1=t, s_2=1 / s$, 则 $s_1 / s_2=t /(1 / s)=s t \in S$ (这样 $s$ 就是乘法意义下的解).
假设 $u$ 是一个非零有理数, 若 $u \notin S$, 则 $u=q s$, 其中 $s \in S$, 于是我们有 $u^2=q^2 s^2$.
若 $q^2 \notin S$, 则可设 $q^2=q t(t \in S)$, 则 $q=t \in S$, 矛盾.
所以 $q^2 \in S$, $u^2 \in S$.
假如 $x \in S$, 则由 $(3 / 5)^2 、(4 / 5)^2$ 为平方数可知,
$$
x(3 / 5)^2 \in S, x(4 / 5)^2 \in S .
$$
又 $x=x(3 / 5)^2+x(4 / 5)^2$, 取 $y=x(3 / 5)^2, z=x(4 / 5)^2$, 则命题得证.
|
[] |
|
./raw_volume-zh/volume1/chapter6.tex
|
proof
|
例8. 证明: 对任意的 $n \in \mathbf{N}, n \geqslant 2$, 都存在 $n$ 个互不相等的自然数组成的集合 $M$, 使得对任意的 $a \in M$ 和 $b \in M$, 都有 $(a-b) \mid(a+b)$.
|
分析:设 $a_1<a_2<\cdots<a_n$ 为 $M$ 的 $n$ 个元素, 我们用归纳的方法来构造这些元素.
当 $n=2$ 时,取 $a_1=1, a_2=2$ 即可.
假设 $n=k$ 时, $k$ 个元素 $a_1<a_2<\cdots<a_k$ 组成的集合符合要求.
当 $n=k+1$ 时则取如下 $k+1$ 个数
$$
a_{k} !, a_{k} !+a_1, a_{k} !+a_2, \cdots, a_{k} !+a_k,
$$
组成的集合符合要求.
事实上,
$$
\frac{\left(a_{k} !+a_i\right)+a_{k} !}{\left(a_{k} !+a_i\right)-a_{k} !}=\frac{2\left(a_{k} !\right)+a_i}{a_i} \in \mathbf{N}^* \quad(i=1,2, \cdots, k) .
$$
又不妨设 $i>j(i, j=1,2, \cdots, n)$, 则
$$
\begin{aligned}
A & =\frac{\left(a_{k} !+a_i\right)+\left(a_{k} !+a_j\right)}{\left(a_{k} !+a_i\right)-\left(a_{k} !+a_j\right)} \\
& =\frac{2\left(a_{k} !\right)+a_i+a_j}{a_i-a_j} .
\end{aligned}
$$
因为 $\left(a_i-a_j\right) \mid\left(a_i+a_j\right)$ (归纳假设), $\left(a_i-a_j\right) \mid 2\left(a_k\right.$ !), 所以 $A \in \mathbf{N}^*$.
说明对上面的分析稍作整理即为本例的证明.
略.
|
[] |
|
./raw_volume-zh/volume1/chapter6.tex
|
proof
|
例9. 平面上整点的集合 $M=\{(x, y) \mid x, y \in \mathbf{Z}$, 且 $1 \leqslant x \leqslant 12,1 \leqslant y \leqslant 13\}$. 证明: 不少于 49 个点的 $M$ 的每一个子集, 必包含一个矩形的 4 个顶点, 且此矩形的边平行于坐标轴.
|
分析:设 $S$ 为 $M$ 的任一个 49 元子集.
其中纵坐标相同的点的横坐标的集合为:
$$
X_i=\{x \mid(x, i) \in S\}, i=1,2, \cdots, 13 .
$$
若存在关于整点横坐标的二元集 $(r, s)$ 同时是 $X_i 、 X_j \quad(i \neq j)$ 的子集, 则原题得证.
证明设 $S$ 为 $M$ 的任一个 49 元子集.
令
$$
X_i=\{x \mid(x, i) \in S\}, i=1,2, \cdots, 13,
$$
则 $\left|X_i\right|=x_i, \sum_{i=1}^{13} x_i=49,0 \leqslant x_i \leqslant 12$. 记
$$
P_i=\{\{r, s\} \mid r \neq s,(r, i),(s, i) \in S\}, i=1,2, \cdots, 13 .
$$
显然, 全体 $P_i$ 中只有 $\mathrm{C}_{12}^2=66$ 种不同的二元集.
又 $\sum_{i=1}^{13}\left|P_i\right|=\sum_{i=1}^{13} \mathrm{C}_{x_i}^2$, 考虑其最小值.
利用局部调整: 当 $x_1+x_2=c$ 时,
$$
\mathrm{C}_{x_1}^2+\mathrm{C}_{x_2}^2=\frac{c^2-c}{2}-x_1 x_2 \geqslant \frac{c^2-c}{2}-\frac{c^2}{4},
$$
$x_1=\left[\frac{c}{2}\right], x_2=c-\left[\frac{c}{2}\right]$ 时, $\mathrm{C}_{x_1}^2+\mathrm{C}_{x_2}^2$ 取得最小值.
由此知, $\sum_{i=1}^{13} \mathrm{C}_{x_i}^2$ 取得最小值必须是将 $49=\sum_{i=1}^{13} x_i$ 尽可能地平均到 $\left\{x_i\right\}$ 中, 即 $\left\{x_i\right\}$ 中有 $j$ 个 $\left[\frac{49}{13}\right]=3$, $(13-j)$ 个 $\left[\frac{49}{13}\right]+1=4$, 从而得 $j=3$.
所以
$$
\left(\sum_{i=1}^{13} \mathrm{C}_{x_i}^2\right)_{\min }=3 \mathrm{C}_3^2+10 \mathrm{C}_4^2=69
$$
从而, 有
$$
\sum_{i=1}^{13}\left|P_i\right|=\sum_{i=1}^{13} \mathrm{C}_{x_i}^2 \geqslant 69>66
$$
由此推知存在 $i \neq j$, 使得 $(r, s) \in P_i,(r, s) \in P_j$.
故有 $(r, i),(s, i),(r, j),(s, j) \in S$, 结论成立.
|
[] |
|
./raw_volume-zh/volume1/chapter6.tex
|
proof
|
例10. 设 $S=\{1,2, \cdots, 17\}$, 而 $\left\{a_1, a_2, \cdots, a_8\right\}$ 为 $S$ 的一个 8 元子集.
求证:
(1) 存在 $k \in \mathbf{N}^*$, 使得方程 $a_i-a_j=k$ 至少有 3 组不同的解;
(2) 对于 $S$ 的 7 元子集 $\left\{a_1, a_2, \cdots, a_7\right\}$,(1) 中的结论不再总是成立.
|
分析:(1) 不妨设 $a_1<a_2<\cdots<a_8$, 则
$$
a_8-a_1=\left(a_8-a_7\right)+\left(a_7-a_6\right)+\cdots+\left(a_2-a_1\right) \leqslant 16 .
$$
若上式中间 7 个括号中没有 3 个两两相等, 那么必各有两个分别等于 1 、 $2 、 3$, 一个等于 4 .
(2) 作出一个使 (1) 中结论不成立的 7 元子集即可.
证明 (1) 若不然,则存在 $S$ 的一个 8 元子集 $\left\{a_1, a_2, \cdots, a_8\right\}$,使对任何 $k \in \mathbf{N}^*$,方程 $a_i-a_j=k$ 都至多有两组解, 即 $\left|a_i-a_j\right|(1 \leqslant i<j \leqslant 8)$ 共 28 个差数中, 不存在 3 个值相等的差数.
不妨设 $a_1<a_2<\cdots<a_8$, 于是 $a_8-a_1 \leqslant 16$, 亦即有
$$
\left(a_8-a_7\right)+\left(a_7-a_6\right)+\cdots+\left(a_2-a_1\right) \leqslant 16 .
$$
既然(1)式左端的 7 个差数中没有 3 数相同, 故必有
$$
\begin{aligned}
\left(a_8-a_7\right)+\left(a_7-a_6\right)+\cdots+\left(a_2-a_1\right) & \geqslant 1+1+2+2+3+3+4 \\
& =16 .
\end{aligned}
$$
(1)和(2)结合起来表明, 这 7 个差数中恰有 $1 、 2 、 3$ 各两个而另一个是 4 .
考察这 7 个差数的排列情形, 由于已经有两个 2 、两个 3 和 1 个 4 , 所以必有
(i) 两个 1 不能相邻, 1 和 2 也不能相邻;
(ii) 1 和 $3 、 2$ 和 2 至多有 1 组相邻.
先看两个 1 与两个 2 这 4 个差数的排列顺序, 由对称性知只有下列 4 种不同情形:
(a) $1,1,2,2$;
(b) $1,2,1,2$;
(c) $1,2,2,1$;
(d) $2,1,1,2$.
余下的 3 个差数 $3 、 3 、 4$ 将放人这 4 个数的空隙中.
易见, 在 (b) 和 (d) 两种情形中, 依次相邻的 3 对数在 7 个差数的排列中都不能相邻, 所以 3 个空隙中必须各放人 $3 、 3 、 4$ 中的一个数, 从而两个 3 都与 1 相邻, 导致有 3 个差值为 4 , 矛盾.
对于 (a), 两个 1 之间不能只夹 3 , 所以 4 必须夹在两个 1 之间.
于是 1 与 2 之间只能插人 3 . 这样一来, 两个 2 也不能相邻, 只能插人另一个 3 , 这导致 4 个差值为 5 ,矛盾.
对于 (c), 1 与 $2 、 2$ 与 1 之间不能都填 3 , 必有一个填 4 而另两个空隙中填 3 , 导致 4 个差值为 5 , 矛盾.
(2) 考察 $S$ 的 7 元子集 $\{1,2,4,7,11,16,17\}$. 它的 21 对元素的差值 (大数减小数) 中有 $1,3,5,6,9,10,15$ 各两个, $2,4,7,12,13,14,16$ 各 1 个.
没有 3 个差数有相同的值, 即 (1) 中的结论不再成立.
|
[] |
|
./raw_volume-zh/volume1/chapter7.tex
|
proof
|
例5. 证明: 任何一个三角形可以被分割成三个多边形(包括三角形), 其中之一为钝角三角形, 且能重新拼为一个矩形 (多边形允许被翻转).
|
解:若 $\triangle A B C$ 为等腰三角形(如图(<FilePath:./images/volume1/figures/fig-c7e5-1.png>)), 且 $A B=A C$, 则取底边中点 $D$ 和底边另一点 $E$, 连结顶点和底边上这两个点, 把三角形分为三部分, 易知其中 $\triangle A E C$ 为钝角三角形, 且能按照图(<FilePath:./images/volume1/figures/fig-c7e5-2.png>)拼成矩形.
若 $\triangle A B C$ 为非等腰三角形(如图(<FilePath:./images/volume1/figures/fig-c7e5-3.png>)), 不妨设 $\angle A$ 为其最大的角.
作 $A D \perp B C$ 于点 $D$, 在线段 $B D$ 上取点 $M$, 使 $M D=D C$. 设 $B M 、 A B$ 的中点分别为 $E 、 F$, 连结 $E F$. 则 $\triangle B E F 、 \triangle A D C$ 、四边形 $A D E F$ 可按照图(<FilePath:./images/volume1/figures/fig-c7e5-4.png>)拼成矩形, 且易知 $\triangle B E F$ 为钝角三角形.
在解有关整数的问题时,常常利用剩余类来分类.
|
[
"./images/volume1/figures/fig-c7e5-1.png",
"./images/volume1/figures/fig-c7e5-2.png",
"./images/volume1/figures/fig-c7e5-3.png",
"./images/volume1/figures/fig-c7e5-4.png"
] |
|
./raw_volume-zh/volume1/chapter7.tex
|
proof
|
例9. 设 $S$ 为集合 $\{1,2, \cdots, 50\}$ 的具有下列性质的子集, $S$ 中任意两个不同的元素之和不被 7 整除.
则 $S$ 中的元素最多可能有几个?
|
解:将 $\{1,2, \cdots, 50\}$ 按照模 7 分成 7 类:
$$
\begin{aligned}
& K_1=\{1,8,15,22,29,36,43,50\}, \\
& K_2=\{2,9,16,23,30,37,44\}, \\
& K_3=\{3,10,17,24,31,38,45\}, \\
& K_4=\{4,11,18,25,32,39,46\}, \\
& K_5=\{5,12,19,26,33,40,47\}, \\
& K_6=\{6,13,20,27,34,41,48\}, \\
& K_0=\{7,14,21,28,35,42,49\} .
\end{aligned}
$$
下面证明 $S=K_1 \cup K_2 \cup K_3 \cup\{7\}$ 为满足要求的元素最多的集合.
首先, 对 $a, b \in S, a \neq b$, 有 3 种可能:
(1) $a, b \in K_i(1 \leqslant i \leqslant 3)$, 则
$$
a+b \equiv 2 i(\bmod 7),
$$
有 $a+b$ 不能被 7 整除.
(2) $a \in K_i, b \in K_j(1 \leqslant i \neq j \leqslant 3)$, 则
$$
a+b \equiv i+j(\bmod 7),
$$
有 $a+b$ 不能被 7 整除.
(3) $a \in K_i, b=7(1 \leqslant i \leqslant 3)$, 则
$$
a+b=i(\bmod 7) \text {, }
$$
有 $a+b$ 不能被 7 整除.
综上知, $S$ 中任两个元素之和不能被 7 整除.
其次证明, 若给 $S$ 添加一个元素 $c$, 则必存在 $S$ 中的一个元素与 $c$ 之和, 能被 7 整除.
添加的 $c$ 有 4 种可能:
(1) $c \in K_4$, 则 $c$ 与 $K_3$ 中的元素之和能被 7 整除.
(2) $c \in K_5$, 则 $c$ 与 $K_2$ 中的元素之和能被 7 整除.
(3) $c \in K_6$, 则 $c$ 与 $K_1$ 中的元素之和能被 7 整除.
(4) $c \in K_0$, 则 $c$ 与 7 之和能被 7 整除.
综上知, $S$ 中的元素不能再增添.
所以 $S$ 中元素数目的最大值为
$$
|S|=\left|K_1\right|+\left|K_2\right|+\left|K_3\right|+1=23 .
$$
说明这里首先按模 7 的剩余类对集合 $\{1,2, \cdots, 50\}$ 的元素分类是自然的.
后面的解答中又进行了两次分类, 但这两个分类的理由已经蕴涵在最初的分类之中了.
|
[] |
|
./raw_volume-zh/volume1/chapter7.tex
|
proof
|
例10. 设 $n 、 m 、 k$ 都是自然数, 且 $m \geqslant n$. 证明: 如果
$$
1+2+\cdots+n=m k,
$$
则可将数 $1,2, \cdots, n$ 分成 $k$ 组, 使每一组数的和都等于 $m$.
|
证明:对 $n$ 进行归纳.
当 $n=1$ 时,结论显然成立.
假设对一切小于 $n$ 的自然数结论成立, 我们来考察集合 $S_n=\{1$, $2, \cdots, n\}$ 的情形.
如果 $m=n$, 那么 $\frac{1}{2}(n+1)=k$ 为整数, 于是可按如下方式分组:
$$
\{n\},\{1, n-1\},\{2, n-2\}, \cdots,\left\{-\frac{1}{2}(n-1), \frac{1}{2}(n+1)\right\} .
$$
如果 $m=n+1$, 那么 $n=2 k$ 为偶数, 则分组方式具有形式:
$$
\{1, n\},\{2, n-1\}, \cdots,\left\{\frac{n}{2}, \frac{n}{2}+1\right\} .
$$
对其余情形再分三种情况讨论:
情况 1: $n+1<m<2 n, m$ 为奇数.
我们先从 $S_n$ 中分出 $S_{m-n-1}=\{1$, $2, \cdots, m-n-1\}$; 再将其余 $2 n-m+1$ 个数两两配对, 使各对之和皆为 $m$ : $\{m-n, n\},\{m-n+1, n-1\}, \cdots,\left\{\frac{1}{2}(m-1), \frac{1}{2}(m+1)\right\}$. 由于 $S_{m-n-1}$ 中的数字之和为
$$
\begin{aligned}
& \frac{1}{2}(m-n-1)(m-n) \\
= & -\frac{1}{2}\left[m^2-m(2 n+1)\right]+\frac{1}{2} n(n+1) \\
= & m\left[\frac{1}{2}(m-2 n-1)+k\right],
\end{aligned}
$$
知该和数可被 $m$ 整除, 且因 $m \geqslant m-n-1$, 于是由归纳假设知, 可将 $S_{m-n-1}$ 中的数字分组,使得各组数字之和皆为 $m$.
情况 $2: n+1<m<2 n, m$ 为偶数.
这时, 我们仍然先从 $S_n$ 中分出 $S_{m-n-1}$
来; 并先将其余数字两两配对, 使各对数字之和为 $m:\{m-n, n\}$, $\{m-n+1, n-1\}, \cdots,\left\{\frac{m}{2}-1, \frac{m}{2}+1\right\}$, 这时还剩下一个数字 $\frac{m}{2} . S_{m \rightarrow n-1}$ 中的数字之和可以表示成 $\frac{m}{2}(m-2 n-1+2 k)$ 的形式, 它可被 $\frac{m}{2}$ 整除.
又由 $m<2 n$ 得 $m \leqslant 2 n-2, \frac{m}{2} \geqslant m-n-1$. 于是由归纳假设知, 可将 $S_{m-n-1}$ 中的数字分为 $m-2 n-1+2 k$ 组, 使每组之和皆为 $\frac{m}{2}$. 由于 $m-2 n-1+2 k$ 是一个奇数, 所以当将刚才剩下的单独一个数 $\frac{m}{2}$ 作为一组补人其中后, 即可将这些和为 $\frac{m}{2}$ 的组两两合并, 使得各组之和都成为 $m$.
情况 $3: m \geqslant 2 n$. 此时 $k=\frac{n(n+1)}{2 m} \leqslant \frac{1}{4}(n+1)$, 所以 $n-2 k \geqslant 2 k- 1>0$. 我们从 $S_n$ 中分出 $S_{n-2 k}$, 后者中的数字之和为
$$
\frac{1}{2}(n-2 k)(n-2 k+1)=\frac{1}{2} n(n+1)-k(2 n+1)+2 k^2,
$$
它可被 $k$ 整除,且所得之商不小于 $n-2 k$. 这是因为
$$
\frac{(n-2 k)(n-2 k+1)}{2(n-2 k)}=\frac{1}{2}(n-2 k+1) \geqslant k .
$$
于是由归纳假设知可将 $S_{n-2 k}$ 中的数字分为 $k$ 组, 使各组之和相等.
再将剩下的 $2 k$ 个数字两两配对, 使各对数字之和相等: $\{n-2 k+1, n\},\{n- 2 k+2, n-1\}, \cdots$ 然后再将这 $k$ 对数字分别并人前面所分出的 $k$ 组数字, 即可得到合乎需要的 $k$ 组数字.
综上, 对 $S_n$ 结论成立.
说明上述解答是在用数学归纳法证明的过程中采用分类法: 在归纳证明的第二步中, 我们对 $m$ 的取值范围分了 5 类来讨论.
|
[] |
|
./raw_volume-zh/volume1/chapter8.tex
|
proof
|
例1. 已知 $S_1 、 S_2 、 S_3$ 为非空整数集合, 且对于 $1 、 2 、 3$ 的任意一个排列 $i 、 j 、 k$, 若 $x \in S_i, y \in S_j$, 则 $x-y \in S_k$.
(1) 证明: $S_1 、 S_2 、 S_3$ 三个集合中至少有两个相等.
(2) 这三个集合中是否可能有两个集合无公共元素?
|
证明:(1)由已知,若 $x \in S_i, y \in S_j$, 则
$$
y-x \in S_k,(y-x)-y=-x \in S_i,
$$
所以每个集合中均有非负元素.
当三个集合中的元素都为零时, 命题显然成立.
否则, 设 $S_1 、 S_2 、 S_3$ 中的最小正元素为 $a$, 不妨设 $a \in S_1$. 设 $b$ 为 $S_2 、 S_3$ 中最小的非负元素, 不妨设 $b \in S_2$, 则 $b-a \in S_3$.
若 $b>0$, 则 $0 \leqslant b-a<b$, 与 $b$ 的取法矛盾.
所以 $b=0$.
任取 $x \in S_1$, 因 $0 \in S_2$, 故 $x-0=x \in S_3$, 所以 $S_1 \subseteq S_3$. 同理 $S_3 \subseteq S_1$. 故 $S_1=S_3$.
(2) 可能.
例如 $S_1=S_2=$\{ 奇数 $\} 、 S_3=\{$ 偶数 $\}$ 显然满足条件, 但 $S_1$ 和 $S_2$ 与 $S_3$ 都无公共元素.
|
[] |
|
./raw_volume-zh/volume1/chapter8.tex
|
proof
|
例2. 设 $n$ 元集合 $X$ 的某些三元子集组成集合 $S$, 且 $S$ 中每两个元素(子集)之间至多有 1 个公共元素.
试证: 存在集合 $A \subset X$, 使得 $|A| \geqslant[\sqrt{2 n}]$, 且 $S$ 中的任何元素都不是 $A$ 的子集.
|
分析:依题设 $X$ 的三元子集族 $S$ 显然没有包含 $X$ 的全部三元子集, 故存在 $X$ 的不包含 $S$ 中任何元素 ( $X$ 的三元子集) 的子集, 毫无疑问应选取其中元素最多者来做 $A$.
证明设在 $X$ 的不包含 $S$ 中任何元素的子集中, $A$ 是元素数目最多的一个, $|A|=a$. 对于每个 $x \in X-A, A \cup\{x\}$ 中必包含 $S$ 中的一个元素, 否则与 $a$ 的最大性矛盾.
设 $x, y \in X-A, x \neq y$, 则 $A \cup\{x\}$ 与 $A \cup\{y\}$ 分别包含 $S$ 中的元素 $s(x)$ 和 $s(y)$. 显然, $s(x) \neq s(y)$. 按已知, 二者至多有 1 个公共元素, 所以相应的 $A$ 中的两个二元子集也不同, 即
$$
s(x)-\{x\} \neq s(y)-\{y\} .
$$
这样一来, 我们就定义了一个由 $X-A$ 到 $A$ 的所有二元子集组成的集合的单射:
$$
X-A \ni x \longmapsto s(x)-\{x\} \subset A .
$$
从而有
$$
\begin{gathered}
n-a \leqslant \mathrm{C}_a^2, \\
a+\frac{1}{2}>\sqrt{2 n} .
\end{gathered}
$$
因为 $a \in N$, 所以 $a \geqslant[\sqrt{2 n}]$.
|
[] |
|
./raw_volume-zh/volume1/chapter8.tex
|
proof
|
例3. 某地区网球俱乐部的 20 名成员举行 14 场单打比赛, 每人至少上场一次.
求证: 必有六场比赛, 其 12 个参赛者各不相同.
|
证明:记参加第 $j$ 场比赛的选手为 $\left(a_j, b_j\right)$, 并记
$$
S=\left\{\left(a_j, b_j\right) \mid j=1,2, \cdots, 14\right\} .
$$
设 $M$ 为 $S$ 的一个子集.
如果 $M$ 中所含选手对中出现的选手互不相同, 则称 $M$ 为 $S$ 的一个“好”子集.
显然, 这样的“好”子集只有有限个, 其中必有一个元素最多的, 设这个元素最多的“好”子集为 $M_0$, 它的元素个数为 $r$, 显然只需证明 $r \geqslant 6$.
如果 $r \leqslant 5$, 由于 $M_0$ 是元素个数最多的“好” 子集, 所以在 $M_0$ 中未出现过的 $20-2r$ 名选手之间互相没有比赛, 否则与 $M_0$ 的最大性矛盾.
这就意味着, 这 $20-2 r$ 名选手所参加的比赛一定是同前 $2 r$ 名选手进行的.
由于每名选手至少参加一场比赛, 所以除了 $M_0$ 中的 $r$ 场比赛之外, 至少还要进行 $20-2 r$ 场比赛.
因此, 总比赛场数至少为
$$
r+20-2 r=20-r \geqslant 15,
$$
与总比赛场次为 14 场矛盾.
于是 $r \geqslant 6$. 问题得证.
|
[] |
|
./raw_volume-zh/volume1/chapter8.tex
|
proof
|
例4. 已知 $x_1, x_2, \cdots, x_n$ 是实数, $a_1, a_2, \cdots, a_n$ 和 $b_1, b_2, \cdots, b_n$ 均是正整数, 令
$$
\begin{aligned}
& a=\frac{a_1 x_1+a_2 x_2+\cdots+a_n x_n}{a_1+a_2+\cdots+a_n}, \\
& b=\frac{b_1 x_1+b_2 x_2+\cdots+b_n x_n}{b_1+b_2+\cdots+b_n} .
\end{aligned}
$$
求证: 在 $x_1, x_2, \cdots, x_n$ 中必存在两个数 $x_i 、 x_j$, 使 $|a-b| \leqslant \mid a - x_i|\leqslant| x_j-x_i \mid$ 成立.
|
分析:要证明存在 $x_i$ 使 $|a-b| \leqslant\left|a-x_i\right|$ 成立, 自然要在 $\left|a-x_1\right|$, $\left|a-x_2\right|, \cdots,\left|a-x_n\right|$ 中取最大者来做 $\left|a-x_i\right|$. 同样的, 对于存在 $x_i 、 x_j$ 使 $\left|a-x_i\right| \leqslant\left|x_j-x_i\right|$ 的证明, $\left|x_j-x_i\right|$ 应取 $\left|x_1-x_i\right|,\left|x_2-x_i\right|, \cdots, \left|x_n-x_i\right|$ 中最大者.
证明
$$
\begin{aligned}
& |a-b| \\
= & \left|a-\frac{b_1 x_1+b_2 x_2+\cdots+b_n x_n}{b_1+b_2+\cdots+b_n}\right| \\
= & \frac{\left|b_1\left(a-x_1\right)+b_2\left(a-x_2\right)+\cdots+b_n\left(a-x_n\right)\right|}{b_1+b_2+\cdots+b_n} \\
\leqslant & \frac{b_1\left|a-x_1\right|+b_2\left|a-x_2\right|+\cdots+b_n\left|a-x_n\right|}{b_1+b_2+\cdots+b_n} .
\end{aligned}
$$
在 $\left|a-x_1\right|,\left|a-x_2\right|, \cdots,\left|a-x_n\right|$ 中必有一个最大者, 设为 $\left|a-x_i\right|$.
则有
$$
\begin{aligned}
|a-b| & \leqslant \frac{b_1\left|a-x_i\right|+b_2\left|a-x_i\right|+\cdots+b_n\left|a-x_i\right|}{b_1+b_2+\cdots+b_n} \\
& =\frac{\left(b_1+b_2+\cdots+b_n\right)\left|a-x_i\right|}{b_1+b_2+\cdots+b_n} \\
& =\left|a-x_i\right| .
\end{aligned}
$$
下面再计算 $\left|a-x_i\right|$.
$$
\begin{aligned}
\left|a-x_i\right| & =\left|\frac{a_1 x_1+a_2 x_2+\cdots+a_n x_n}{a_1+a_2+\cdots+a_n}-x_i\right| \\
& =\frac{\left|a_1\left(x_1-x_i\right)+a_2\left(x_2-x_i\right)+\cdots+a_n\left(x_n-x_i\right)\right|}{a_1+a_2+\cdots+a_n} \\
& \leqslant \frac{a_1\left|x_1-x_i\right|+a_2\left|x_2-x_i\right|+\cdots+a_n \mid x_n-x_i \mid}{a_1+a_2+\cdots+a_n} .
\end{aligned}
$$
在 $\left|x_1-x_i\right|,\left|x_2-x_i\right|, \cdots,\left|x_n-x_i\right|$ 中必有最大者, 设为 $\left|x_j-x_i\right|$.
则
$$
\begin{aligned}
\left|a-x_i\right| & \leqslant \frac{a_1\left|x_j-x_i\right|+a_2\left|x_j-x_i\right|+\cdots+a_n\left|x_j-x_i\right|}{a_1+a_2+\cdots+a_n} \\
& =\frac{\left(a_1+a_2+\cdots+a_n\right)\left|x_j-x_i\right|}{a_1+a_2+\cdots+a_n} \\
& =\left|x_j-x_i\right| .
\end{aligned}
$$
于是, 存在 $x_i 、 x_j$, 使
$$
|a-b| \leqslant\left|a-x_i\right| \leqslant\left|x_j-x_i\right|
$$
成立.
|
[] |
|
./raw_volume-zh/volume1/chapter8.tex
|
proof
|
例5. 求方程
$$
x^4+4 y^4=2\left(z^4+4 u^4\right)
$$
的整数解.
|
分析:本例可以运用无穷递降法来解.
设 $(x, y, z, u)$ 是方程的一组解, 且其中 $x$ 是所有解中取最小正整数者, 我们就让 “无穷递降” 的过程从此开始, 看看后面会出现什么情况.
解显然, 方程(1)有解
$$
x=y=z=u=0 .
$$
我们证明这是方程(1)的惟一一组整数解.
若 ( $x, y, z, u)$ 是方程(1)的解, 则 ( $|x|, y, z, u)$ 必是方程 (1) 的解.
故不妨设 ( $x, y, z, u)$ 是方程 (1) 的所有解中 $x$ 取最小正整数者.
易知, $x$ 为偶数.
设 $x=2 x_1, x_1 \in \mathbf{N}^*$, 则有
$$
\begin{aligned}
16 x_1^4+4 y^4 & =2\left(z^4+4 u^4\right), \\
8 x_1^4+2 y^4 & =z^4+4 u^4 .
\end{aligned}
$$
因而 $z$ 是偶数.
设 $z=2 z_1, z_1 \in \mathbf{Z}$, 则有
$$
\begin{gathered}
8 x_1^4+2 y^4=16 z_1^4+4 u^4, \\
4 x_1^4+y^4=8 z_1^4+2 u^4 .
\end{gathered}
$$
因而 $y$ 是偶数.
设 $y=2 y_1, y_1 \in \mathbf{Z}$, 则有
$$
\begin{gathered}
4 x_1^4+16 y_1^4=8 z_1^4+2 u^4, \\
2 x_1^4+8 y_1^4=4 z_1^4+u^4 .
\end{gathered}
$$
因而 $u$ 是偶数.
设 $u=2 u_1, u_1 \in \mathbf{Z}$, 则有
$$
\begin{aligned}
2 x_1^4+8 y_1^4 & =4 z_1^4+16 u_1^4, \\
x_1^4+4 y_1^4 & =2\left(z_1^4+4 u_1^4\right) .
\end{aligned}
$$
由(2)知, $\left(x_1, y_1, z_1, u_1\right)$ 也是方程 (1) 的解.
但 $0<x_1<x$, 这与 $x$ 的取法矛盾.
所以,方程 (1) 有惟一解 $(0,0,0,0)$.
说明由 $(x, y, z, u)=\left(2 x_1, 2 y_1, 2 z_1, 2 u_1\right)$ 知, 若方程 (1) 有 $x=y=z=u=0$ 以外的解, 则 $x 、 y 、 z 、 u$ 至少有一个不等于零.
由于它们在方程中的次数均为偶数, 故可设其中任一个为正整数.
由上面的证法同样可导出矛盾.
这就是我们“不妨设”的理由.
|
[] |
|
./raw_volume-zh/volume1/chapter8.tex
|
proof
|
例6. 已知正整数 $a$ 和 $b$ 使得 $a b+1$ 整除 $a^2+b^2$, 求证 $\frac{a^2+b^2}{a b+1}$ 是某个正整数的平方.
|
证明:令 $A=\left\{(a, b)\left|a, b \in \mathbf{N}^*, a \geqslant b, a b+1\right| a^2+b^2\right\}$. 本题的结论是: 对所有 $(a, b) \in A$, 都有
$$
f(a, b)=\frac{a^2+b^2}{a b+1}=k^2\left(k \in \mathbf{N}^*\right) .
$$
记 $B=\left\{(a, b) \mid(a, b) \in A\right.$, 且 $\left.f(a, b) \neq k^2, k \in \mathbf{N}^*\right\}$. 我们只需证明 $B=\varnothing$.
若 $B \neq \varnothing$, 则不妨设 $B$ 中使 $a+b$ 最小的正整数对为 $(a, b)$. 令
$$
f(a, b)=\frac{a^2+b^2}{a b+1}=t\left(\neq k^2\right),
$$
则有
$$
a^2-t b a+b^2-t=0 .
$$
把(2)看作是关于 $a$ 的二次方程, 显然 $a$ 是方程(2)的一个根, 设 $c$ 为(2)的另一根, 则由韦达定理有
$$
\left\{\begin{array}{l}
a+c=t b \\
a c=b^2-t
\end{array}\right.
$$
由 (3) 知 $c$ 是整数, 由 (4) 知 $c \neq 0$.
若 $c<0$, 则由 $t>0, b>0$ 知
$$
-t c b-t \geqslant 0 \text {. }
$$
由 $c$ 是(2)的根得
$$
c^2-t c b+b^2-t=0,
$$
于是 $c^2+b^2=t c b+t \leqslant 0$. 出现矛盾.
因而 $c>0$. 由(4)知
$$
0<a c=b^2-t<b^2 \leqslant a^2,
$$
所以 $0<c<a$. 由(5)得
$$
t=\frac{c^2+b^2}{c b+1},
$$
于是 $(b, c)$ 或 $(c, b) \in B$. 但此时
$$
b+c<a+b
$$
与 $(a, b)$ 的选择即 $a+b$ 最小矛盾.
所以 $B=\varnothing$, 从而命题得证.
说明这是第 26 届 IMO 的一道试题, 曾难倒主办国不少数论专家, 但就在此次竞赛中就有选手因上面的解法而获特别奖.
此题还有另外一个用无穷递降法的证明.
另证当 $a=b$ 时, 有正整数 $q$, 使得 $\frac{2 a^2}{a^2+1}=q$, 即 $(2-q) a^2=q$.
显然, $q=1=1^2$, 此时结论成立.
由对称性, 不妨设 $a>b$.
设 $s$ 与 $t$ 是满足下列条件的整数:
$$
\left\{\begin{array}{l}
a=b s-t \\
s \geqslant 2,0 \leqslant t<b .
\end{array}\right.
$$
(1)
将 (1) 代入 $\frac{a^2}{a b} \frac{+b^2}{+1}$ 得
$$
\frac{a^2+b^2}{a b+1}=\frac{b^2 s^2-2 b s t+t^2+b^2}{b^2 s-b t+1} .
$$
考察这个数与 $s-1$ 的差
$$
\begin{aligned}
& \frac{b^2 s^2-2 b s t+t^2+b^2}{b^2 s-b t+1}-(s-1) \\
= & \frac{b^2 s-b s t+b^2+t^2-s-b t+1}{b(b s-t)+1} \\
= & \frac{s\left(b^2-b t-1\right)+b(b-t)+t^2+1}{b(b s-t)+1} .
\end{aligned}
$$
因为 $t<b$, 所以 $b-t \geqslant 1$, 从而
$$
b^2-b t-1 \geqslant 0, b-t>0, t^2+1>0,
$$
于是 (2) 式大于 0 , 即
$$
\frac{b^2 s^2-2 b s t+t^2+b^2}{b^2 s-b t+1}>s-1 .
$$
同理
$$
\frac{b^2 s^2-2 b s t+t^2+b^2}{b^2 s-b t+1}<s+1 .
$$
由于 $\frac{a^2+b^2}{a b+1}=\frac{b^2 s^2-2 b s t+t^2+b^2}{b^2 s-b t+1}$ 是整数, 所以由 (3)、(4) 可得
$$
\frac{b^2 s^2-2 b s t+t^2+b^2}{b^2 s-b t+1}=s,
$$
由此得
$$
b^2+t^2=b t s+s
$$
即
$$
\frac{b^2+t^2}{b t+1}=s=\frac{a^2+b^2}{a b+1}
$$
因为 $a>b>t$, 所以 $t=0$ 时, $s=b^2$ 为平方数; 若 $t \neq 0$, 可仿此继续下去, 经过有限步之后, 总可以使最小的数变为 0 , 所以 $s$ 是平方数, 即 $\frac{a^2+b^2}{a b+1}$ 是某个正整数的平方.
|
[] |
|
./raw_volume-zh/volume1/chapter8.tex
|
proof
|
例7. 在平面上有 $n$ 个 $(n \geqslant 2)$ 不全共线的点.
试证: 一定存在一条直线恰好通过这 $n$ 个点中的两个点.
|
分析:假设结论不成立.
不妨设其中三点 $A$ 、 $B 、 C$ 都在直线 $l$ 上, 且 $B$ 在 $A 、 C$ 之间, $D$ 为 $l$ 外一点, 如图(<FilePath:./images/volume1/figures/fig-c8e7-1.png>),作 $D P_1 \perp A C$. 不妨设 $A 、 B$ 在 $P_1$ 的同侧, 再作 $B P_2 \perp A D$. 易知 $D P_1>B P_2$. 如直线 $A D$ 上还有第三点 $E$, 不妨设 $D 、 E$ 在 $P_2$ 的同侧, 且 $D P_2>E P_2$, 作 $E P_3 \perp B D$, 则 $B P_2> E P_3$. 由假设,这个过程可以无限地进行下去, 而且每次得到的 “点到直线的距离” 都比前一次小.
另一方面, 过 $n$ 个点的每两点作一条直线 (可能有三点共线), 然后由 $n$ 个点中每一点作到这些直线的距离, 显然这样的距离只有有限个.
于是出现矛盾.
至此, 我们实际上已找到了本例的一种证明方法.
下面我们用最小数原理来改写上面的过程.
证明由 $n$ 个点中每两点作一条直线 (可能出现三点共线), 考虑 $n$ 个点中每一点到这些直线的距离所成之集, 这样的距离只有有限个, 其中必有一个最小者.
不妨设点 $P$ 到直线 $l$ 的距离最短.
下面证明: $l$ 上仅有已知点中的两个点.
若 $l$ 上有已知 $n$ 个点中的三个点, 过点 $P$ 作 $P F \perp l$ 于 $F$, 则必有两点在点 $F$ 的同侧,如图(<FilePath:./images/volume1/figures/fig-c8e7-2.png>),设点 $X$ 、 点 $Y$ 在点 $F$ 的同侧 (如图 8-2), 且 $Y F>X F$. 设过点 $P$ 与点 $Y$ 的直线为 $m$, 这时点 $X$ 到 $m$ 的距离 $X Z$ 小于点 $P$ 到 $l$ 的距离 $P F$, 与假设 $P F$ 最小矛盾.
所以, 直线 $l$ 上仅有已知点中的两个点.
$l$ 即为所求.
|
[
"./images/volume1/figures/fig-c8e7-1.png",
"./images/volume1/figures/fig-c8e7-2.png"
] |
|
./raw_volume-zh/volume1/chapter8.tex
|
proof
|
例8. 在某个星系的每一个星球上, 都有一位天文学家在观测最近的星球.
若每两个星球间的距离都不相等, 证明: 当星球的个数为奇数时, 一定有一个星球任何人都看不到.
|
证明:设有 $n$ 个星球 (同时也表示 $n$ 个天文学家) $A_1, A_2, \cdots, A_n, n$ 为奇数.
这些星球两两之间的距离所成的集合是有限集, 故必有最小值, 不妨设 $A_1 A_2$ 最小.
除 $A_1 、 A_2$ 外还有 $n-2$ 个星球和 $n-2$ 位天文学家.
假若他们当中至少有一位看见已选出的星球.
例如 $A_3$ 看见 $A_2$, 如果谁也看不见 $A_3$, 则结论成立; 否则还有一位天文学家如 $A_4$ 可看见 $A_3$. 如果谁也看不见 $A_4$, 结论同样成立; 否则还有一位天文学家如 $A_5$ 可看见 $A_4$. 仿此下去.
由于上述过程中前面星球上的天文学家看不见后面的行星, 而 $n$ 是一个有限数,必然有最后一颗星球任何人都看不到.
如果其他天文学家都看不到 $A_1 、 A_2$, 则再从 $n-2$ 颗星球中选择距离最近的两个.
依此类推.
因为 $n$ 是奇数, 所以最后存在一颗星球, 任何人都看不到它.
|
[] |
|
./raw_volume-zh/volume1/chapter8.tex
|
proof
|
例9. 平面上已给出 997 个点, 将连结每两点的线段的中点染成红色.
证明至少有 1991 个红点.
能否找到恰有 1991 个红点的点集?
|
证明:由 997 个点连结每两点的线段只有有限条, 所以必有一条最长者.
设 $A B$ 为诸线段中的最长者.
$A$ 与其他 996 个点连结的线段的中点均在以 $A$ 为圆心, $\frac{1}{2} A B$ 为半径的圆的内部或圆周上.
$B$ 与其他 996 个点连结的线段的中点均在以 $B$ 为圆心, $\frac{1}{2} A B$ 为半径的圆的内部或圆周上.
所以至少有
$$
2 \times 996-1=1991
$$
个中点, 即有 1991 个红点.
下面我们构造恰有 1991 个红点的 997 个点的点集:
在 $x$ 轴上取 997 个点,坐标分别为 $1,2, \cdots, 997$, 则区间 $(1,997)$ 内分母为 1 或 2 的有理点就是全部的红点, 个数恰为 1991 个.
|
[] |
|
./raw_volume-zh/volume1/chapter8.tex
|
proof
|
例10. 若干名儿童围成一圈, 他们手中都拿有一些糖块.
规定进行如下传递, 每次传递的方法是: 如果某人手中糖块数是奇数, 则他可再领取一块, 然后每人都把手中糖块的一半传给右边的小朋友.
求证: 一定可以经过若干次传递,使得所有儿童手中的糖块数都相同.
|
分析:由题设知, 在每次传递前, 每个儿童手中都有偶数块糖, 其中必有最多者和最少者.
证明不妨设某次传递前手中糖块数最多的人有 $2 m$ 块, 最少的有 $2 n$ 块, $m>n$. 进行一次传递后, 结果是
(1) 传递后每人手中的糖块数仍在 $2 n$ 与 $2 m$ 之间;
(2) 原来手中糖块数超过 $2 n$ 块的,传递后仍然超过 $2 n$ 块;
(3) 至少有一名原来糖块数为 $2 n$ 的孩子,传递后糖块数超过了 $2 n$.
事实上, 圈子中至少有一名拿 $2 n$ 块糖的孩子的左邻手中糖块数为 $2 h> 2 n$. 传递之后, 原拿 $2 n$ 块糖的孩子手中的糖块数变为 $n+h>2 n$.
由于每传递一次,拿 $2 n$ 块糖的孩子数至少减少 1 , 故若干次后, 将使所有孩子手中的糖块数都大于 $2 n$. 当他们都通过领取而使自己手中糖块数为偶数时,孩子手中糖块数的最小值至少上升了 2 .
由于孩子手中糖块数的最大值在传递过程中不增, 而经过若干次传递之后最小值至少上升 2 , 故知经过多次传递后总可以使最大值与最小值相等, 即所有孩子手中的糖块数都相同.
|
[] |
|
./raw_volume-zh/volume1/chapter8.tex
|
proof
|
例11. 在 $n$ 名选手参加的循环赛中, 每两人比赛一场 (无平局). 试证下列两种情形恰有一种发生:
(1) 可将所有选手分成两个非空集合,使得一个集合中的任何一名选手都战胜另一个集合中的所有选手;
(2) 可将 $n$ 名选手从 1 到 $n$ 编号, 使得第 $i$ 名选手战胜第 $i+1$ 名选手, $i=1,2, \cdots, n$, 其中将 $n+1$ 理解为 1 .
|
证明:显然, (1) 和 (2) 不能同时出现, 以下证明 (1) 和 (2) 至少有一种出现.
设选手 $A$ 胜场最多.
若 $A$ 战胜其他所有选手, 则 (1) 成立, 否则必有选手 $C$ 胜 $A$. 因 $A$ 胜场最多, 故必有负于 $A$ 的选手 $B$ 战胜 $C$, 于是得到一个选手圈 $\{A, B, C\}: A$ 胜 $B, B$ 胜 $C, C$ 胜 $A$.
设这样的圈中含选手数最多的其中之一为 $\left\{A_1, A_2, \cdots, A_m\right\}$, 其中 $A_1$ 胜 $A_2, A_2$ 胜 $A_3, \cdots, A_{m-1}$ 胜 $A_m, A_m$ 胜 $A_1$. 若 $m=n$, 则 (2) 成立.
以下设 $m<n$. 令
$$
S_1=\left\{A_1, A_2, \cdots, A_m\right\} .
$$
对任意 $B \notin S_1$, 或者 $B$ 战胜 $S_1$ 中所有选手, 或者 $B$ 负于 $S_1$ 中的所有选手.
若不然, 则存在 $A_i, A_j \in S_1$, 使 $B$ 负于 $A_i$ 而战胜 $A_j$. 不妨设 $i<j$, 从而有 $k, i \leqslant k \leqslant j-1$, 使 $B$ 负于 $A_k$ 而战胜 $A_{k+1}$. 但这将导致更长的选手圈, 矛盾.
再令
$$
\begin{aligned}
& S_2=\left\{B \mid B \text { 胜 } A_i, i=1,2, \cdots, m\right\}, \\
& S_3=\left\{B \mid B \text { 负于 } A_i, i=1,2, \cdots, m\right\} .
\end{aligned}
$$
对任意 $B \in S_2$ 与 $C \in S_3$, 若有 $C$ 胜 $B$, 则可将选手 $C$ 和 $B$ 加人 $\left\{A_1, A_2, \cdots, A_m\right\}$ 中而得到更长的圈, 矛盾, 故必有 $B$ 胜 $C$. 若 $S_2$ 非空, 则令 $S= S_2, T=S_1 \cup S_3$; 若 $S_3$ 非空, 则令 $S=S_1 \cup S_2, T=S_3$. 易见, $S$ 中的任一名选手都战胜 $T$ 中的所有选手, 即 (1) 成立.
说明上述证明中两次运用了最小数原理的推论,一次是“设选手 $A$ 胜场最多”, 导致“三怕”选手圈 $\{A, B, C\}$ 的存在; 另一次是“设这样的圈中含选手数最多的其中之一为 $\left\{A_1, A_2, \cdots, A_m\right\}$ ”, 后面的讨论都是以此为基础的.
|
[] |
|
./raw_volume-zh/volume1/chapter9.tex
|
proof
|
例4. 设 $Z$ 是平面上由 $n(>3)$ 个点组成的点集, 其中任三点不共线, 又设自然数 $k$ 满足不等式 $\frac{n}{2}<k<n$. 如果 $Z$ 中的每个点都至少与 $Z$ 中的 $k$ 个点有线段相连,证明: 这些线段中一定有三条线段构成三角形的三边.
|
证明:因为 $k>\frac{n}{2}>\frac{3}{2}$, 所以 $k \geqslant 2$, 即每个点都至少与 $Z$ 中 2 个点有线段相连.
不妨设 $A B$ 为 $Z$ 中点连成的线段.
令
$$
\begin{aligned}
& M=\{P \mid P \in Z, P \text { 与 } A \text { 有线段相连 }\}-\{B\}, \\
& N=\{P \mid P \in Z, P \text { 与 } B \text { 有线段相连 }\}-\{A\} .
\end{aligned}
$$
由于 $Z$ 中任一点至少引出 $k$ 条线段, 所以有 $|M| \geqslant k-1,|N| \geqslant k-1$. 又由于 $M \cup N$ 中不含 $A 、 B$, 所以有 $|M \cup N| \leqslant n-2$. 因此
$$
\begin{aligned}
|M \cap N| & =|M|+|N|-|M \cup N| \\
& \geqslant(k-1)+(k-1)-(n-2) \\
& =2 k-2-(n-2)
\end{aligned}
$$
$$
>(n-2)-(n-2)=0 .
$$
所以 $M \cap N \neq \varnothing$, 即存在点 $C \in M$, 且 $C \in N(C \neq A, C \neq B)$. 显然线段 $A B 、 A C 、 B C$ 构成三角形的三边.
|
[] |
|
./raw_volume-zh/volume1/chapter9.tex
|
proof
|
例7. 若 $A_1 \cup A_2 \cup \cdots \cup A_m=\left\{a_1, a_2, \cdots, a_n\right\}$, 且 $A_1, A_2, \cdots, A_m$ 均为非空集合, 则集合 $A_1, A_2, \cdots, A_m$ 的组数为
$$
g(m, n)=\sum_{k=0}^{m-1}(-1)^k \mathrm{C}_m^k\left(2^{m-k}-1\right)^n .
$$
|
证明:对于 $A_1 \cup A_2 \cdots \cup A_m=\left\{a_1, a_2, \cdots, a_n\right\}$, 如果对任意正整数 $k$ (其中 $1 \leqslant k \leqslant m-1$ ), 在 $A_1, A_2, \cdots, A_m$ 中至少有 $k$ 个集合为空集, 先确定出 $k$ 个空集, 确定的方式有 $\mathrm{C}_m^k$ 种.
对每一种方式确定出的 $k$ 个空集, 都有剩下的 $m-k$ 个集合.
不妨设它们为 $A_1^{\prime}, A_2^{\prime}, \cdots, A_{m-k}^{\prime}$, 它们的并集仍是 $\left\{a_1, a_2, \cdots, a_n\right\}$.
仿第 2 节例 6 , 知集合 $A_1^{\prime}, A_2^{\prime}, \cdots, A_{m-k}^{\prime}$ 的组数为 $\left(2^{m-k}-1\right)^n$.
即有: 在 $A_1, A_2, \cdots, A_m$ 中至少有 $k$ 个空集时, $A_1, A_2, \cdots, A_m$ 的组数是 $\mathrm{C}_m^k\left(2^{m-k}-1\right)^n$. 记
$$
\mathrm{C}_m^k\left(2^{m-k}-1\right)^n=h(m, n, k) .
$$
若 $A_1, A_2, \cdots, A_m$ 均为非空集合, 且
$$
A_1 \cup A_2 \cup \cdots \cup A_m=\left\{a_1, a_2, \cdots, a_n\right\},
$$
则由容斥原理知集合 $A_1, A_2, \cdots, A_m$ 的组数是
$$
\left(2^m-1\right)^n+\sum_{k=1}^{m-1}(-1)^k h(m, n, k) .
$$
也就是 $g(m, n)=\left(2^m-1\right)^n+\sum_{k=1}^{m-1}(-1)^k \mathrm{C}_m^k\left(2^{m-k}-1\right)^n$
$$
=\sum_{k=0}^{m-1}(-1)^k \mathrm{C}_m^k\left(2^{m-k}-1\right)^n .
$$
|
[] |
|
./raw_volume-zh/volume1/chapter9.tex
|
proof
|
例9. 设 $p_i(i=1,2, \cdots, m)$ 为正整数 $n$ 的全部质因数.
求证:
$$
\varphi(n)=n \prod_{i=1}^m\left(1-\frac{1}{p_i}\right) .
$$
|
证明:记 $S=\{1,2, \cdots, n\}$, 并设
$$
A_i=\left\{a\left|a \in S, p_i\right| a\right\}, i=1,2, \cdots, m .
$$
则 $\varphi(n)=\left|\bigcap_{i=1}^m \complement_S A_i\right|$. 注意到
$$
\begin{aligned}
& \left|A_i\right|=\left[\frac{n}{p_i}\right],\left|A_i \cap A_j\right|=\left[\frac{n}{p_i p_j}\right], \cdots, \\
& \left|A_1 \cap A_2 \cap \cdots \cap A_m\right|=\left[\frac{n}{p_1 p_2 \cdots p_m}\right],
\end{aligned}
$$
而 $p_i$ 为 $n$ 的不同的质因数, 上面各式中 [] 都可去掉, 由筛法公式得
$$
\begin{aligned}
\varphi(n) & =|S|-\sum_{i=1}^m\left[\frac{n}{p_i}\right]+\sum_{1 \leqslant i<j \leqslant m}\left[\frac{n}{p_i p_j}\right]-\cdots+(-1)^m\left[\frac{n}{p_1 p_2 \cdots p_m}\right] \\
& =n\left[1-\sum_{i=1}^m \frac{1}{p_i}+\sum_{1 \leqslant i<j \leqslant m} \frac{1}{p_i p_j}-\cdots+(-1)^m \frac{1}{p_1 p_2 \cdots p_m}\right] \\
& =n \prod_{i=1}^m\left(1-\frac{1}{p_i}\right) .
\end{aligned}
$$
容厉原理是一个非常有用的计数方法, 但我们在这里只介绍了它在解决集合和整数问题中的简单应用, 因为对那些经典的组合问题的介绍不是本书的任务.
最后我们来看一道离本书的出版日期最近的试题, 其容斥原理的运用并不是解答的全部, 但却是解题的关键一步, 因为我们的结论就是由容斥原理导出的.
|
[] |
|
./raw_volume-zh/volume1/chapter9.tex
|
proof
|
例10. 对于整数 $n \geqslant 4$, 求出最小的整数 $f(n)$, 使得对于任何正整数 $m$, 集合 $\{m, m+1, \cdots, m+n-1\}$ 的任一个 $f(n)$ 元子集中, 均有至少 3 个两两互素的元素.
|
解:当 $n \geqslant 4$ 时, 记 $M=\{m, m+1, m+2, \cdots, m+n-1\}$.
易知, 若 $2 \mid m$, 则 $m+1, m+2, m+3$ 两两互素; 若 $2 \times m$, 则 $m, m+1$, $m+2$ 两两互素.
于是, $M$ 的所有 $n$ 元子集中, 均有至少 3 个两两互素的元素, 因此 $f(n)$ 存在, 且 $f(n) \leqslant n$.
设 $T_n=\{t \mid t \leqslant n+1$ 且 $2 \mid t$ 或 $3 \mid t\}$, 则 $T_n$ 为 $\{2,3, \cdots, n+1\}$ 的子集, 但 $T_n$ 中任 3 个元素均不能两两互素, 因此 $f(n) \geqslant\left|T_n\right|+1$.
由容斥原理知
$$
\left|T_n\right|=\left[\frac{n+1}{2}\right]+\left[\frac{n+1}{3}\right]-\left[\frac{n+1}{6}\right],
$$
从而必有
$$
f(n) \geqslant\left[\frac{n+1}{2}\right]+\left[\frac{n+1}{3}\right]-\left[\frac{n+1}{6}\right]+1 .
$$
因此, $f(4) \geqslant 4, f(5) \geqslant 5, f(6) \geqslant 5, f(7) \geqslant 6, f(8) \geqslant 7, f(9) \geqslant 8$.
以下证明 $f(6)=5$.
设 $x_1, x_2, x_3, x_4, x_5$ 为 $\{m, m+1, \cdots, m+5\}$ 中的 5 个数.
若这 5 个数中有 3 个奇数,则它们两两互素; 若这 5 个数中有 2 个奇数, 则必有 3 个偶数,
不妨设 $x_1 、 x_2 、 x_3$ 为偶数, $x_4 、 x_5$ 为奇数, 当 $1 \leqslant i<j \leqslant 3$ 时, $\left|x_i-x_j\right| \in \{2,4\}$, 所以 $x_1 、 x_2 、 x_3$ 中至多一个被 3 整除, 至多一个被 5 整除, 从而至少有一个既不被 3 整除也不被 5 整除, 不妨设 $3 \nmid x_3, 5 \nmid x_3$, 则 $x_3 、 x_4 、 x_5$ 两两互素.
这就是说这 5 个数中有 3 个两两互素, 即 $f(6)=5$.
又由 $\{m, m+1, \cdots, m+n\}=\{m, m+1, \cdots, m+n-1\} \cup\{m+n\}$, 知 $f(n+1) \leqslant f(n)+1$.
因为 $f(6)=5$, 所以 $f(4)=4, f(5)=5, f(7)=6, f(8)= 7, f(9)=8$.
因此, 当 $4 \leqslant n \leqslant 9$ 时,
$$
f(n)=\left[\frac{n+1}{2}\right]+\left[\frac{n+1}{3}\right]-\left[\frac{n+1}{6}\right]+1 .
$$
以下对 $n$ 用归纳法, 证明(2)对所有 $n$ 都成立:
假设 $n \leqslant k(k \geqslant 9)$ 时(2)式成立.
当 $n=k+1$ 时, 显然
$$
\begin{gathered}
\{m, m+1, \cdots, m+k\}=\{m, m+1, \cdots, m+k-6\} \cup\{m+k-5, m+k-5+1, m+k-5+2, m+k-5+3, m+k-5+4, m+k-5+5\} .
\end{gathered}
$$
而由归纳假设知 $n=6, n=k-5$ 时(2)式成立.
所以
$$
\begin{aligned}
f(k+1) & \leqslant f(k-5)+f(6)-1 \\
& =\left[\frac{k+2}{2}\right]+\left[\frac{k+2}{3}\right]-\left[\frac{k+2}{6}\right]+1 .
\end{aligned}
$$
由(1)、(3)式知,对于 $n=k+1$, (2)式成立.
所以对于任意 $n \geqslant 4$,
$$
f(n)=\left[\frac{n+1}{2}\right]+\left[\frac{n+1}{3}\right]-\left[\frac{n+1}{6}\right]+1 .
$$
|
[] |
|
./raw_volume-zh/volume1/exercise1.tex
|
proof
|
问题13: 设 $E=\{1,2,3, \cdots, 200\}, G=\left\{a_1, a_2, a_3, \cdots, a_{100}\right\} \subseteq E$, 且 $G$ 具有下列两条性质:
(1)对任何 $1 \leqslant i<j \leqslant 100$, 恒有 $a_i+a_j \neq 201$;
(2) $\sum_{i=1}^{100} a_i=10080$.
试证明: $G$ 中的奇数的个数是 4 的倍数, 且 $G$ 中所有数字的平方和为一个定数.
|
解: 由已知得 $\sum_{k=1}^{200} k^2=\sum_{i=1}^{100} a_i^2+\sum_{i=1}^{100}\left(201-a_i\right)^2=2 \sum_{i=1}^{100} a_i^2-402 \sum_{i=1}^{100} a_i+201^2 \times 100$. 由 (2) 及上式得 $\sum_{i=1}^{100} a_i^2$ 为常数.
设 $G$ 中有 $x$ 个奇数, 则由上式可得 $4 \equiv 2 x-0+4(\bmod 8)$, 故 $x \equiv 0(\bmod 4)$.
|
[] |
|
./raw_volume-zh/volume1/exercise1.tex
|
proof
|
问题15: 考虑实数 $x$ 在 3 进制中的表达式.
$K$ 是区间 $[0,1]$ 内所有这样的数 $x$ 的集合,并且 $x$ 的每位数字是 0 或 2. 如果 $S=\{x+y \mid x, y \in K\}$, 求证: $S=\{z \mid 0 \leqslant z \leqslant 2\}=[0,2]$.
|
解: 在 $K$ 内 $x$ 和 $y$ 的每位数字是 0 或 2 , 因此, $\frac{x}{2}$ 和 $\frac{y}{2}$ 的每位数字是 0 或 1 , 从而 $\frac{x}{2}+\frac{y}{2}$ 的每位数字在 3 进制下是 $0 、 1$ 或 2 , 并且由 $x \in[0,1]$, $y \in[0,1]$ 可知 $\frac{x}{2}+\frac{y}{2} \in[0,1]$. 反过来, 对于 $[0,1]$ 上的任何一个数, 它在 3 进制下的每位数字是 $0 、 1$ 或 2 , 显然可以写成两个在 3 进制下每位数字是 0或 1 的数的和.
也就是说, 都可以写成 $\frac{x}{2}+\frac{y}{2}, x, y \in K$ 的形式.
因此, 我们有 $\left\{\frac{x}{2}+\frac{y}{2} \mid x, y \in K\right\}=[0,1]$, 故得 $S=\{x+y \mid x, y \in K\}=[0,2]$.
|
[] |
|
./raw_volume-zh/volume1/exercise1.tex
|
proof
|
问题16: 设 $S=\{1,2,3,4\}, n$ 项的数列: $a_1, a_2, \cdots, a_n$ 有如下性质: 对于 $S$ 的任何一个非空子集 $B(B$ 的元素个数记为 $|B|)$, 在该数列中有相邻的 $|B|$项恰好组成集合 $B$. 求 $n$ 的最小值.
|
解: 首先证明 $s$ 中的每个数在数列 $a_1, a_2, \cdots, a_n$ 中至少出现 2 次.
事实上, 若 $s$ 中的某个数在这个数列中只出现一次, 由于含这个数的二元子集共有 3 个, 但在数列中含这个数的相邻两项至多有两种取法, 因此不可能 3 个含这个数的二元子集都在数列相邻两项中出现.
矛盾.
由此可得, $n \geqslant 8$. 另一方面, 数列 $3,1,2,3,4,1,2,4$ 满足题设条件, 且只有 8 项.
所以, $n$ 的最小值为 8 .
|
[] |
|
./raw_volume-zh/volume1/exercise1.tex
|
proof
|
问题18: 设 $S$ 为满足下列条件的有理数集合:
(1) 若 $a \in S, b \in S$, 则 $a+b \in S, a b \in S$;
(2) 对任一个有理数 $r, 3$ 个关系 $r \in S 、-r \in S 、 r=0$ 中有且仅有一个成立.
证明: $S$ 是由全体正有理数组成的集合.
|
解: 对任意的 $r \in \mathbf{Q}, r \neq 0$, 由 (2) 知 $r \in S,-r \in S$ 之一成立.
再由 (1), 若 $r \in S$, 则 $r^2 \in S$; 若 $-r \in S$, 则 $r^2=(-r) \cdot(-r) \in S$. 总之, 对任意的非零 $r \in \mathbf{Q}$ 均有 $r^2 \in S$. 取 $r=1$, 则 $1=1^2 \in S$. 由 (1), $2=1+1 \in S, 3=1+ 2 \in S, \cdots$, 可知全体正整数都属于 $S$. 设 $p, q \in \mathbf{N}$, 由 (1), $p q \in S$. 又由前证知 $\frac{1}{q^2} \in S$, 所以 $\frac{p}{q}=p q \cdot\left(\frac{1}{q^2}\right) \in S$. 因此, $S$ 含有全体正有理数.
再由(2)知, 0 及全体负有理数不属于 $S$, 即 $S$ 是由全体正有理数组成的集合.
|
[] |
|
./raw_volume-zh/volume1/exercise1.tex
|
proof
|
问题19: $S_1 、 S_2 、 S_3$ 为非空整数集合, 对于 $1 、 2 、 3$ 的任意一个排列 $i 、 j 、 k$, 若 $x \in S_i, y \in S_j$, 则 $y-x \in S_k$.
(1) 证明: 3 个集合中至少有两个相等.
(2) 3 个集合中是否可能有两个集合无公共元素?
|
解: (1) 由已知, 若 $x \in S_i, y \in S_j$, 则 $y-x \in S_k,(y-x)-y= -x \in S_i$, 所以每个集合中均有非负元素.
当三个集合中的元素都为零时,命题显然成立.
否则, 设 $S_1 、 S_2 、 S_3$ 中的最小正元素为 $a$, 不妨设 $a \in S_1$. 设 $b$ 为 $S_2 、 S_3$ 中最小的非负元素, 不妨设 $b \in S_2$. 则 $b-a \in S_3$. 若 $b>0$, 则 $0 \leqslant b- a<b$, 与 $b$ 的取法矛盾, 所以 $b=0$. 任取 $x \in S_1$, 因 $0 \in S_2$, 故 $x-0=x \in S_3$, 所以 $S_1 \subseteq S_3$. 同理, $S_3 \subseteq S_1$. 所以 $S_1=S_3$.
(2)可能.
例如 $S_1=S_2=\{$ 奇数 $\}, S_3=\{$ 偶数 $\}$, 显然满足条件, 但 $S_1$ 和 $S_2$ 与 $S_3$ 都无公共元素.
|
[] |
|
./raw_volume-zh/volume1/exercise1.tex
|
proof
|
问题20: 若 $x \geqslant 1, x^x=x_0, x_0 \in\left(k^k,(k+1)^{(k+1)}\right) \cap \mathbf{Q}$, 其中 $k \in \mathbf{N}^*$. 求证: $x \in \mathbf{Q}^C$. (其中, $\mathbf{Q}$ 为有理数集, $\mathbf{Q}^C$ 为无理数集)
|
解: 因为当 $x_2 \geqslant x_1 \geqslant 1$ 时, 有 $x_2^{x_2} \geqslant x_1^{x_2} \geqslant x_1^{x_1}$. 所以 $y=x^x$ 在 $[1,+\infty)$ 上单调递增.
所以 $x^x=x_0$ 在 $[1,+\infty)$ 上有且仅有一解, 且 $x \in(k, k+1)$. 假设 $x^x=x_0$ 的解为有理数, 可设 $x=\frac{n}{m}, m, n \in \mathbf{N}^*,(m, n)=1$, 且 $m \neq 1$;$p, q \in \mathbf{R}^{+},\left(\frac{q}{p}\right)^m=\frac{n}{m}$. 所以 $x^x=\left(\frac{n}{m}\right)^{\frac{n}{m}}=\left(\frac{q}{p}\right)^n=x_0 \in \mathbf{Q}$. 又因为 $(m$, $n)=1$, 所以存在 $a, b \in \mathbf{Z}$, 使得 $a m+b n=1$, 所以 $\frac{q}{p}=\left(\frac{q}{p}\right)^{a m+b n}= \left[\left(\frac{q}{p}\right)^m\right]^a \cdot\left[\left(\frac{q}{p}\right)^n\right]^b \in \mathbf{Q}$. 所以不妨设 $p, q \in \mathbf{N}^*$, 且 $(p, q)=1$, 则 $\left(p^m\right.$, $\left.q^m\right)=1, \frac{n}{m}=\frac{q^m}{p^m}$. 易得 $m \mid p^m$, 且 $p^m \mid m$, 所以 $m=p^m$. 注意到 $m$ 为大于 1 的整数,矛盾.
所以 $x$ 为无理数.
|
[] |
|
./raw_volume-zh/volume1/exercise2.tex
|
proof
|
问题12 设 $\left\{a_n\right\}$ 为等差数列, $d$ 为公差, 且 $a_1$ 和 $d$ 均为实数, $d \neq 0$, 它的前 $n$ 项和记作 $S_n$. 设集合 $A=\left\{\left(a_n, \frac{S_n}{n}\right) \mid n \in \mathbf{N}^*\right\}, B=\{(x, y) \mid \frac{1}{4} x^2-y^2= 1, x, y \in \mathbf{R}\}$. 下列结论是否正确? 如果正确, 请给出证明; 如果不正确, 请举一个例子说明:
(1) 以集合 $A$ 中的元素为坐标的点都在同一直线上;
(2) $A \cap B$ 至多有一个元素;
(3) $a_1 \neq 0$ 时,一定有 $A \cap B \neq \varnothing \varnothing$.
|
(1) 正确.
因 $S_n=\frac{\left(a_1+a_n\right)^n}{2}$, 所以 $\frac{S_n}{n}=\frac{a_1+a_n}{2}$. 这说明点 $\left(a_n, \frac{S_n}{n}\right)$ 在直线 $y=\frac{a_1+x}{2}$ 上.
(2) 正确.
设 $(x, y) \in A \cap B$, 则 $y=\frac{a_1+x}{2}$, 且 $\frac{1}{4} x^2-y^2=1$. 消去 $y$, 得 $2 a_1 x+a_1^2=-4$. 当 $a_1=0$ 时, 方程组无解; 当 $a_1 \neq 0$ 时, $x=\frac{-4-a_1^2}{2 a_1}$, 方程组恰有一个解.
故 $A \cap B$ 至多有一个元素.
(3) 不正确.
举例如下: 取 $a_1=1, d=1$, 此时若有 $A \cap B \neq \varnothing$, 则存在 $(x, y) \in A \cap B$. 由 (2) 有 $x=\frac{-4-a_1^2}{2 a_1}=-\frac{5}{2}<0$. 另一方面, 对一切 $n \in \mathbf{N}^*, a_n=n>0$, 所以 $(x, y) \notin A$. 这与 $(x, y) \in A \cap B$ 矛盾.
|
[] |
|
./raw_volume-zh/volume1/exercise2.tex
|
proof
|
问题15 设 $Z$ 表示所有整数的集合.
对于固定的 $A, B, C \in \mathbf{Z}$, 令
$$
\begin{aligned}
& M_1=\left\{x^2+A x+B \mid x \in \mathbf{Z}\right\}, \\
& M_2=\left\{2 x^2+2 x+C \mid x \in \mathbf{Z}\right\},
\end{aligned}
$$
求证: 对任何 $A, B \in \mathbf{Z}$, 都可选取 $C \in \mathbf{Z}$, 使得集合 $M_1$ 与 $M_2$ 互不相交.
|
如果 $A$ 为奇数, 则有 $x(x+A)+B \equiv B(\bmod 2)$, 这表明 $M_1$ 中的所有数都与 $B$ 奇偶性相同.
对于 $M_2$ 中的数, 有 $2 x(x+1)+C \equiv C(\bmod 2)$. 可见, 为使 $M_1 \cap M_2=\varnothing$, 只须取 $C=B+1$ 即可.
如果 $A$ 为偶数, 则有 $2 x(x+1)+C \equiv C(\bmod 4)$. 又因 $\left(x+\frac{A}{2}\right)^2$ 作为完全平方数模 4 时只能为 0 或 1 , 故由 $x^2+A x+B=\left(x+\frac{A}{2}\right)^2-\left(\frac{A}{2}\right)^2+B$ 知 $M_1$ 中元素模 4 时只能与 $B 、 B+1$ 或 $B+3$ 同余.
因而, 当取 $C=B+2$ 时, $M_1 \cap M_2=\varnothing$.
|
[] |
|
./raw_volume-zh/volume1/exercise2.tex
|
proof
|
问题16 设集合 $S_n=\{1,2, \cdots, n\}$, 若 $Z$ 是 $S_n$ 的子集, 把 $Z$ 中的所有数的和称为 $Z$ 的“容量” (规定空集的容量为 0 ). 若 $Z$ 的容量为奇 (偶) 数, 则称 $Z$ 为 $S_n$ 的奇 (偶)子集.
(1) 求证: $S_n$ 的奇子集与偶子集个数相等;
(2)求证: 当 $n \geqslant 3$ 时, $S_n$ 的所有奇子集的容量之和与所有偶子集的容量之和相等;
(3)当 $n \geqslant 3$ 时, 求 $S_n$ 的所有奇子集的容量之和.
|
设 $S$ 为 $S_n$ 的奇子集, 令 $T=\left\{\begin{array}{l}S \cup\{1\}, \text { 若 } 1 \notin S, \\ S \backslash\{1\} \text {, 若 } 1 \in S .\end{array}\right.$ 则 $T$ 是偶子集, $S \rightarrow T$ 是奇子集到偶子集的一一对应, 而且对每个偶子集 $T$, 恰有一个奇子集 $S=\left\{\begin{array}{l}T \cup\{1\}, \text { 若 } 1 \notin T, \\ T \backslash\{1\}, \text { 若 } 1 \in T .\end{array}\right.$ 与之对应, 所以(1) 的结论成立.
对任一 $i(1 \leqslant i \leqslant n)$, 含 $i$ 的子集共 $2^{n-1}$ 个, 用上面的对应方法可知当 $i \neq 1$ 时, 这 $2^{n-1}$ 个集中有一半是奇子集.
当 $i=1$ 时, 由于 $n \geqslant 3$, 将上边的 1 换成 3 , 同样可得其中有一半是奇子集.
于是在计算奇子集容量之和时, 元素 $i$ 的贡献是 $2^{n-2} \cdot i$. 奇子集容量之和是 $\sum_{i=1}^n 2^{n-2} i=n(n+1) \cdot 2^{n-3}$. 由上可知, 这也是偶子集的容量之和, 两者相等.
|
[] |
|
./raw_volume-zh/volume1/exercise2.tex
|
proof
|
问题17 已知一族集合 $A_1, A_2, \cdots, A_n$ 具有性质:
(1) 每个 $A_i$ 含 30 个元素;
(2) 对每一对 $i 、 j, 1 \leqslant i<j \leqslant n, A_i \cap A_j$ 恰含有一个元素;
(3) $A_1 \cap A_2 \cap \cdots \cap A_n=\varnothing$.
求使这些集合存在的最大的正整数 $n$.
|
最大的 $n=871$. 若 $n \geqslant 872$, 则 $A_1$ 中必有一个元素 $a$ 至少属于除 $A_1$外的 30 个集合 (因 $29 \times 30+1=871<n$ ). 设 $a \notin A_i$, 每个含 $a$ 的集与 $A_i$ 有一个公共元, 故 $A_i$ 至少有 31 个元, 矛盾.
如下 871 个集满足题设: $A=\left\{a_0, a_1, \cdots, a_{29}\right\} ; B_i=\left\{a_0, a_{i, 1}\right.$, $\left.a_{i, 2}, \cdots, a_{i, 29}\right\}, 1 \leqslant i \leqslant 29 ; A_{i, j}=\left\{a_i\right\} \cup\left\{a_{k, j+(k-1)(i-1)}, k=1,2, \cdots, 29\right\}$, $1 \leqslant i, j \leqslant 29$, 其中 $a_{k, s}$ 与 $a_{k, s+29}$ 是同一个元素.
容易验证 $A \cap B_i, A \cap A_{i, j}$, $B_i \cap B_j(i \neq j), B_s \cap A_{i, j}, A_{i, j} \cap A_{i, t}(j \neq t), A_{i, j} \cap A_{s, j}(i \neq s)$ 为单元素集.
而 $A_{i, j} \cap A_{s, t}=\left\{a_{h, j+(h-1)(i-1)}\right\}, i \neq s, j \neq \bar{t}$, 其中 $h$ 是 $(x-1)(i-s) \equiv t-j(\bmod 29)$ 的惟一解.
|
[] |
|
./raw_volume-zh/volume1/exercise2.tex
|
proof
|
问题18 设 $M=\{1,2, \cdots, 20\}$, 对于 $M$ 的任一 9 元子集 $S$, 函数 $f(S)$ 取 $1 \sim 20$ 之间的整数值.
求证: 不论 $f$ 是怎样的一个函数, 总存在 $M$ 的一个 10 元子集 $T$, 使得对所有的 $k \in T$, 都有
$$
f(T-\{k\}) \neq k(T-\{k\} \text { 为 } T \text { 对 }\{k\} \text{的差集}).
$$
|
如果一个 10 元子集 $T$ 具有性质: 对任何 $k \in T$, 均有 $f(T-\{k\}) \neq k$, 我们就称 $T$ 为 “好集”. 不是 “好集” 的 10 元子集称为 “坏集”, 也就是说, 如果 $T$ 为“坏集”, 则在 $T$ 中必有一 $k_0$, 使 $f\left(T-\left\{k_0\right\}\right)=k_0$. 若令 $S=T-\left\{k_0\right\}$, 这是一个 9 元子集, 则一方面 $f(S)=k_0$, 另一方面 $T=S \cup\left\{k_0\right\}$, 即 $T=S \cup \{f(S)\}$. 上式表示了 “坏集” 的结构, 它可由某一个 9 元子集 $S$ 生成, 即 $S$ 与 $\{f(S)\}$ 的并集构成了“坏集”.
如果 $f(S) \in S$, 那么 $S \cup\{f(S)\}$ 是一个 9 元子集, 而不是 10 元子集, 即不构成“坏集”. 因此任一 9 元子集按上式至多能生成一个“坏集”(由函数的定义,对于给定的 $S, f(S)$ 是惟一的). 于是, “坏集” 的个数 $\leqslant 9$ 元子集的个数 $<10$ 元子集的个数.
最后一个不等式成立是因为, 9 元子集的个数是 $\mathrm{C}_{20}^9$, 而 10 元子集的个数是 $\mathrm{C}_{20}^{10}=\frac{11}{10} \mathrm{C}_{20}^9>\mathrm{C}_{20}^9$. 由此可知, “好集”是存在的.
|
[] |
|
./raw_volume-zh/volume1/exercise2.tex
|
proof
|
问题19 设 $k \geqslant 6$ 为自然数, $n=2 k-1$. $T$ 为所有 $n$ 元数组 $\left(x_1, x_2, \cdots, x_n\right)$ 的集合, 其中 $x_i \in\{0,1\}, i=1,2, \cdots, n$. 对于 $x=\left(x_1, x_2, \cdots, x_n\right), y= \left(y_1, y_2, \cdots, y_n\right) \in T$, 定义
$$
d(x, y)=\sum_{i=1}^n\left|x_i-y_i\right| .
$$
特别地有 $d(x, x)=0$. 设有一个由 $T$ 的 $2^k$ 个元素组成的子集 $S$, 使对任何 $x \in T$, 都存在惟一的 $y \in S$, 使得 $d(x, y) \leqslant 3$. 求证: $n=23$.
|
由于 $d(x, x)=0<3$, 所以 $S$ 中任何两个元素的距离都大于 3 . 因而当将 $S$ 中的元素 $x$ 的 $n$ 个分量改变 1 个、2 个或 3 个 (1 变为 0 或 0 变为 1 ) 时, 所得的元素都在 $T-S$ 中, 且 $S$ 中的不同元素按上述办法所得的元素也互不相同.
按假设知, 对每个 $x \in T$, 都有惟一的 $y \in S$, 使得 $d(x, y) \leqslant 3$, 所以 $T- S$ 中的每个元素都可由 $S$ 中的元素按上述办法生成.
从而有 $2^n=2^k\left(\mathrm{C}_n^0+\mathrm{C}_n^1+\right. \left.\mathrm{C}_n^2+\mathrm{C}_n^3\right)$. 由于 $n=2 k-1$, 故有 $3 \cdot 2^{k-2}=k\left(2 k^2-3 k+4\right)$. (1) 若 $3 \times k$, 则 $k= 2^m$. 由于 $k \geqslant 6$, 所以 $m \geqslant 3$. 于是 $2 k^2-3 k+4$ 是 4 的倍数但不是 8 的倍数.
从而由 (1) 可得 $2 k^2-3 k+4=12$, 这个方程无解, 所以 $k$ 为 3 的倍数.
记 $k= 3 h=3 \cdot 2^q, q \geqslant 1$, 于是 (1) 式化为 $2^{3 h-2}=h\left(18 h^2-9 h+4\right)$. (2) 当 $q \geqslant 3$ 时, $18 h^2-9 h+4$ 是 4 的倍数不是 8 的倍数.
由 (2) 知 $18 h^2-9 h+4=4$, 无解.
从而 $q=1$ 或 $2, h=2$ 或 4 . 代入 (2) 式知 $h=2$ 不是根而 $h=4$ 是根.
所以得到 $n=2 k-1==6 h-1=23$.
|
[] |
|
./raw_volume-zh/volume1/exercise2.tex
|
proof
|
问题20 设 $n \in \mathbf{N}^*$, 而 $A_1, A_2, \cdots, A_{2 n+1}$ 是集合 $B$ 的一族子集且满足条件:
(1) 每个 $A_i$ 中恰含有 $2 n$ 个元素;
(2) $A_i \cap A_j(1 \leqslant i<j \leqslant 2 n+1)$ 恰含有一个元素;
(3) $B$ 中每个元素至少属干两个子集 $A_{i_1}$ 和 $A_{i_2}, 1 \leqslant i_1<i_2 \leqslant 2 n+1$.
试问:对怎样的 $n \in \mathbf{N}^*$, 可以将 $B$ 中的每一个元素贴上一张写有 0 或 1 的标签, 使得每个 $A_i$ 中恰好有 $n$ 个元素贴有标签 0 ? 试说明理由.
|
首先, 由条件(1)-(3) 可以导出更强的条件: (3') $B$ 中每个元素恰好属于 $A_1, A_2, \cdots, A_{2 n+1}$ 中的两个.
若不然, 不妨设有 $b \in B$, 使得 $b \in A_1 \cap A_2 \cap A_3$. 因为对任何 $i \neq j, A_i \cap A_j$ 恰有一个元素, 故知 $A_1 \cap\left(\bigcup_{j=2}^{2 n+1} A_j\right)= \left[A_1 \cap\left(A_2 \cup A_3\right)\right] \cup\left[A_1 \cap\left(\bigcup_{j=4}^{2 n+1} A_j\right)\right]$ 中至多有 $2 n-1$ 个元素.
另一方面, 又由 (3) 知 $A_1$ 中每个元素至少属于另外的某个 $A_i(i \neq 1)$, 所以 $A_1 \cap\left(\bigcup_{j=2}^{2 n+1} A_j\right)$ 中又应有 $2 n$ 个元素, 矛盾.
由 $\left(3^{\prime}\right)$ 知, 对于每个 $a \in B, a$ 都对应于由两个不同正整数组成的一个数对.
具体地说, 若 $a \in A_i \cap A_j$, 则令 $a$ 与 $\{i, j\}$ 相对应.
显然, 这个对应是个双射.
若能按要求为 $B$ 中的数标上 0 和 1 , 则在上述数组中恰有一半与 0 对应.
这样的数组的个数为 $\frac{1}{2} \mathrm{C}_{2 n+1}^2=\frac{1}{2} n(2 n+1)$. 可见, $n$ 必为偶数.
用构造法证明, 把一个圆周用 $2 n+1$ 个点均分成 $2 n+1$ 等分.
在这些点依逆时针顺序标上 $1,2, \cdots, 2 n+1$. 对于任何 $1 \leqslant i<j \leqslant 2 n+1$, 看 $i$ 与 $j$ 在圆周上的劣弧, 若 $i$ 与 $j$ 之间有奇数段弧, 则给 $\{i, j\}$ 所对应的 $a \in B$ 标上数 1 ; 若有偶数段弧, 则标上数 0 . 由于 $n$ 为偶数, 因此不论 $i$ 为何值, $A_i$ 中的元素都恰有一半标有数 0 而另一半标有数 1 .
|
[] |
|
./raw_volume-zh/volume1/exercise3.tex
|
proof
|
问题9 设 $A$ 是一个正整数的集合, 且对任意 $x, y \in A$, 都有 $|x-y| \geqslant \frac{x y}{25}$. 求证: $A$ 中最多有 9 个元素.
|
设 $A=\left\{x_1, x_2, \cdots, x_n\right\}, x_{i+1}=x_i+d_i, d_i>0$. 由条件 $|x-y| \geqslant \frac{x y}{25}$ 易知, $A$ 中至多有 1 个元素不小于 25 , 从而有 $x_{n-1} \leqslant 24$. 由 $d_i=\mid x_{i+1}- x_i \mid \geqslant \frac{x_{i+1} \cdot x_i}{25}=\frac{\left(x_i+d_i\right) x_i}{25}$, 解得 $d_i \geqslant \frac{x_i^2}{25-x_i}$. 若 $x_5 \geqslant 5$, 则 $d_5 \geqslant \frac{25}{20}>1$, 故有 $x_6 \geqslant 7$. 从而 $d_6>2, x_7 \geqslant 10 ; d_7>6, x_8 \geqslant 17 ; d_8>36, x_9 \geqslant 54>25$. 可见, $A$ 中至多有 9 个元素.
另一方面, 容易验证集合 $\{1,2,3,4,5,7,10$, 17,543 满足题中要求.
从而知集合 $A$ 中最多有 9 个元素.
|
[] |
|
./raw_volume-zh/volume1/exercise3.tex
|
proof
|
问题10 对于集合 $S=\left\{\left(a_1, a_2, \cdots, a_5\right) \mid a_i=0\right.$ 或 $\left.1, i=1,2, \cdots, 5\right\}$ 中的任意两个元素 $A=\left(a_1, a_2, \cdots, a_5\right)$ 和 $B=\left(b_1, b_2, \cdots, b_5\right)$, 定义它们的距离为: $d(A, B)=\left|a_1-b_1\right|+\cdots+\left|a_5-b_5\right|$. 取 $S$ 的一个子集 $T$, 使 $T$ 中任意两个元素之间的距离都大于 2 . 问子集 $T$ 中最多含多少个元素? 证明你的结论.
|
假设有一个 5 个元素的子集也符合条件,则这 5 个元素中至少有 3 个的第一位数码相同.
不妨设 $A 、 B 、 C$ 这三个元素的第一位数码相同.
同样, 在 $A 、 B 、 C$ 中, 第二、三、四、五个数码上, 每一位都至少有两个元素的对应数码相同.
但 $A 、 B 、 C$ 三元素两两分组只有 3 组, 故至少有两个元素, 它们除第一数码相同外, 至少还有两位数码相同, 不妨设 $A$ 与 $B$, 则 $A 、 B$ 的距离不大于 2 , 矛盾.
故 $T$ 的元素不多于 4 个.
可令 $T=\{(1,1,1,1,1),(0,0,0,1$, $1),(1,1,0,0,0),(0,0,1,0,0)\}$, 则不难验证结论成立.
所以 $|T|_{\max }=4$.
|
[] |
|
./raw_volume-zh/volume1/exercise3.tex
|
proof
|
问题12 我们称一个正整数的集合 $A$ 是“一致”的,是指: 删除 $A$ 中任何一个元素之后, 剩余的元素可以分成两个不相交的子集, 而且这两个子集的元素之和相等.
求最小的正整数 $n(n>1)$, 使得可以找到一个具有 $n$ 个元素的 “一致”集合 $A$.
|
设 $A=\left\{a_1, a_2, \cdots, a_n\right\}$, 又设 $M$ 是 $A$ 中各元素之和.
根据题中的条件可以断定, 对任何 $i=1,2, \cdots, n, M-a_i$ 是偶数.
如果 $M$ 是偶数, 则 $A$ 中每个元素也都是偶数, 即 $a_i=2 b_i$, 而集合 $\left\{b_1, b_2, \cdots, b_n\right\}$ 仍然是“一致” 的.
假定 $M$ 是奇数,故对于 $i=1,2, \cdots, n, a_i$ 也都是奇数.
由于 $a_1+a_2+\cdots+ a_n=M, n$ 也是奇数.
$n=7$ 时, 容易验证集合 $A=\{1,3,5,7,9,11,13\}$ 是一致的.
假定 $n \leqslant 5$, 对于 $n=1,3$ 的情形是显然的.
设 $n=5$. 将元素按升序排列, 即 $a_1<a_2<a_3<a_4<a_5$. 将集合 $\left\{a_2, a_3, a_4, a_5\right\}$ 分成两个不相交的子集, 且使每个子集的元素之和相等,有两种方式: $a_5+a_2=a_3+a_4, a_5=a_2+a_3+ a_4$. 类似地对集合 $\left\{a_1, a_3, a_4, a_5\right\}$ 有, $a_5+a_1=a_3+a_4, a_5=a_1+a_3+a_4$. 考虑其可能的组合: 如果 $a_5+a_2=a_3+a_4$, 且 $a_5+a_1=a_3+a_4$, 则有 $a_1= a_2$, 矛盾.
如果 $a_5+a_2=a_3+a_4$, 且 $a_5=a_1+a_3+a_4$, 则有 $a_1+a_2=0$, 矛盾.
如果 $a_5=a_2+a_3+a_4$, 且 $a_5+a_1=a_3+a_4$, 则有 $a_1+a_2=0$, 矛盾.
如果 $a_5=a_2+a_3+a_4$, 且 $a_5=a_1+a_3+a_4$, 则有 $a_1=a_2$, 矛盾.
因此, $n \neq 5$, 故 $n=7$.
|
[] |
|
./raw_volume-zh/volume1/exercise3.tex
|
proof
|
问题15 设 $O x y z$ 是空间直角坐标系, $S$ 是空间中的一个由有限个点所形成的集合, $S_x 、 S_y 、 S_z$ 分别是 $S$ 中所有的点在 $O y z$ 平面、 $O z x$ 平面、 $O x y$ 平面上的正交投影所成的集合.
证明
$$
|S|^2 \leqslant\left|S_x\right| \cdot\left|S_y\right| \cdot\left|S_z\right|,
$$
其中 $|A|$ 表示有限集合 $A$ 中的元素数目.
(说明: 所谓一个点在一个平面上的正交投影是指由点向平面所作垂线的垂足.
)
|
记 $S_{i, j}$ 是 $S$ 中形如 $(x, i, j)$ 的点的集合, 即 $S$ 中在 $O y z$ 平面内正交投影坐标为 $(i, j)$ 的一切点的集合.
显然 $S=\bigcup_{(i, j) \in S_x} S_{i, j}$. 由柯西不等式, 得 $|S|^2=\left(\sum_{\left(i, j, \in S_x\right.}\left|S_{i, j}\right|\right)^2 \leqslant \sum_{(i, j) \in S_x} 1^2 \times \sum_{(i, j) \in S_x}\left|S_{i, j}\right|^2=\left|S_x\right| \sum_{(i, j) \in S_x}\left|S_{i, j}\right|^2$. 令 $X= \bigcup_{(i, j) \in S_x}\left(S_{i, j} \times S_{i, j}\right)$, 则 $|X|=\sum_{(i, j) \in S_x}\left|S_{i, j}\right|^2$.
作映射 $f: X \rightarrow S_y \times S_z, f\left((x, i, j),\left(x^{\prime}, i, j\right)\right)=\left((x, j),\left(x^{\prime}, i\right)\right)$. 显然 $f$ 为单射.
因此, $|X| \leqslant\left|S_y\right| \cdot\left|S_z\right|$. 故 $|S|^2 \leqslant\left|S_x\right| \cdot|X| \leqslant\left|S_x\right| \cdot \left|S_y\right| \cdot\left|S_z\right|$.
|
[] |
|
./raw_volume-zh/volume1/exercise3.tex
|
proof
|
问题18 设 $S$ 为平面上给定的有限整点集, $A$ 为 $S$ 的满足任两点的连线都不平行于坐标轴的元素个数最多的子集, $B$ 为整数集的满足对任意 $(x, y) \in S$, 总有 $x \in B$ 或 $y \in B$ 的元素个数最少的子集.
证明: $|A| \geqslant|B|$.
|
记 $\tau=|B|$, 从集合 $S$ 中尽可能多的去掉一些点得到 $S$ 的子集 $S^{\prime}$, 使得 $S^{\prime}$ 满足: 1) 若 $B^{\prime}$ 为 $\mathbf{Z}$ 的满足 $\forall(x, y) \in S^{\prime}$, 总有 $x \in B^{\prime}$ 或 $y \in B^{\prime}$ 的元素个数最小的子集, 则 $\left.\left|B^{\prime}\right|=\tau ; 2\right)$ 对 $\forall b \in S^{\prime}$, 若 $B^{\prime \prime}$ 是 $\mathbf{Z}$ 的满足 $\forall(x, y) \in S^{\prime} \backslash\{b\}$, 总有 $x \in B^{\prime \prime}$ 或 $y \in B^{\prime \prime}$ 的元素个数最小的子集, 则 $\left|B^{\prime \prime}\right|<\tau$. 我们要证明: $S^{\prime}$ 中任两点的连线都不平行于坐标轴, 从而 $|A| \geqslant\left|S^{\prime}\right| \geqslant \tau= |B|$.
反证: 若有 $a, b \in S^{\prime}$ 使 $a b$ 平行于 (不妨设) $x$ 轴,则 $a 、 b$ 的第二个坐标分量相同, 记为 $z$. 考虑 $S^{\prime} \backslash\{a\}$, 由 $S^{\prime}$ 的性质, 存在 $\mathbf{Z}$ 的子集 $U_a$, 使 $S^{\prime} \backslash\{a\}$ 的任一元素至少有一个坐标分量在 $U_a$ 中, 且 $\left|U_a\right|=\tau-1$ 及 $a$ 的两个坐标分量都不在 $U_a$ 中; 对于 $S^{\prime} \backslash\{b\}$, 有类似的 $U_b \subset \mathbf{Z}$. 设 $S^{\prime \prime}$ 为 $S^{\prime}$ 的子集满足: $S^{\prime \prime}$ 中任一点的两个坐标分量都在 $\{z\} \cup\left(U_a \cup U_b-U_a \cap U_b\right)$ 中, 令 $t=\left|U_a \cap U_b\right|$, 则 $S^{\prime \prime}$ 所有不同的坐标分量最多有 $2(\tau-1-t)+1$. 显然对 $S^{\prime \prime}$, 存在 $\mathbf{Z}$ 的子集 $C$, 使 $S^{\prime \prime}$ 的任意一个元素总有一个分量在 $C$ 中, 且 $|C| \leqslant \tau-1-t$.
现在令 $C^{\prime}=C \cup\left(U_a \cap U_b\right)$, 则 $S^{\prime}$ 的任一元素总有一个分量在 $C^{\prime}$ 中.
事实上, 取 $u \in S^{\prime}$, (1) $u=a$ 或 $b$, 则 $u \in S^{\prime \prime}$, 于是由 $C$ 的性质, $u$ 至少有一个分量在 $C$ 中, 从而在 $C^{\prime}$ 中; (2) $u \neq a$ 及 $b$, 则 $u$ 总有一个分量在 $U_a$ 中, 且 $u$ 总有一个分量在 $U_b$ 中.
如果 $u$ 的同一个坐标分量在 $U_a$ 和 $U_b$ 中, 则在 $U_a \cap U_b$ 中, 从而在 $C^{\prime}$ 中; 如果 $u$ 的一个坐标在 $U_a$ 中而另一个在 $U_b$ 中, 则 $u$ 在 $S^{\prime \prime}$ 中, 从而至少有一个分量在 $C$ 中, 从而在 $C^{\prime}$ 中.
故 $\tau \leqslant\left|C^{\prime}\right|=\left|C \cup\left(U_a \cap U_b\right)\right| \leqslant |C|+t \leqslant \tau-1-t+t=\tau-1$, 矛盾!
|
[] |
|
./raw_volume-zh/volume1/exercise4.tex
|
proof
|
问题1 已知集合 $\{1,2,3, \cdots, 3 n-1,3 n\}$, 可以分为 $n$ 个互不相交的三元组 $\{x, y, z\}$, 其中 $x+y=3 z$. 则满足上述要求的两个最小的正整数 $n$ 是
|
5,8 . 设三元组为 $\left\{x_i, y_i, z_i\right\}$, 且 $x_i+y_i=3 z_i, i=1,2, \cdots, n$. 则有 $\sum_{i=1}^n\left(x_i+y_i+z_i\right)=4 \sum_{i=1}^n z_1=\frac{3 n(3 n+1)}{2}$. 当 $2 \mid n$ 时, $8 \mid n$, 得 $n$ 的最小值为 8 ; 当 $2 \times n$ 时, $8 \mid 3 n+1$, 得 $n$ 的最小值为 5 . 可验证 $5 、 8$ 即为所求.
|
[] |
|
./raw_volume-zh/volume1/exercise4.tex
|
proof
|
问题3 设集合 $A \cup B=\{1,2, \cdots, 9\}, A \cap B=\varnothing$. 求证: 在 $A$ 或 $B$ 中含有三个元素 $x 、 y 、 z$, 使得 $x+z=2 y$.
|
假设结论不成立.
不妨设 $5 \in A$, 则 $1 、 9$ 不同时属于 $A$. 若 $1 \in A$, 且 $9 \in B$, 则 $3 \in B \Rightarrow 6 \in A \Rightarrow 4,7 \in B \Rightarrow 2,8 \in A$, 与 $5 \in A$ 矛盾.
若 $1,9 \in B$, 当 $7 \in A \Rightarrow 3,6 \in B$, 与 $9 \in B$ 矛盾; 当 $7 \in B \Rightarrow 8,4 \in A \Rightarrow 3 \in B \Rightarrow 2 \in A$, $2,5,8 \in A$ 矛盾.
|
[] |
|
./raw_volume-zh/volume1/exercise4.tex
|
proof
|
问题5 试证: 对于每个大于 1 的整数 $r$, 都能找到一个最小的整数 $h(r)>1$, 使在集合 $\{1,2, \cdots, h(r)\}$ 分成 $r$ 组的任何分划中,都存在整数 $a \geqslant 0,1 \leqslant x \leqslant y$, 使数 $a+x, a+y, a+x+y$ 含于分划的同一组中.
|
考察将 $\{1,2, \cdots, 2 r\}$ 分成 $r$ 组的任一分划.
在 $r, r+1, \cdots, 2 r$ 这 $r+1$ 个数中, 必有两个数 $u$ 和 $v$ 属于同一组, 不妨设 $u<v$. 令 $a=2 u-v \geqslant 0, x=y=v-u \geqslant 1$, 则 $a+x=a+y=u, a+x+y=v$ 在同一组中.
由此可见, $h(r) \leqslant 2 r$.
另一方面, 考察 $\{1,2, \cdots, 2 r-1\}$ 的如下分划: $\{1,1+r\},\{2,2+ r\}, \cdots,\{r-1,2 r-1\},\{r\}$. 显然 $a+x 、 a+y 、 a+x+y$ 不能同在 $\{r\}$ 中.
设它们都在 $\{k, k+r\}$ 中, 于是只能是 $a+x=a+y=k, a+x+y=a+ 2 x=k+r$. 从而有 $x=y=r$. 这样一来, $a=k-r<0$, 矛盾.
而且由证明可知, 当 $n<2 r$ 时, $\{1,2, \cdots, n\}$ 都不能满足要求.
综上可知, $h(r)=2 r$.
|
[] |
|
./raw_volume-zh/volume1/exercise4.tex
|
proof
|
问题6 已知整个空间被分成互不相交的 5 个非空集合, 求证: 必有一个平面, 它至少与其中的 4 个集合有公共点.
|
若不然,则任何一个平面至多与其中的 3 个集合相交.
在 5 个集合中各取 1 点, 5 点分别为 $A 、 B 、 C 、 D 、 E$, 则其中任何 4 点都不共面, 因而其中任何 3 点都不共线.
考察以 $A B$ 为公共交线的 3 个平面 $A B C 、 A B D$ 和 $A B E$, 不难看出, 其中必有一个平面, 使得另两点分别属于该平面将空间分成的两个半空间中.
不妨设点 $D$ 和 $E$ 分别位于平面 $A B C$ 的两侧.
从而直线 $D E$ 与平面 $A B C$ 相交, 记交点为 $F$. 由于 $A 、 B 、 C 3$ 点分属于 3 个集合, 而平面 $A B C$ 只与 3 个集合相交, 所以点 $F$ 必属于点 $A 、 B 、 C$ 所在的 3 个集合之一, 不妨设 $F$ 与 $A$ 属于同一个集合.
这样一来, 4 点 $D 、 F 、 E 、 B$ 所决定的平面便与 4 个集合相交,矛盾.
|
[] |
|
./raw_volume-zh/volume1/exercise4.tex
|
proof
|
问题8 (1)证明:正整数集 $\mathbf{N}^*$ 可以表示为三个彼此不相交的集合的并集,使得: 若 $m, n \in \mathbf{N}^*$, 且 $|m-n|=2$ 或 5 , 则 $m 、 n$ 属于不同的集合.
(2)证明: 正整数集 $\mathbf{N}^*$ 可以表示为 4 个彼此不相交的集合的并集,使得: 若 $m, n \in \mathbf{N}^*$, 且 $|m-n|=2 、 3$ 或 5 , 则 $m 、 n$ 属于不同的集合.
并说明:此时将 $\mathbf{N}^*$ 拆分为三个彼此不相交的集合的并集时,命题不成立.
|
(1) 令 $A=\left\{3 k \mid k \in \mathbf{N}^*\right\}, B=\left\{3 k-2 \mid k \in \mathbf{N}^*\right\}, C=\{3 k- 1 \mid k \in \mathbf{N}^*\}$, 则 $A 、 B 、 C$ 彼此的交集为空集, 且 $A \cup B \cup C=\mathbf{N}^*$, 并且此时属于同一集合的两个元素之差为 3 的倍数,不等于 2 或 5 .
(2) 令 $A=\left\{4 k \mid k \in \mathbf{N}^*\right\}, B=\left\{4 k-1 \mid k \in \mathbf{N}^*\right\}, C=\{4 k-2 \mid k \in \mathbf{N}^*\}, D=\left\{4 k-3 \mid k \in \mathbf{N}^*\right\}$. 同上讨论, 可知命题成立.
假设可以将 $\mathbf{N}^*$ 表示为三个集合 $A 、 B 、 C$ (彼此不相交) 的并集, 使得:对任意的 $m, n \in \mathbf{N}^*$, 当 $|m-n|=2 、 3$ 或 5 时, $m 、 n$ 属于不同的集合.
不妨设 $1 \in A$, 则 $3 \bar{\epsilon} A$, 从而 $3 \in B$ 或 $3 \in C$. 不妨设 $3 \in B$, 则 $6 \bar{\epsilon} A$ 且 $6 \bar{\in} B$, 故 $6 \in C$. 依此类推, 可知 $4 \in B, 8 \in A, 5 \in C, 7 \in A$. 这时, 9 属于 $A 、 B 、 C$ 中任何一个集合均导致矛盾.
|
[] |
|
./raw_volume-zh/volume1/exercise4.tex
|
proof
|
问题9 试确定所有的正整数 $n$, 使得集合 $\{1,2, \cdots, n\}$ 可以分成 5 个互不相交的子集,每个子集中元素之和相等.
|
我们先找一个必要条件, 若 $\{1,2, \cdots, n\}$ 能分成 5 个互不相交的子集, 各个子集的元素和相等, 则 $1+2+\cdots+n=\frac{n(n+1)}{2}$ 能被 5 整除, 所以 $n=5 k$ 或 $n=5 k-1$. 显然, 当 $k=1$ 时, 上述条件不是充分的.
可用数学归纳法证明, 当 $k \geqslant 2$ 时, 条件是充分的.
当 $k=2,3$, 即 $n=9,10,14,15$ 时, 可以验证结论成立:
若集合 $\{1,2, \cdots, n\}$ 能分成 5 个互不相交的子集, 且它们各自的元素和相等, 则易证 $\{1,2, \cdots, n, n+1, \cdots, n+10\}$ 也能分成 5 个互不相交的子集, 且它们每个的元素和相等.
假设对于 $n=5 k-1 、 5 k$, 命题正确, 由上面的讨论知, 对于 $n=5(k+2)-$ 1、 $5(k+2)$ 命题也成立.
|
[] |
|
./raw_volume-zh/volume1/exercise4.tex
|
proof
|
问题13 设 $S$ 为 $n$ 个正实数组成的集合, 对 $S$ 的每个非空子集 $A$, 令 $f(A)$ 为 $A$ 中所有元素的和.
求证: 集合 $\{f(A) \mid A \subseteq S, A \neq \varnothing\}$ 可以分拆为 $n$ 个互不相交的子集, 每个子集中最大数与最小数之比小于 2 .
|
设 $S=\left\{u_1, u_2, \cdots, u_n\right\}$, 且 $u_1<u_2<\cdots<u_n . u_i>0(i=1, \cdots$, $n)$. 令 $S_1=\left\{f(A) \mid A=\left\{u_1\right\}\right\} . S_k=\left\{f(A) \mid u_1+u_2+\cdots+u_{k-1}<f(A) \leqslant u_1+u_2+\cdots+u_k\right\}, k=2,3, \cdots, n$. 则 $S_1, S_2, \cdots, S_n$ 是符合要求的分拆.
事实上, 在 $S_1$ 中最大数与最小数都等于 $u_1$, 结论显然.
当 $k \geqslant 2$ 时, 若$u_{k}\leq u_{1}+u_{2}+\cdots+u_{k-1}$ ,则在 ${S_{k}}$ 中 $\frac{最大数}{最小数}<{\frac{u_{1}+\cdots+u_{k-1}+u_{k}}{u_{1}+\cdots+u_{k-1}}}<2;$ 若 $u_{k}\ >u_{1}+u_{2}+\cdots+u_{k-1}$ 则$\frac{最大数}{最小数}={\frac{u_{1}+u_{2}+\cdots+u_{k-1}+u_{k}}{u_{k}}}<2$ .
|
[] |
|
./raw_volume-zh/volume1/exercise4.tex
|
proof
|
问题15 给定集合 $S=\left\{z_1, z_2, \cdots, z_{1993}\right\}$, 其中 $z_1, z_2, \cdots, z_{1993}$ 都是非零复数 (可看作平面上的非零向量). 求证: 可以把 $S$ 中的元素分成若干组, 使得
(1) $S$ 中的每个元素属于且仅属于其中一组;
(2) 每组中的任一复数与该组中所有复数之和的夹角不超过 $90^{\circ}$;
(3) 将任意两组中的所有复数分别求和, 所得的两个和数之间的夹角大于 $90^{\circ}$.
|
考虑集合 $S$ 的所有子集并计算每个子集中所有复数的和的模.
因这样得到的模数只有有限多个, 故其中必有最大数.
将模取最大值的子集之一记为 $A$. 如果 $S-A \neq \varnothing$, 再将 $S-A$ 的所有子集中能使其中所有复数之和的模达到最大的一个子集取为 $B$. 如果 $S-(A \cup B) \neq \varnothing$, 则令 $C=S-(A \cup B)$. 这样选取的至多 3 个子集便满足题中要求.
将 $A 、 B 、 C$ 中所有元素之和分别记为 $a 、 b 、 c$.
(i) 对任意 $z \in A$, 如果 $z$ 与 $a$ 的夹角为钝角, 则 $-z$ 与 $a$ 的夹角为锐角.
于是有 $|a+(-z)|>|a|$, 即子集 $A-\{z\}$ 中所有元素之和的模大于 $a$ 的模, 此与 $|a|$ 的最大性矛盾.
这就证明了 $A$ 中任一元素与 $a$ 的夹角都不超过 $90^{\circ}$. 同理, $B$ 中任一元素与 $b$ 的夹角也不超过 $90^{\circ}$.
(ii) 对任意 $\xi \in S-A, \xi$ 与 $a$ 的夹角都是钝角.
否则又导致 $|a+\xi|>|a|$, 矛盾.
同理, $C$ 中任一元素 $\eta$ 与 $b$ 的夹角都是钝角.
由此可见, $B$ 中所有元素均与 $a$ 成钝角, 从而其和 $b$ 与 $a$ 夹钝角.
同理, $c$ 与 $a, c$ 与 $b$ 都夹钝角, 即 (3) 成立.
(iii) 若存在 $\xi \in C$, 使 $\xi$ 与 $c$ 夹针角, 则由 (ii) 知, 4 个数 $a 、 b 、 c 、 \xi$ 两两之间都夹钝角, 此不可能.
所以, $C$ 中任一元素与 $c$ 的夹角都不超过 $90^{\circ}$.
|
[] |
|
./raw_volume-zh/volume1/exercise4.tex
|
proof
|
问题16 设 $r 、 s 、 n$ 都是正整数, 并且 $r+s=n$. 求证: 集合
$$
\begin{aligned}
& A=\left\{\left[\frac{n}{r}\right],\left[\frac{2 n}{r}\right], \cdots,\left[\frac{(r-1) n}{r}\right]\right\}, \\
& B=\left\{\left[\frac{n}{s}\right],\left[\frac{2 n}{s}\right], \cdots,\left[\frac{(s-1)}{s} \underline{n}\right]\right\}
\end{aligned}
$$
构成 $N=\{1,2, \cdots, n-2\}$ 的分划的充要条件是 $r$ 和 $s$ 都与 $n$ 互质, 其中 $[x]$ 表示不超过实数 $x$ 的最大整数.
|
因 $|A|=r-1,|B|=s-1,|N|=n-2$, 故 $A$ 与 $B$ 构成 $N$ 的一个分划等价于 $A \cap B=\varnothing$.
必要性.
若 $r 、 n$ 之一与 $n$ 有公因数 $d$, 则另一个也与 $n$ 有公因数 $d$. 设 $r= r^{\prime} d, s=s^{\prime} d$, 于是 $\left[\frac{r^{\prime} n}{r}\right]=\left[\frac{s^{\prime} n}{s}\right], A \cap B \neq \varnothing$, 矛盾.
充分性.
假设 $A \cap B \neq \varnothing$, 则存在整数 $a 、 b$, 使 $\left[\frac{a n}{r}\right]=\left[\frac{b n}{s}\right]=p, p< \frac{a n}{r}<p+1, p<\frac{b n}{s}<p+1$, 即 $\frac{p r}{n}<a<\frac{(p+1) r}{n}, \frac{p s}{n}<b<\frac{(p+1) s}{n}$, 相加得 $p<a+b<p+1$. 矛盾.
|
[] |
|
./raw_volume-zh/volume1/exercise5.tex
|
proof
|
问题7 设 $S$ 为 $\{1,2, \cdots, 9\}$ 的子集, 且 $S$ 中任意两个不同的数作和, 所得的数两两不同,问: $S$ 中最多有多少个元素?
|
容易验证: 当 $S=\{1,2,3,5,8\}$ 时符合题中要求.
如果 $T \subseteq\{1$, $2, \cdots, 9\},\{T\} \geqslant 6, T$ 中任意两个不同的数之和介于 3 与 17 之间, 故至多可以形成 15 个不同的和数.
而 $T$ 中任取两个数, 有至少 $\mathrm{C}_6^2=15$ 种取法.
如果 $T$ 满足条件, 则所得和数中必须 3 与 17 同时出现, 即 $1 、 2 、 8 、 9$ 都在 $T$ 中出现, 但这时 $1+9=2+8, T$ 不合题意.
|
[] |
|
./raw_volume-zh/volume1/exercise5.tex
|
proof
|
问题8 设 $r(r \geqslant 2)$ 是一个固定的正整数, $F$ 是一个无限集合族, 且每个集合中有 $r$ 个元素.
若 $F$ 中任意两个集合的交集非空, 证明: 存在--个具有 $r^{--1}$ 个元素的集合与 $F$ 中的每一个集合的交集均非空.
|
可以证明命题: 如果集合 $A$ 的元素个数小于 $r$, 且包含于 $F$ 的无穷多个集合中, 则要么 $A$ 与 $F$ 中所有集合的交集非空, 要么存在一个 $x \notin A$, 使得 $A \cup\{x\}$, 包含于 $F$ 的无穷多个集合中.
当然,这样的集合 $A$ (如空集) 是存在的.
重复运用这个命题 $r$ 次, 即得所要证明的结论.
因为具有 $r$ 个元素的集合不可能包含于 $F$ 的无穷多个集合中.
假设 $F$ 中的某个集合 $R=\left\{x_1, x_2, \cdots, x_r\right\}$ 与集合 $A$ 的交集是空集, 由于 $F$ 中有无穷多个集合包含 $A$, 且每一个集合与 $R$ 的交集非空, 于是, 存在某个 $x_i$ 属于无穷多个集合.
设 $x=x_i$, 则 $A \cup\{x\}$ 包含于 $F$ 的无穷多个集合中.
|
[] |
|
./raw_volume-zh/volume1/exercise5.tex
|
proof
|
问题11 设 $n \in \mathbf{N}^*, n \geqslant 2, S$ 是一个 $n$ 元集合.
求最小的正整数 $k$, 使得存在 $S$ 的子集 $A_1, A_2, \cdots, A_k$ 具有如下性质:对 $S$ 中的任意两个不同元 $a 、 b$, 存在 $j \in\{1,2, \cdots, k\}$,使得 $A_j \bigcap\{a, b\}$ 为 $S$ 的一元子集.
|
构造如右表格:
\begin{tabular}{|c|c|c|c|c|c|}
\hline & 1 & 2 & 3 & $\cdots$ & $n$ \\
\hline$A_1$ & & & & & \\
\hline$A_2$ & & & & & \\
\hline$A_3$ & & & & & \\
\hline$\vdots$ & & & & & \\
\hline$A_k$ & & & & & \\
\hline & $P_1$ & $P_2$ & $P_3$ & $\cdots$ & $P_n$ \\
\hline
\end{tabular}
如果 $i \in A_j$, 那么在 $A_j$ 所在行、 $i$ 所在列处的方格中标上 1 , 其余的方格中标上 0 . 考虑表格中的列构成的序列 $P_1, P_2, \cdots, P_n$. 我们证明: $S$ 的子集 $A_1, A_2, \cdots, A_k$ 具有题中性质的充要条件是: $P_1, P_2, \cdots, P_n$ 两两不同.
若 $P_1, P_2, \cdots, P_n$ 两两不同,则对任意 $a, b \in S, a \neq b$, 有 $P_a \neq P_b$. 于是在某一行 (设为第 $j$行)上,第 $a$ 列与第 $b$ 列的方格中一个为 1 , 而另一个为 0 . 这表明 $A_j \cap\{a, b\}$ 为单元集, 故 $A_1, A_2, \cdots, A_k$ 具有题中性质.
由于对任意 $a, b \in S, a \neq b$, 存在 $j \in\{1,2, \cdots, k\}$, 使 $A_j \cap\{a, b\}$ 为单元素集, 故 $P_a$ 与 $P_b$ 在第 $j$ 行处的两个方格中的数一个为 1 , 而另一个为 0 , 故 $P_a \neq P_b$. 所以, $P_1, P_2, \cdots, P_n$ 两两不同.
由于由 $0 、 1$ 构成的 $k$ 元序列有且仅有 $2^k$ 个两两不同, 从而由抽屉原则及前面所证明的结论知 $2^k \geqslant n$. 所以, 所求的最小正整数 $k$ 为不小于 $\log _2 n$ 的最小正整数.
|
[] |
|
./raw_volume-zh/volume1/exercise6.tex
|
proof
|
问题1 已知 $S=\left\{\frac{(3 n) !}{6^n \cdot n} \mid n \in \mathbf{N}^*\right\}$, 设 $s \in S$. 求证 $: s \in \mathbf{N}^*$.
|
显然 $2^n \mid(3 n)$ !. 而 $3^n \cdot n !=3 \cdot 6 \cdot 9 \cdot \cdots \cdot 3 n$, 所以 $\left(3^n \cdot n !\right) \mid(3 n)$ !.
|
[] |
|
./raw_volume-zh/volume1/exercise6.tex
|
proof
|
问题2 数集 $M$ 由 2003 个不同的正数组成,对于 $M$ 中任何三个不同的元素 $a 、 b$ 、 $c$, 数 $a^2+b c$ 都是有理数.
证明: 可以找到一个正整数 $n$, 使得对于 $M$ 中任何数 $a$, 数 $a \sqrt{n}$ 都是有理数.
|
$a, b, c, d \in M$, 且两两不同.
由 $d^2+a b \in \mathbf{Q}, d^2+b c \in \mathbf{Q}$, 得 $b c-a b \in \mathbf{Q}$. 故 $a^2+a b=a^2+b c+(a b-b c) \in \mathbf{Q}$, 同理 $b^2+a b \in \mathbf{Q}$. 从而, 对 $M$ 中任何两个不同的数 $a 、 b$, 都有 $q=\frac{a}{b}=\frac{a^2+a b}{b^2+a b} \in \mathbf{Q}$. 于是, $a=q b$, 从而 $a^2+a b= b^2\left(q^2+q\right)=l \in \mathbf{Q}, b=\sqrt{\frac{l}{q^2+q}}=\sqrt{\frac{m}{k}}, m, k \in \mathbf{N}$. 令 $n=m k$, 得 $b \sqrt{n}=m \in \mathbf{Q}$. 故对任何 $c \in M$, 有 $c \sqrt{n}=\frac{c}{b} \cdot b \sqrt{n} \in \mathbf{Q}$.
|
[] |
|
./raw_volume-zh/volume1/exercise6.tex
|
proof
|
问题3 已知由 2003 个正数组成的集合,该集合中的任意两个数 $a$ 与 $b(a>b)$ 的和 $a+b$ 与差 $a-b$ 中至少有一个属于该集合.
证明: 若将该集合中的数按递增的顺序排列, 则相邻两个数的差相同.
|
将 2003 个正数按递增顺序排列, 并记 $A=\left\{a_1, a_2, \cdots, a_{2003}\right\}$. 因为 $\left(a_{2003}+a_i\right) \notin A$, 所以 $\left(a_{2003}-a_i\right) \in A, i=1,2, \cdots, 2002$. 即 $a_{2003}-a_{2002}< a_{2003}-a_{2001}<\cdots<a_{2003}-a_1<a_{2003} \in A$. 故 $a_{2003}-a_i=a_{2003-i}, i=1,2, \cdots$, 2002. 同理 $a_{2002}-a_i=a_{2002-i}, i=1,2, \cdots, 2001$. 所以, $a_{i+1}=a_{2003}-a_{2002-i}= a_i+a_{2003}-a_{2002}$. 从而, $a_{i+1}-a_i=a_{2003}-a_{2002}, i=1,2, \cdots, 2002$.
|
[] |
|
./raw_volume-zh/volume1/exercise6.tex
|
proof
|
问题4 设 $S=\left\{\frac{m n}{m^2+n^2} \mid m, n \in \mathbf{N}^*\right\}$. 求证: 如果 $x, y \in S$, 且 $x<y$, 那么一定存在 $z \in S$, 使得 $x<z<y$.
|
设 $x, y \in S, x=\frac{m n}{m^2+n^2}, y=\frac{a b}{a^2+b^2}, x<y$. 不妨设 $m \leqslant n, a \leqslant b$. 考虑函数 $f(x)=\frac{x}{1+x^2}$, 易证 $f(x)$ 在 $[0,1]$ 上严格递增.
所以对所有 $c, d \in[0,1]$, 有 $f(c)<f(d) \Leftrightarrow c<d$. 因为 $f\left(\frac{m}{n}\right)=\frac{m n}{m^2+n^2}<\frac{a b}{a^2+b^2}=f\left(\frac{a}{b}\right)$, 所以 $\frac{m}{n}<\frac{a}{b}$. 因此, 可选择有理数 $\frac{p}{q}$ (其中 $p, q \in \mathbf{N}^*$ ), 使得 $\frac{m}{n}<\frac{p}{q}<\frac{a}{b}$(例如取 $\left.\frac{p}{q}=-\frac{1}{2} \cdot\left(\frac{m}{n}+\frac{a}{b}\right)\right)$, 那么, 就有 $f\left(\frac{m}{n}\right)<f\left(\frac{p}{q}\right)<f\left(\frac{a}{b}\right)$. 令 $z=f \left(\frac{p}{q}\right)=\frac{p q}{p^2+q^2}$ 即可.
|
[] |
|
./raw_volume-zh/volume1/exercise6.tex
|
proof
|
问题6 设集合 $M=\left\{x_1, x_2, \cdots, x_{30}\right\}$ 由 30 个互不相同的正数组成, $A_n(1 \leqslant n \leqslant 30)$ 是 $M$ 中所有的 $n$ 个不同元素之和的和数.
证明: 若 $A_{15}>A_{10}$, 则 $A_1>1$.
|
只需证明: 如果 $A_1 \leqslant 1$, 那么对一切 $1 \leqslant n \leqslant 29$, 都有 $A_{n+1}<A_n$. 由于 $A_1 \leqslant 1$, 所以, $A_n \geqslant A_1 A_n$. 将 $A_1$ 与 $A_n$ 乘开, 并且整理以后, 可知 $A_1 A_n= A_{n+1}+S_n$, 易证 $S_n>0$, 由此即得所证.
|
[] |
|
./raw_volume-zh/volume1/exercise6.tex
|
proof
|
问题7 $S$ 为 $m$ 个无序正整数对 $(a, b)(1 \leqslant a<b \leqslant n)$ 所成的集合.
证明: 至少有 $4 m \cdot \frac{m-\frac{n^2}{4}}{3 n}$ 个无序三元数组 $(a, b, c)$, 使得 $(a, b),(b, c),(c, a)$ 都属于 $S$.
|
考虑 $n$ 个点 $1,2, \cdots, n$. 如果 $(i, j) \in S$, 则在 $i$ 与 $j$ 之间连一条线.
我们来求这个图中的三角形的个数.
设 $(i, j) \in S$, 并且自 $i$ 引出的线有 $d_i$ 条, 则以 $(i, j)$ 为边的三角形至少有 $d_i+d_j-n$ 个.
由于每个三角形有三条边, 所以 $S$ 中至少有 $\frac{1}{3} \sum_{(i, j) \in S}\left(d_i+d_j-n\right)$ (1)个三角形.
$\sum_{(i, j) \in S} 1=m, \sum_{(i, j) \in S} n=n m$. (2)
对于每个固定的 $i$, 恰有 $d_i$ 个 $j$ 使 $(i, j) \in S$, 所以在(1)中的 $d_i$ 出现了 $d_i$ 次.
注意 $(i, j)$ 既可作为自 $i$ 引出的边, 又可作为自 $j$ 引出的边, 被计算了 2 次.
因此 $\sum_{(i, j) \in S}\left(d_i+d_j\right)=\sum_{i=1}^n d_i^2$. 由柯西不等式, $\sum_{i=1}^n d_i^2 \geqslant \frac{1}{n}\left(\sum_{i=1}^n d_i\right)^2= \frac{1}{n}(2 m)^2=\frac{4 m^2}{n}$. 由(1)、(2)及上式得 $T \geqslant \frac{1}{3}\left(\frac{4 m^2}{n}-m m\right)=4 m \cdot \frac{m-\frac{n^2}{4}}{3 n}$.
|
[] |
|
./raw_volume-zh/volume1/exercise6.tex
|
proof
|
问题8 设 $L$ 是坐标平面中的一个子集.
定义如下:
$$
L=\{(41 x+2 y, 59 x+15 y) \mid x, y \in \mathbf{Z}\} .
$$
试证: 每个以原点为中心, 面积等于 1990 的平行四边形至少包含集 $L$ 中的两个点.
|
设 $F$ 是以 $(0,0),(41,59),(43,74),(2,15)$ 为顶点的平行四边形, 它的 4 个顶点都属于 $L$, 且 $F$ 中其他点都不属于 $L$. 将 $F$ 在坐标平面上向各方向平移,便形成以 $F$ 为基本区域的网络, 网络的结点都是 $L$ 中的点.
$F$ 的面积
$$
S_F=\left|\begin{array}{ccc}
0 & 0 & 1 \\
41 & 59 & 1 \\
2 & 15 & 1
\end{array}\right|=497
$$
设 $P$ 是以原点为中心, 面积等于 1990 的平行四边形.
作以原点为中心, 相似比为 $\frac{1}{2}$ 的位似变换.
记平行四边形 $P$ 的位似象为 $P^{\prime}$, 则 $P^{\prime}$ 的面积为 $1990 \times \frac{1}{4}=497 \frac{1}{2}>497$. 这样一来, 当将平行四边形 $P^{\prime}$ 被网络所分成的诸块都平移到基本区域 $F$ 中时, 必有两点重叠.
设这两点是 $D_1^{\prime}\left(x_1, y_1\right)$ 和 $D_2^{\prime}\left(x_2, y_2\right)$. 由于平移是沿网格线移动的, 所以点 $M\left(x_1-x_2, y_1-y_2\right) \in L$.
另一方面, 因为 $D_1^{\prime}, D_2^{\prime} \in P^{\prime}$, 所以 $D_1\left(2 x_1, 2 y_1\right), D_2\left(2 x_2, 2 y_2\right) \in P$. 又因 $P$ 是以原点为中心的平行四边形, 所以 $D_3\left(-2 x_2,-2 y_2\right) \in P$. 从而线段 $D_1 D_3$ 的中点 $M \in P$, 且 $M \neq(0,0)$. 这就证明了平行四边形 $P$ 中至少含有 $L$ 中的两个点.
|
[] |
|
./raw_volume-zh/volume1/exercise6.tex
|
proof
|
问题9 证明: 在集合 $\left\{1,2,3, \cdots, \frac{3^n+1}{2}\right\}\left(n \in \mathbf{N}^*\right)$ 中可取出 $2^n$ 个数, 其中无三个数成等差数列.
|
用数学归纳法.
当 $n=k+1$ 时, 可分别从 $A_1=\left\{1,2, \cdots, \frac{3^k+1}{2}\right\}$ 及 $A_2=\left\{3^k+1,3^k+2, \cdots, 3^k+\frac{3^k+1}{2}\right\}$ 中各取 $2^k$ 个数满足条件.
|
[] |
|
./raw_volume-zh/volume1/exercise6.tex
|
proof
|
问题10 设 $a_j 、 b_j 、 c_j$ 为整数, 这里 $1 \leqslant j \leqslant N$, 且对任意的 $j$, 数 $a_j 、 b_j 、 c_j$ 中至少有一个为奇数.
证明: 存在一组数 $r 、 s 、 t$, 使得集合 $ \{r a_j+s b_j+t c_j \mid 1 \leqslant j \leqslant N\}$ 中, 至少有 $\frac{4 N}{7}$ 个数为奇数.
|
考虑不全为零的 7 个数组 $(x, y, z)$, 其中 $x, y, z \in\{0,1\}$. 容易证明: 若 $a_j 、 b_j 、 c_j$ 不全为偶数, 则集合 $A_j=\{x a_j+y b_j+z c_j \mid x, y, z \in\{0,1\}\}$ 中恰有 4 个为偶数, 也恰有 4 个为奇数, 这里 $1 \leqslant j \leqslant N$. 当然, 在 $x=y= z=0$ 时, $x a_j+y b_j+z c_j$ 为偶数.
由此可知 $ \{x a_j+y b_j+z c_j \mid x, y, z \in\{0,1\}, x 、 y, z$ 不全为零, $1 \leqslant j \leqslant N\}$ 中, 恰有 $4 N$ 个数为奇数.
于是, 由抽庶原则, 可知存在一组数 $(x, y, z), x, y, z \in\{0,1\}, x 、 y 、 z$ 不全为零, 使得 $\left\{x a_j+y b_j+z c_j \mid 1 \leqslant j \leqslant N\right\}$ 中至少有 $\frac{4 N}{7}$ 个数为奇数.
|
[] |
|
./raw_volume-zh/volume1/exercise6.tex
|
proof
|
问题11 平面上不含零向量的集合 $A$, 若其至少有三个元素, 且对任意 $u \in A$, 存在 $v, w \in A$, 使 $v \neq w, u=v+w$, 则称 $A$ 具有性质 $S$. 证明:
(1) 对任意 $n \geqslant 6$, 存在具有性质 $S$ 的向量集;
(2) 具有性质 $S$ 的有限向量集合都至少有 6 个元素.
|
(1) 对 $n(n \geqslant 6)$ 进行归纳.
当 $n=6$ 时, 考虑 $\triangle A B C$ 及 $A B 、 B C 、 C A$ 、 $B A 、 C B 、 A C$. 对于具有性质 $S$ 的 $n$ 元集合 $A$, 设其非零向量为 $\boldsymbol{v}_1, \boldsymbol{v}_2, \cdots, v_n$. 设 $v_i 、 v_j$ 是 $A$ 的两个不同向量, $v_i$ 与 $v_j$ 的夹角是 $A$ 中各向量之间的最小角.
则 $\left(\boldsymbol{v}_i+\boldsymbol{v}_j\right) \notin A$, 否则与最小性矛盾.
因此, $A \cup\left\{\boldsymbol{v}_i+\boldsymbol{v}_j\right\}$ 有 $(n+1)$ 个元素, 且满足性质 $S$.
(2) 考虑一个均由 $O$ 为始点的具有性质 $S$ 的向量集合 $A= \{\boldsymbol{O} \boldsymbol{X}_1,\boldsymbol{O} \boldsymbol{X}_2, \cdots, \boldsymbol{O X _ { n }} \}$, 若 $\boldsymbol{u}$ 与 $\boldsymbol{v}$ 不平行, 且使得 $\boldsymbol{u}$ 或 $\boldsymbol{v}$ 平行于 $A$ 中的一个向量或 $\boldsymbol{X}_i \boldsymbol{X}_j (i \neq j)$ 中的一个向量.
记 $\boldsymbol{O} \boldsymbol{X}_i=a_i \boldsymbol{u}+b_i \boldsymbol{v}$, 对向量 $\boldsymbol{O} \boldsymbol{X}_i$ 分解, $i=1,2, \cdots, n$. 实数集合 $M=\left\{a_1, a_2, \cdots, a_n\right\}$ 具有类似于 $S$ 的性质.
设 $M$ 中的最大数为 $a$. 显然, $a>0$, 存在 $b 、 c>0$, 使得 $a=b+c, b \neq c$. 否则, $a$ 不是 $M$ 中的最大元素.
同理, 对于 $M$ 中的最小元素 $a^{\prime}$, 存在 $b^{\prime}, c^{\prime} \in M$, 且 $b^{\prime} 、 c^{\prime}<0, b^{\prime} \neq c^{\prime}$, 使得 $a^{\prime}=b^{\prime}+c^{\prime}$. 由此得出 $M$ 中的 6 个不同元素.
|
[] |
|
./raw_volume-zh/volume1/exercise6.tex
|
proof
|
问题12 平面上的点集 $H$ 称为是好的, 如果 $H$ 中任意 3 个点都存在一条对称轴, 使得这 3 个点关于这条对称轴对称.
证明:
(1) 一个好的集合不一定是轴对称的;
(2) 如果一个好的集合中恰有 2003 个点,则这 2003 个点在一条直线上.
|
(1) 如图(<FilePath:./images/volume1/figures/fig-c6p12-1.png>), $\triangle A B C 、 \triangle A D C 、 \triangle B C D$ 均为等腰三角形, $A 、 B 、 D$ 也共线.
所以, 任意三个点皆有一条对称轴.
故它是一个好的集合.
但是 $A 、 B$ 、 $C 、 D$ 不是轴对称的.
(2) 反证法.
假设结论不成立.
于是, 不可能有集合中的 6 个点共线.
否则, 在这条直线外必有 1 个属于集合的点 $K$, 过点 $K$ 作此直线的垂线, 则此直线上必有至少 3 个点在这条垂线的同侧, 记为 $A 、 B 、 C$ (如图(<FilePath:./images/volume1/figures/fig-c6p12-2.png>)). 因为 $\angle K C B>\frac{\pi}{2}$, 所以, $B K>B C$. 由于 $K 、 C 、 B$ 有对称轴, 则 $B C=C K$. 同理, $A C=C K$,矛盾.
故不可能有集合中的 6 个点共线.
不妨设 $A 、 B$ 为这个集合中距离最短的两个点(如图(<FilePath:./images/volume1/figures/fig-c6p12-3.png>)). 则其余 2001 个点有以下 4 种情况:(i) 在线段 $A B$ 中垂线上; (ii) 在 $A B$ 所在直线上; (iii) 在以 $A$ 为圆心、 $A B$ 长为半径的圆上; (iv) 在以 $B$ 为圆心、 $A B$ 长为半径的圆上.
由前面的证明可知, (i)、(ii) 两种情况点的总数不超过 10 个.
又因为 $A B$ 的距离最小, 所以 (iii)、(iv)两种情况点的总数不超过 10 个.
故 $10+10+2<2003$. 矛盾.
|
[
"./images/volume1/figures/fig-c6p12-1.png",
"./images/volume1/figures/fig-c6p12-2.png",
"./images/volume1/figures/fig-c6p12-3.png"
] |
|
./raw_volume-zh/volume1/exercise6.tex
|
proof
|
问题13 一个正整数的集合 $C$ 称为 “好集”, 是指对任何整数 $k$, 都存在着 $a, b \in C$, $a \neq b$, 使得数 $a+k$ 与 $b+k$ 不是互质的数.
证明: 如果一个好集 $C$ 的元素之和为 2003 , 则存在一个 $c \in C$, 使得集合 $C \backslash\{c\}$ 仍是一个“好集”.
|
设 $p_1, p_2, \cdots, p_n$ 是 $C$ 中两个数的差的所有可能的质因子.
假定对每个 $p_i$, 都存在一个剩余 $\alpha_i$, 使得 $C$ 中至多有一个数关于模 $p_i$ 与 $\alpha_i$ 同余.
利用中国剩余定理 (即孙子定理) 可得, 存在一个整数 $k$, 满足 $k \equiv p_i-\alpha_i(\bmod p_i ), i=1,2, \cdots, n$. 利用题中的条件可得, 存在某个 $j$ 和某个 $a, b \in C$, 使得 $p_j$ 整除 $a+k$ 与 $b+k$. 于是, $a$ 和 $b$ 关于模 $p_j$ 与 $\alpha_j$ 同余.
这与 $\alpha_j$ 的假定矛盾.
由此可以断定关于模 $p$ 的每个剩余, 在 $C$ 的数的剩余中至少出现两次.
假定每个剩余都恰好出现两次, 则 $C$ 中元素的和等于 $p r+2(0+1+\cdots+ p-1)=p(r+p-1), r \geqslant 1$, 这与 2003 是质数矛盾.
因此, 一定存在某个剩余, 它至少出现三次.
将具有这种性质的 $C$ 中的元素删除一个, 就得到了一一个新的“好集”.
|
[] |
BlueMO
🚀 BlueMO: A Comprehensive Collection of Challenging Mathematical Olympiad Problems from the Little Blue Book Series
BlueMO is a comprehensive and challenging dataset comprising mathematical olympiad problems paired with detailed solutions, meticulously curated from the esteemed "Little Blue Book" (小蓝书) series (Second Edition)—a vital resource for Chinese students training for national and international olympiad math competitions.
Designed to advance and assess sophisticated reasoning in LLMs, this dataset serves as a benchmark or training resource for high-level problem-solving in AI.
Introduction for "Little Blue Book" (小蓝书)
The "Little Blue Book" (小蓝书) series, published by East China Normal University Press, is a cornerstone resource for students striving to master mathematical olympiads. Renowned for its depth, challenging problems, and elegant solutions, the series spans critical domains—Sets, Trigonometric, Geometry, Number Theory, Graph Theory, and Extremal Combinatorics—providing rigorous training for olympiad competitions.
About Dataset
BlueMO encompasses a total of 14 volumes from the third edition of the "Little Blue Book" series, covering a wide range of mathematical topics for both middle and high school levels.
The dataset is structured as follows:
High School Collection:
小蓝书高中卷1 集合 - Little Blue Book High School Vol.1: Sets
小蓝书高中卷2 函数与函数方程 - Little Blue Book High School Vol.2: Functions & Functional Equations
小蓝书高中卷3 三角函数 - Little Blue Book High School Vol.3: Trigonometric Functions
小蓝书高中卷4 平均值不等式与柯西不等式 - Little Blue Book High School Vol.4: Mean Value & Cauchy Inequalities
小蓝书高中卷5 不等式的解题方法与技巧 - Little Blue Book High School Vol.5: Methods & Techniques for Solving Inequalities
小蓝书高中卷6 数列与数学归纳法 - Little Blue Book High School Vol.6: Sequences & Mathematical Induction
小蓝书高中卷7 平面几何 - Little Blue Book High School Vol.7: Plane Geometry
小蓝书高中卷8 复数与向量 - Little Blue Book High School Vol.8: Complex Numbers & Vectors
小蓝书高中卷9 几何不等式 - Little Blue Book High School Vol.9: Geometric Inequalities
小蓝书高中卷10 数论 - Little Blue Book High School Vol.10: Number Theory
小蓝书高中卷11 组合数学 - Little Blue Book High School Vol.11: Combinatorics
小蓝书高中卷12 图论 - Little Blue Book High School Vol.12: Graph Theory
小蓝书高中卷13 组合极值 - Little Blue Book High School Vol.13: Extremal Combinatorics
小蓝书高中卷14 高中数学竞赛中的解题方法与策略 - Little Blue Book High School Vol.14: Problem-Solving Methods & Strategies for Math Competitions
Potential Usages
This dataset is a resource for AI researchers and developers, with key applications including:
Training & Fine-Tuning – Enhancing large language models’ capabilities in advanced mathematical reasoning.
AI Evaluation – Benchmarking the problem-solving proficiency and logical rigor of AI systems.
Formal Verification – Formalizing problems into mathematical languages (e.g., LEAN) to evaluate AI's reasoning capability with formal methods.
Comparative Analysis – Systematically assessing reasoning skills across different models and methodologies.
How to Use
We provide the raw data (*.tex) and the processed dataset, including calculation, proof, text and images they referred to.
A case in calculation
.
{
"source_file": "./raw_volume-zh/volume1/chapter1.tex",
"problem_type": "calculation",
"problem": "例1. 设集合 $M=\\left\\{x |{\\frac{a x-5}{x^{2}-a}}<0,\\,x\\in\\mathbb{R}\\right\\}$ \n(1)当 $a=4$ 时,化简集合 $M$ ;\n(2)若 $3\\in M,$ ,且 $5\\notin M,$ 求实数a的取值范围.",
"solution": "分析: 化简集合 $M$, 实际上就是解不等式 ${\\frac{a x-5}{x^{2}-a}}<0.$ \n解: (1) 当 $a=4$ 时,有\n$$\n{\\frac{4x-5}{x^{2}-4}}<0\\,, \n$$\n即\n$$\n\\left(x-\\frac{5}{4}\\right)(x+2)(x-2)<0. \n$$\n$x<-2$ 或 ${\\frac{5}{4}}<x<2.$ \n所以 $M=(-\\infty,-2)\\cup\\bigl({\\frac{5}{4}}, 2\\bigr).$ \n(2)由 $3\\in M,$ 得 ${\\frac{3a-5}{3^{2}-a}}<0$,即 $\\left(a-\\frac{5}{3}\\right)(a-9)\\geqslant0$ ,所以\n$$\na<{\\frac{5}{3}}或a>9. \n$$\n由 $5\\notin M$ 得, ${\\frac{5a-5}{5^{2}-a}}\\geqslant0$ 或 $5^{2}-a=0$ ,所以\n$$\n1\\leq a\\leq25. \n$$\n可得 $x\\in\\left[1,{\\frac{5}{3}}\\right)\\cup\\left(9,25\\right]$.\n说明: $5\\notin M$ 隐含了条件 $5^{2}-a=$ 0,这是容易被忽视的.\n由概括原则我们知道,判断一个对象 $x$ 是否为集合 $S$ 的元素,等价于判断 $x$ 是否具有性质 $P$.",
"remark": "",
"figures": []
}
source_file
: Path to the original .tex source file containing this problem.
problem_type
: Problem category (e.g., "calculation", "proof", etc.).
problem
: Complete problem statement in LaTeX format, including sub-questions.
solution
: Step-by-step solution with mathematical derivations in LaTeX.
remark
: Additional notes or comments about the problem (empty if none).
figures
: Array containing any associated diagram files (empty if none).
Citation
If you use the BlueMO dataset in your research, please consider citing it as follows:
@misc{bluemo2024,
title={BlueMO: A comprehensive collection of challenging mathematical olympiad problems from the little blue book series},
author={Chen, Yizhou and Luo, Yifan and Zhang, Yifan and Yuan, Yang},
year={2024},
}
- Downloads last month
- 74